You are on page 1of 120

MCQ GYAIN&obstetrec

Preconception Counseling, Genetics, and Prenatal Diagnosis

1/ . A 20-year-old female presents to your office for routine well-woman examination. She has a
history of acne, for which she takes minocycline and isotretinoin on a daily basis. She also has a
history of epilepsy that is well controlled on valproic acid. She also takes a combined oral
contracep- tive birth control pill containing norethindrone acetate and ethinyl estra- diol. She is
a nonsmoker but drinks alcohol on a daily basis. She is concerned about the effectiveness of her
birth control pill, given all the medications that she takes. She is particularly worried about the
effects of her medications on a developing fetus in the event of an unintended preg- nancy.
Which of the following drugs has the lowest potential to cause birth defects?
a. Alcohol
b. Isotretinoin (Accutane)
c. Tetracyclines
d. Progesterone
e. Valproic acid (Depakote)

2/ . A patient presents for prenatal care in the second trimester. She was born outside the United
States and has never had any routine vaccinations. Which of the following vaccines is
contraindicated in pregnancy?
a. Hepatitis A
b. Tetanus
c. Typhoid
d. Hepatitis B
e. Measles

3/ . Your 25-year-old patient is pregnant at 36 weeks gestation. She has an acute urinary tract
infection (UTI). Which of the following medications is contraindicated in the treatment of the
UTI in this patient?
a. Ampicillin
b. Nitrofurantoin
c. Trimethoprim/sulfamethoxazole
d. Cephalexin
e. Amoxicillin/clavulanate
4/ . You diagnose a 21-year-old woman at 12 weeks gestation with gonorrhea cervicitis. Which of
the following is the most appropriate treatment for her infection?
a. Doxycycline
b. Chloramphenicol
c. Tetracycline
d. Minocycline
e. Ceftriaxone

5/ . A 36-year-old G0 who has been epileptic for many years is contem- plating pregnancy. She
wants to go off her phenytoin because she is con- cerned about the adverse effects that this
medication may have on her unborn fetus. She has not had a seizure in the past 5 years. Which
of the following is the most appropriate statement to make to the patient?
a. Babies born to epileptic mothers have an increased risk of structural anomalies even in
the absence of anticonvulsant medications.

1
b. She should see her neurologist to change from phenytoin to valproic acid because
valproic acid is not associated with fetal anomalies.
c. She should discontinue her phenytoin because it is associated with a 1% to 2%
risk of spina bifida.
d. Vitamin C supplementation reduces the risk of congenital anomalies in fetuses of
epileptic women taking anticonvulsants.
e. The most frequently reported congenital anomalies in fetuses of epileptic women are limb
defects.

6/ . At 1 year of age, a child has six deciduous teeth, which are discolored and have hypoplasia of
the enamel.
Match the appropriate scenario with the antibiotic most likely responsible for the clinical
findings presented.
a. Tetracycline
b. Streptomycin
c. Nitrofurantoin
d. Chloramphenicol
e. Sulfonamides

7/ . During routine auditory testing of a 2-day-old baby, the baby failed to respond to high-pitched
tones.
Match the appropriate scenario with the antibiotic most likely responsible for the clinical
findings presented.
a. Tetracycline
b. Streptomycin
c. Nitrofurantoin
d. Chloramphenicol
e. Sulfonamides

Maternal-Fetal Physiology and Placentation

8/ . A 24-year-old primigravida with twins presents for routine ultra- sonography at 20 weeks
gestation. Based on the ultrasound findings, the patient is diagnosed with dizygotic twins.
Which of the following is true regarding the membranes and placentas of dizygotic twins?
a. They are dichorionic and monoamniotic only if the fetuses are of the same sex.
b. They are dichorionic and monoamniotic regardless of the sex of the fetuses.
c. They are monochorionic and monoamniotic if they are conjoined twins.
d. They are dichorionic and diamniotic regardless of the sex of the twins.
e. They are monochorionic and diamniotic if they are of the same sex.
9/ . After delivery of a term infant with Apgar scores of 2 at 1 minute and 7 at 5 minutes, you ask
that umbilical cord blood be collected for pH. The umbilical arteries carry which of the
following?
a. Oxygenated blood to the placenta
b. Oxygenated blood from the placenta
c. Deoxygenated blood to the placenta
d. Deoxygenated blood from the placenta

2
10/ . During the routine examination of the umbilical cord and placenta after a spontaneous vaginal
delivery, you notice that the baby had only one umbilical artery. Which of the following is true
regarding the finding of a single umbilical artery?
a. It is a very common finding and is insignificant.
b. It is a rare finding in singleton pregnancies and is therefore not significant.
c. It is an indicator of an increased incidence of congenital anomalies of the fetus.
d. It is equally common in newborns of diabetic and nondiabetic mothers.
e. It is present in 5% of all births.

11/ . A 22-year-old G1P0 at 28 weeks gestation by LMP presents to labor and delivery complaining
of decreased fetal movement. She has had no pre- natal care. On the fetal monitor there are no
contractions. The fetal heart rate is 150 beats per minute and reactive. There are no
decelerations in the fetal heart tracing. An ultrasound is performed in the radiology department
and shows a 28-week fetus with normal-appearing anatomy and size con- sistent with dates. The
placenta is implanted on the posterior uterine wall and its margin is well away from the cervix.
A succenturiate lobe of the pla- centa is seen implanted low on the anterior wall of the uterus.
Doppler flow studies indicate a blood vessel is traversing the cervix connecting the two lobes.
This patient is most at risk for which of the following?
a. Premature rupture of the membranes
b. Fetal exsanguination after rupture of the membranes
c. Torsion of the umbilical cord caused by velamentous insertion of the umbilical cord
d. Amniotic fluid embolism
e. Placenta accreta

12/ . A healthy 34-year-old G1P0 patient comes to see you in your office for a routine OB visit at 12
weeks gestational age. She tells you that she has stopped taking her prenatal vitamins with iron
supplements because they make her sick and she has trouble remembering to take a pill every
day. A review of her prenatal labs reveals that her hematocrit is 39%. Which of the following
statements is the best way to counsel this patient?
a. Tell the patient that she does not need to take her iron supplements because her prenatal
labs indicate that she is not anemic and therefore she will not absorb the iron supplied in
prenatal vitamins
b. Tell the patient that if she consumes a diet rich in iron, she does not need to take any iron
supplements
c. Tell the patient that if she fails to take her iron supplements, her fetus will be anemic
d. Tell the patient that she needs to take the iron supplements even though she is not anemic
in order to meet the demands of pregnancy
e. Tell the patient that she needs to start retaking her iron supplements when her
hemoglobin falls below 11 g/dL
13/ . A pregnant patient of yours goes to the emergency room at 20 weeks gestational age with
complaints of hematuria and back pain. The emer- gency room physician orders an intravenous
pyelogram (IVP) as part of a workup for a possible kidney stone. The radiologist indicates the
absence of nephrolithiasis but reports the presence of bilateral hydronephrosis and hydroureter,
which is greater on the right side than on the left. Which of the following statements is true
regarding this IVP finding?
a. The bilateral hydronephrosis is of concern, and renal function tests, including
BUN and creatinine, should be run and closely monitored.
b. These findings are consistent with normal pregnancy and are not of concern.

3
c. The bilateral hydronephrosis is of concern, and a renal sonogram should be ordered
emergently.
d. The findings indicate that a urology consult is needed to obtain recommenda- tions for
further workup and evaluation.
e. The findings are consistent with ureteral obstruction, and the patient should be referred
for stent placement.
14/ . During a routine return OB visit, an 18-year-old G1P0 patient at 23 weeks gestational age
undergoes a urinalysis. The dipstick done by the nurse indicates the presence of trace
glucosuria. All other parameters of the urine test are normal. Which of the following is the most
likely etiology of the increased sugar detected in the urine?
a. The patient has diabetes.
b. The patient has a urine infection.
c. The patient’s urinalysis is consistent with normal pregnancy.
d. The patient’s urine sample is contaminated.
e. The patient has kidney disease.

15/ . A 33-year-old G2P1 is undergoing an elective repeat cesarean section at term. The infant is
delivered without any difficulties, but the placenta cannot be removed easily because a clear
plane between the placenta and uterine wall cannot be identified. The placenta is removed in
pieces. This is followed by uterine atony and hemorrhage.
Match the descriptions with the appropriate placenta type.
a. Succenturiate placenta
b. Vasa previa
c. Placenta previa
d. Membranaceous placenta
e. Placenta accreta

Antepartum Care and Fetal Surveillance

16/ . The shortest distance between the sacral promontory and the symphysis pubis is called which of
the following?
a. Interspinous diameter
b. True conjugate
c. Diagonal conjugate
d. Obstetric (OB) conjugate
e. Biparietal diameter
17/ . A patient presents in labor at term. Clinical pelvimetry is performed. She has an oval-shaped
pelvis with the anteroposterior diameter at the pelvic inlet greater than the transverse diameter.
The baby is occiput posterior. The patient most likely has what kind of pelvis?
a. A gynecoid pelvis
b. An android pelvis
c. An anthropoid pelvis
d. A platypelloid pelvis
e. An androgenous pelvis
18/ . On pelvic examination of a patient in labor at 34 weeks, the patient is noted to be 6 cm dilated,
completely effaced with the fetal nose and mouth palpable. The chin is pointing toward the

4
maternal left hip. This is an example of which of the following?
a. Transverse lie
b. Mentum transverse position
c. Occiput transverse position
d. Brow presentation
e. Vertex presentation

19/ . A patient comes to your office with her last menstrual period 4 weeks ago. She denies any
symptoms such as nausea, fatigue, urinary frequency, or breast tenderness. She thinks that she
may be pregnant because she has not had her period yet. She is very anxious to find out because
she has a history of a previous ectopic pregnancy and wants to be sure to get early prenatal care.
Which of the following actions is most appropriate at this time?
a. No action is needed because the patient is asymptomatic, has not missed her period, and
cannot be pregnant.
b. Order a serum quantitative pregnancy test.
c. Listen for fetal heart tones by Doppler equipment.
d. Perform an abdominal ultrasound.
e. Perform a bimanual pelvic examination to assess uterine size
.
20/ . A patient presents for her first initial OB visit after performing a home pregnancy test and gives
a last menstrual period of about 8 weeks ago. She says she is not entirely sure of her dates,
however, because she has a long history of irregular menses. Which of the following is the most
accurate way of dating the pregnancy?
a. Determination of uterine size on pelvic examination
b. Quantitative serum human chorionic gonadotropin (HCG) level
c. Crown-rump length on abdominal or vaginal ultrasound
c. Determination of progesterone level along with serum HCG level
d. Quantification of a serum estradiol level

21/ . A healthy 31-year-old G3P2002 patient presents to the obstetrician’s office at 34 weeks
gestational age for a routine return visit. She has had an uneventful pregnancy to date. Her
baseline blood pressures were 100 to 110/60 to70, and she has gained a total of 20 lb so far.
During the visit, the patient complains of bilateral pedal edema that sometimes causes her feet
to ache at the end of the day. Her urine dip indicates trace protein, and her blood pressure in the
office is currently 115/75. She denies any other symptoms or complaints. On physical
examination, there is pitting edema of both legs without any calf tenderness. Which of the
following is the most appropriate response to the patient’s concern?
a. Prescribe Lasix to relieve the painful swelling.
b. Immediately send the patient to the radiology department to have venous.
Doppler studies done to rule out deep vein thromboses.
c. Admit the patient to L and D to rule out preeclampsia.
d. Reassure the patient that this is a normal finding of pregnancy and no treatment is
needed.
e. Tell the patient that her leg swelling is caused by too much salt intake and instruct her to
go on a low-sodium diet.
22/ . A 28-year-old G1P0 presents to your office at 18 weeks gestational age for an unscheduled visit
secondary to right-sided groin pain. She describes the pain as sharp and occurring with
movement and exercise. She denies any change in urinary or bowel habits. She also denies any

5
fever or chills. The application of a heating pad helps alleviate the discomfort. As her
obstetrician, what should you tell this patient is the most likely etiology of this pain?
a. Round ligament pain
b. Appendicitis
c. Preterm labor
d. Kidney stone
e. Urinary tract infection
23/ . A 19-year-old G1P0 presents to her obstetrician’s office for a routine OB visit at 32 weeks
gestation. Her pregnancy has been complicated by gestational diabetes requiring insulin for
control. She has been noncompli- ant with diet and insulin therapy. She has had two prior
normal ultra- sounds at 20 and 28 weeks gestation. She has no other significant past medical or
surgical history. During the visit, her fundal height measures 38 cm. Which of the following is
the most likely explanation for the discrepancy between the fundal height and the gestational
age?
a. Fetal hydrocephaly
b. Uterine fibroids
c. Polyhydramnios
d. Breech presentation
e. Undiagnosed twin gestation

24/ . A 30-year-old G2P1001 patient comes to see you in the office at 37 weeks gestational age for
her routine OB visit. Her first pregnancy resulted in a vagi- nal delivery of a 9-lb 8-oz baby boy
after 30 minutes of pushing. On doing Leopold maneuvers during this office visit, you
determine that the fetus is breech. Vaginal examination demonstrates that the cervix is 50%
effaced and 1 to 2 cm dilated. The presenting breech is high out of the pelvis. The esti- mated
fetal weight is about 7 lb. The patient denies having any contractions. You send the patient for a
sonogram, which confirms a fetus with a double footling breech presentation. There is a normal
amount of amniotic fluid present and the head is hyperextended in the “stargazer” position.
Which of the following is the best next step in the management of this patient?
a. Allow the patient to undergo a vaginal breech delivery whenever she goes into labor.
b. Send the patient to labor and delivery immediately for an emergent cesarean section.
c. Schedule a cesarean section at or after 41 weeks gestational age.
d. Schedule an external cephalic version in the next few days.
e. Allow the patient to go into labor and do an external cephalic version at that time if the
fetus is still in the double footling breech presentation.
25/ . A 29-year-old G1P0 presents to the obstetrician’s office at 41 weeks gestation. On physical
examination, her cervix is 1 centimeter dilated, 0% effaced, firm, and posterior in position. The
vertex is presenting at –3 station. Which of the following is the best next step in the
management of this patient?
a. Send the patient to the hospital for induction of labor since she has a favorable
Bishop score.
b. Teach the patient to measure fetal kick counts and deliver her if at any time there are less
than 20 perceived fetal movements in 3 hours.
c. Order BPP testing for the same or next day.
d. Schedule the patient for induction of labor at 43 weeks gestation.
e. Schedule cesarean delivery for the following day since it is unlikely that the patient will
go into labor.
26/ . Your patient had an ultrasound examination today at 39 weeks gestation for size less than dates.

6
The ultrasound showed oligohydramnios with an amniotic fluid index of 1.5 centimeters. The
patient’s cervix is unfavorable. Which of the following is the best next step in the management
of this patient?
a. Admit her to the hospital for cesarean delivery.
b. Admit her to the hospital for cervical ripening then induction of labor.
c. Write her a prescription for misoprostol to take at home orally every 4 hours until she
goes into labor.
d. Perform stripping of the fetal membranes and perform a BPP in 2 days.
e. Administer a cervical ripening agent in your office and have the patient present to the
hospital in the morning for induction with oxytocin.

27/ . An 18-year-old G2P1001 with the first day of her last menstrual period of May 7 presents for
her first OB visit at 10 weeks. What is this patient’s estimated date of delivery?
a. February 10 of the next year
b. February 14 of the next year
c. December 10 of the same year
d. December 14 of the same year
e. December 21of the same year
28/ . A new patient presents to your office for her first prenatal visit. By her last menstrual period she
is 11 weeks pregnant. This is the first pregnancy for this 36-year-old woman. She has no
medical problems. At this visit you observe that her uterus is palpable midway between the
pubic symphysis and the umbilicus. No fetal heart tones are audible with the Doppler
stethoscope. Which of the following is the best next step in the manage- ment of this patient?
a. Reassure her that fetal heart tones are not yet audible with the Doppler stetho- scope at
this gestational age.
b. Tell her the uterine size is appropriate for her gestational age and schedule her for routine
ultrasonography at 20 weeks.
c. Schedule genetic amniocentesis right away because of her advanced maternal age.
d. Schedule her for a dilation and curettage because she has a molar pregnancy since her
uterus is too large and the fetal heart tones are not audible.
e. Schedule an ultrasound as soon as possible to determine the gestational age and viability
of the fetus.
29/ . A 16-year-old primigravida presents to your office at 35 weeks gesta- tion. Her blood pressure
is 170/110 mm Hg and she has 4+ proteinuria on a clean catch specimen of urine. She has
significant swelling of her face and extremities. She denies having contractions. Her cervix is
closed and unef- faced. The baby is breech by bedside ultrasonography. She says the baby’s
movements have decreased in the past 24 hours. Which of the following is the best next step in
the management of this patient?
a. Send her to labor and delivery for a BPP.
b. Send her home with instructions to stay on strict bed rest until her swelling and blood
pressure improve.
c. Admit her to the hospital for enforced bed rest and diuretic therapy to improve her
swelling and blood pressure.
d. Admit her to the hospital for induction of labor.
e. Admit her to the hospital for cesarean delivery.

7
Obstetrical Complications of Pregnancy

30/ . A 29-year-old G3P2 presents to the emergency center with com- plaints of abdominal
discomfort for 2 weeks. Her vital signs are: blood pressure 120/70 mm Hg, pulse 90 beats per
minute, temperature 36.94°C, respiratory rate 18 breaths per minute. A pregnancy test is
positive and an ultrasound of the abdomen and pelvis reveals a viable 16-week gestation located
behind a normal-appearing 10 × 6 × 5.5 cm uterus. Both ovaries appear normal. No free fluid is
noted. Which of the following is the most likely cause of these findings?
a. Ectopic ovarian tissue
b. Fistula between the peritoneum and uterine cavity
c. Primary peritoneal implantation of the fertilized ovum
d. Tubal abortion
e. Uterine rupture of prior cesarean section scar
31/ . A 32-year-old G2P1 at 28 weeks gestation presents to labor and delivery with the complaint of
vaginal bleeding. Her vital signs are: blood pressure 115/67 mm Hg, pulse 87 beats per minute,
temperature 37.0°C, respiratory rate 18 breaths per minute. She denies any contraction and
states that the baby is moving normally. On ultrasound the placenta is anteriorly located and
completely covers the internal cervical os. Which of the following would most increase her risk
for hysterectomy?
a. Desire for sterilization
b. Development of disseminated intravascular coagulopathy (DIC)
c. Placenta accreta
d. Prior vaginal delivery
e. Smoking
32/ . A patient at 17 weeks gestation is diagnosed as having an intrauter- ine fetal demise. She returns
to your office 5 weeks later and her vital signs are: blood pressure 110/72 mm Hg, pulse 93
beats per minute, tempera- ture 36.38°C, respiratory rate 16 breaths per minute. She has not had
a miscarriage, although she has had some occasional spotting. Her cervix is closed on
examination. This patient is at increased risk for which of the following?
a. Septic abortion
b. Recurrent abortion
c. Consumptive coagulopathy with hypofibrinogenemia
d. Future infertility
e. Ectopic pregnancies
33/ . A 24-year-old presents at 30 weeks with a fundal height of 50 cm. Which of the following
statements concerning polyhydramnios is true?
a. Acute polyhydramnios rarely leads to labor prior to 28 weeks.
b. The incidence of associated malformations is approximately 3%.
c. Maternal edema, especially of the lower extremities and vulva, is rare.
d. Esophageal atresia is accompanied by polyhydramnios in nearly 10% of cases.
e. Complications include placental abruption, uterine dysfunction, and postpartum
hemorrhage.
34/ . A 20-year-old G1 at 32 weeks presents for her routine obstetric (OB) visit. She has no medical
problems. She is noted to have a blood pressure of 150/96 mm Hg, and her urine dip shows 1+
protein. She complains of a constant headache and vision changes that are not relieved with rest
or a pain reliever. The patient is sent to the hospital for further management. At the hospital, her
blood pressure is 158/98 mm Hg and she is noted to have tonic-clonic seizure. Which of the

8
following is indicated in the manage- ment of this patient?
a. Low-dose aspirin
b. Dilantin (phenytoin)
c. Antihypertensive therapy
d. Magnesium sulfate
e. Cesarean delivery

35/ . A 32-year-old G5P1 presents for her first prenatal visit. A complete obstetrical, gynecological,
and medical history and physical examination is done. Which of the following would be an
indication for elective cerclage placement?
a. Three spontaneous first-trimester abortions
b. Twin pregnancy
c. Three second-trimester pregnancy losses without evidence of labor or abruption
d. History of loop electrosurgical excision procedure for cervical dysplasia
e. Cervical length of 35 mm by ultrasound at 18 weeks
36/ . Uterine bleeding at 12 weeks gestation accompanied by cervical dilation without passage of
tissue.
Match above description with the correct type of abortion.
a. Complete abortion
b. Incomplete abortion
c. Threatened abortion
d. Missed abortion
e. Inevitable abortion
37/ . Passage of some but not all placental tissue through the cervix at 9 weeks gestation.
Match above description with the correct type of abortion.
a. Complete abortion
b. Incomplete abortion
c. Threatened abortion
d. Missed abortion
e. Inevitable abortion
38/ . Fetal death at 15 weeks gestation without expulsion of any fetal or maternal tissue for at least 8
weeks.
Match above description with the correct type of abortion.
a. Complete abortion
b. Incomplete abortion
c. Threatened abortion
d. Missed abortion
e. Inevitable abortion
39/ . Uterine bleeding at 7 weeks gestation without any cervical dilation.
Match above description with the correct type of abortion.
a. Complete abortion
b. Incomplete abortion
c. Threatened abortion
d. Missed abortion
e. Inevitable abortion
40/ . Expulsion of all fetal and placental tissue from the uterine cavity at 10 weeks gestation.
Match above description with the correct type of abortion.
a. Complete abortion

9
b. Incomplete abortion
c. Threatened abortion
d. Missed abortion
e. Inevitable abortion
41/ . A 20-year-old G1P0 presents to your clinic for follow-up for a suc- tion dilation and curettage
for an incomplete abortion. She is asymptomatic without any vaginal bleeding, fever, or chills.
Her examination is normal. The pathology report reveals trophoblastic proliferation and
hydropic degenera- tion with the absence of vasculature; no fetal tissue is identified. A chest x-
ray is negative for any evidence of metastatic disease. Which of the following is the best next
step in her management?
a. Weekly human chorionic gonadotropin (hCG) titers
b. Hysterectomy
c. Single-agent chemotherapy
d. Combination chemotherapy
e. Radiation therapy
42/ . A 22-year-old G1P0 presents to your clinic for follow-up of evacuation of a complete
hydatidiform mole. She is asymptomatic and her examination is normal. Which of the following
would be an indication to start single-agent chemotherapy?
a. A rise in hCG titers
b. A plateau of hCG titers for 1 week
c. Return of hCG titer to normal at 6 weeks after evacuation
d. Appearance of liver metastasis
e. Appearance of brain metastasis
43/ . A 32-year-old female presents to the emergency department with abdominal pain and vaginal
bleeding. Her last menstrual period was 8 weeks ago and her pregnancy test is positive. On
examination she is tachycardic and hypotensive and her abdominal examination findings reveal
peritoneal signs, a bedside abdominal ultrasound shows free fluid within the abdominal cavity.
The decision is made to take the patient to the operating room for emergency exploratory
laparotomy. Which of the following is the most likely diagnosis?
a. Ruptured ectopic pregnancy
b. Hydatidiform mole
c. Incomplete abortion d. Missed abortion
e. Torsed ovarian corpus luteal cyst

44/ . A 27-year-old has just had an ectopic pregnancy. Which of the following events would be most
likely to predispose to ectopic pregnancy?
a. Previous cervical conization
b. Pelvic inflammatory disease (PID)
c. Use of a contraceptive uterine device (IUD)
d. Induction of ovulation
e. Exposure in utero to diethylstilbestrol (DES)

45/ . A 34-year-old G2P1 at 31 weeks gestation presents to labor and delivery with complaints of
vaginal bleeding earlier in the day that resolved on its own. She denies any leakage of fluid or
uterine contractions. She reports good fetal movement. In her last pregnancy, she had a low
trans- verse cesarean delivery for breech presentation at term. She denies any medical problems.
Her vital signs are normal and electronic external monitoring reveals a reactive fetal heart rate
tracing and no uterine contractions. Which of the following is the most appropriate next step in

10
the management of this patient?
a. Send her home, since the bleeding has completely resolved and she is experiencing good
fetal movements
b. Perform a sterile digital examination
c. Perform an amniocentesis to rule out infection
d. Perform a sterile speculum examination
e. Perform an ultrasound examination
46/ . A 34-year-old G2P1 at 31 weeks gestation with a known placenta previa presents to the hospital
with vaginal bleeding. On assessment, she has normal vital signs and the fetal heart rate tracing
is 140 beats per minute with accelerations and no decelerations. No uterine contractions are
demonstrated on external tocometer. Heavy vaginal bleeding is noted. Which of the following is
the best next step in the management of this patient?
a. Administer intramuscular terbutaline
b. Administer methylergonovine
c. Admit and stabilize the patient
d. Perform cesarean delivery
e. Induce labor

47/ . A 40-year-old G2P1001 presents to your office for a routine OB visit at 30 weeks gestational
age. Her first pregnancy was delivered 10 years ago and was uncomplicated. She had a normal
vaginal delivery at 40 weeks and the baby weighed 7 lb. During this present pregnancy, she has
not had any complications, and she reports no significant medical history. She is a non- smoker
and has gained about 25 lb to date. Despite being of advanced maternal age, she declined any
screening or diagnostic testing for Down syndrome. Her blood pressure range has been 100 to
120/60 to 70. During her examination, you note that her fundal height measures only 25 cm.
Which of the following is a likely explanation for this patient’s decreased fundal height?
a. Multiple gestation
b. Hydramnios
c. Fetal growth restriction
d. The presence of fibroid tumors in the uterus
e. Large ovarian mass

48/ . A 26-year-old G1 at 37 weeks presents to the hospital in active labor. She has no medical
problems and has a normal prenatal course except for fetal growth restriction. She undergoes an
uncomplicated vaginal delivery of a female infant weighing 1950 g. The infant is at risk for
which of the following complications?
a. Hyperglycemia
b. Fever
c. Hypertension
d. Anemia
e. Hypoxia

49/ . A 39-year-old G1P0 at 39 weeks gestational age is sent to labor and delivery from her
obstetrician’s office because of a blood pressure reading of 150/100 mm Hg obtained during a
routine OB visit. Her baseline blood pressures during the pregnancy were 100 to 120/60 to 70.
On arrival to labor and delivery, the patient denies any headache, visual changes, nausea,
vomiting, or abdominal pain. The heart rate strip is reactive and the toco- dynamometer
indicates irregular uterine contractions. The patient’s cervix is 3 cm dilated. Her repeat blood

11
pressure is 160/90 mm Hg. Hematocrit is
34.0, platelets are 160,000, SGOT is 22, SGPT is 15, and urinalysis is neg- ative for protein.
Which of the following is the most likely diagnosis?
a. Preeclampsia
b. Chronic hypertension
c. Chronic hypertension with superimposed preeclampsia
d. Eclampsia
e. Gestational hypertension
50/ . A 20-year-old G1 at 36 weeks is being monitored for preeclampsia; she rings the bell for the
nurse because she is developing a headache and feels funny. As you and the nurse enter the
room, you witness the patient undergoing a tonic-clonic seizure. You secure the patient’s
airway, and within a few minutes the seizure is over. The patient’s blood pressure monitor
indicates a pressure of 160/110 mm Hg. Which of the following medications is recommended
for the prevention of a recurrent eclamptic seizure?
a. Hydralazine
b. Magnesium sulfate
c. Labetalol
d. Pitocin
e. Nifedipine

Medical and Surgical Complications of Pregnancy

51/ . A 22-year-old G1 at 14 weeks gestation presents to your office with a history of recent exposure
to her 3-year-old nephew who had a rubella viral infection. In which time period does maternal
infection with rubella virus carry the greatest risk for congenital rubella syndrome in the fetus?
a. Preconception
b. First trimester
c. Second trimester
d. Third trimester
e. Postpartum

52/ . A 30-year-old class D diabetic is concerned about pregnancy. She can be assured that which of
the following risks is the same for her as for the general population?
a. Preeclampsia and eclampsia
b. Infection
c. Fetal cystic fibrosis
d. Postpartum hemorrhage after vaginal delivery
e. Hydramnios

53/ . A 23-year-old G1P0 reports to your office for a routine OB visit at 28 weeks gestational age.
Labs drawn at her prenatal visit 2 weeks ago reveal a 1-hour glucose test of 128, hemoglobin of
10.8, and a platelet count of 80,000. All her other labs were within normal limits. During the
present visit, the patient has a blood pressure of 120/70 mm Hg. Her urine dip is negative for
protein, glucose, and blood. The patient denies any com- plaints. The only medication she is
currently taking is a prenatal vitamin. She does report a history of epistaxis on occasion, but no
other bleeding. Which of the following medical treatments should you recommend to treat the
thrombocytopenia?

12
a. No treatment is necessary
b. Stop prenatal vitamins
c. Oral corticosteroid therapy
d. Intravenous immune globulin
e. Splenectomy

Normal and Abnormal Labor and Delivery

54/ . A 20-year-old G1 at 38 weeks gestation presents with regular painful contractions every 3 to 4
minutes lasting 60 seconds. On pelvic examina- tion, she is 3 cm dilated and 90% effaced; an
amniotomy is performed and clear fluid is noted. The patient receives epidural analgesia for
pain man- agement. The fetal heart rate tracing is reactive. One hour later on repeat
examination, her cervix is 5 cm dilated and 100% effaced. Which of the following is the best
next step in her management?
a. Begin pushing
b. Initiate Pitocin augmentation for protracted labor
c. No intervention; labor is progressing normally
d. Perform cesarean delivery for inadequate cervical effacement
e. Stop epidural infusion to enhance contractions and cervical change
55/ . A 30-year-old G2P0 at 39 weeks is admitted in active labor with spontaneous rupture of
membranes occurring 2 hours prior to admission. The patient noted clear fluid at the time. On
examination, her cervix is 4 cm dilated and completely effaced. The fetal head is at 0 station
and the fetal heart rate tracing is reactive. Two hours later on repeat examination her cervix is 5
cm dilated and the fetal head is at +1 station. Early decelerations are noted on the fetal heart rate
tracing. Which of the following is the best next step in her labor management?
a. Administer terbutaline
b. Initiate amnioinfusion
c. Initiate Pitocin augmentation
d. Perform cesarean delivery for arrest of descent
e. Perform cesarean delivery of early decelerations

56/ . A 27-year-old G2P1 at 38 weeks gestation was admitted in active labor at 4 cm dilated;
spontaneous rupture of membranes occurred prior to admission. She has had one prior
uncomplicated vaginal delivery and denies any medical problems or past surgery. She reports
an allergy to sulfa drugs. Currently, her vital signs are normal and the fetal heart rate tracing is
reactive. Her prenatal record indicates that her Group B streptococcus (GBS) culture at 36
weeks was positive. What is the recommended antibiotic for prophylaxis during labor?
a. Cefazolin
b. Clindamycin
c. Erythromycin
d. Penicillin
e. Vancomycin

57/ . A 23-year-old G1 at 38 weeks gestation presents in active labor at


6 cm dilated with ruptured membranes. On cervical examination the fetal nose, eyes, and lips
can be palpated. The fetal heart rate tracing is 140 beats per minute with accelerations and no

13
decelerations. The patient’s pelvis is adequate. Which of the following is the most
appropriate management for this patient?
a. Perform immediate cesarean section without labor.
b. Allow spontaneous labor with vaginal delivery.
c. Perform forceps rotation in the second stage of labor to convert mentum posterior to
mentum anterior and to allow vaginal delivery.
d. Allow patient to labor spontaneously until complete cervical dilation is achieved and
then perform an internal podalic version with breech extraction.
e. Attempt manual conversion of the face to vertex in the second stage of labor.
58/ . A 32-year-old G3P2 at 39 weeks gestation presented to the hospital with ruptured membranes
and 4 cm dilated. She has a history of two prior vaginal deliveries, with her largest child
weighing 3800 g at birth. Over the next 2 hours she progresses to 7 cm dilated. Two hours later,
she remains 7 cm dilated. The estimated fetal weight by ultrasound is 3200 g. Which of the
following labor abnormalities best describes this patient?
a. Prolonged latent phase
b. Protracted active-phase dilation
c. Hypertonic dysfunction
d. Secondary arrest of dilation
e. Primary dysfunction
59/ . You are following a 38-year-old G2P1 at 39 weeks in labor. She has had one prior vaginal
delivery of a 3800-g infant. One week ago, the esti- mated fetal weight was 3200 g by
ultrasound. Over the past 3 hours her cervical examination remains unchanged at 6 cm. Fetal
heart rate tracing is reactive. An intrauterine pressure catheter (IUPC) reveals two contractions
in 10 minutes with amplitude of 40 mm Hg each. Which of the following is the best
management for this patient?
a. Ambulation
b. Sedation
c. Administration of oxytocin
d. Cesarean section
e. Expectant
60/ . A primipara is in labor and an episiotomy is about to be cut. Compared with a midline
episiotomy, which of the following is an advantage of mediolateral episiotomy?
a. Ease of repair
b. Fewer breakdowns
c. Less blood loss
d. Less dyspareunia
e. Less extension of the incision
61/ . A 27-year-old woman (G3P2) comes to the delivery floor at 37 weeks gestation. She has had no
prenatal care. She complains that, on bending down to pick up her 2-year-old child, she
experienced sudden, severe back pain that now has persisted for 2 hours. Approximately 30
minutes ago she noted bright red blood coming from her vagina. By the time she arrives at the
delivery floor, she is contracting strongly every 3 minutes; the uterus is quite firm even between
contractions. By abdominal palpation, the fetus is vertex with the head deeply engaged. Fetal
heart rate is 130 beats per minutes. The fundus is 38 cm above the symphysis. Blood for clotting
is drawn, and a clot forms in 4 minutes. Clotting studies are sent to the laboratory. Which of the
following actions can most likely wait until the patient is stabilized?
a. Stabilizing maternal circulation
b. Attaching a fetal electronic monitor

14
c. Inserting an intrauterine pressure catheter
d. Administering oxytocin
e. Preparing for cesarean section
62/ . A 24-year-old primigravid woman, at term, has been in labor for 16 hours and has been dilated
to 9 cm for 3 hours. The fetal vertex is in the right occiput posterior position, at +1 station, and
molded. There have been mild late decelerations for the past 30 minutes. Twenty minutes ago,
the fetal scalp pH was 7.27; it is now 7.20.
For above clinical description, select the most appropriate procedure.
a. External version
b. Internal version
c. Midforceps rotation
d. Low transverse cesarean section
e. Classic cesarean section
63/ . You have just delivered an infant weighing 2.5 kg (5.5 lb) at 39 weeks gestation. Because the
uterus still feels large, you do a vaginal examination. A second set of membranes is bulging
through a fully dilated cervix, and you feel a small part presenting in the sac. A fetal heart is
auscultated at 60 beats per minute.
For above clinical description, select the most appropriate procedure.
a. External version
b. Internal version
c. Midforceps rotation
d. Low transverse cesarean section
e. Classic cesarean section
64/ . A 24-year-old woman (G3P2) is at 40 weeks gestation. The fetus is in the transverse lie
presentation.
For above clinical description, select the most appropriate procedure.
a. External version
b. Internal version
c. Midforceps rotation
d. Low transverse cesarean section
e. Classic cesarean section

65/ . A nulliparous woman is in active labor (cervical dilation 5 cm with complete effacement, vertex
at 0 station); the labor curve shows pro- tracted progression without descent following the
administration of an epidural block. An IUPC shows contractions every 4 to 5 minutes, peaking
at 40 mm Hg.
Select the most appropriate treatment for above clinical situation.
a. Epidural block
b. Meperidine (Demerol) 100 mg intramuscularly
c. Oxytocin intravenously
d. Midforceps delivery
e. Cesarean section
66/ . A nulliparous woman has had arrest of descent for the past 2 hours and arrest of dilation for the
past 3 hours. The cervix is dilated to 7 cm and the vertex is at +1 station. Monitoring shows a
normal pattern and adequate contractions. Fetal weight is estimated at 7.5 lb.
Select the most appropriate treatment for above clinical situation.
a. Epidural block
b. Meperidine (Demerol) 100 mg intramuscularly

15
c. Oxytocin intravenously
d. Midforceps delivery
e. Cesarean section
67/ . Appears to lengthen the second stage of labor.
Match above description with the most appropriate type of obstetric anesthesia.
a. Paracervical block
b. Pudendal block
c. Spinal block
d. Epidural block

68/ . A 23-year-old G1 at 40 weeks gestation presents to the hospital with the complaint of
contractions. She states they are occurring every 4 to 8 minutes and each lasts approximately 1
minute. She reports good fetal movement and denies any leakage of fluid or vaginal bleeding.
The nurse places an external tocometer and fetal monitor and reports that the patient is having
contractions every 2 to 10 minutes. The nurse states that the con- tractions are mild to palpation.
On examination the cervix is 2 cm dilated, 50% effaced, and the vertex is at −1 station. The
patient had the same cervical examination in your office last week. The fetal heart rate tracing
isn140 beats per minute with accelerations and no decelerations. Which of the following stages
of labor is this patient in?
a. Active labor
b. Latent labor
c. False labor
d. Stage 1 of labor
e. Stage 2 of labor

69/ . A 28-year-old G1 at 38 weeks had a normal progression of her labor. She has an epidural and
has been pushing for 2 hours. The fetal head is direct occiput anterior at +3 station. The fetal
heart rate tracing is 150 beats per minute with variable decelerations. With the patient’s last
push the fetal heart rate had a prolonged deceleration to the 80s for 3 minutes. You recommend
forceps to assist the delivery owing to the nonreassuring fetal heart rate tracing. Compared to
the use of the vacuum extractor, forceps are associated with an increased risk of which of the
following neonatal complications?
a. Cephalohematoma
b. Retinal hemorrhage
c. Jaundice
d. Intracranial hemorrhage
e. Corneal abrasions
70/ . You performed a forceps-assisted vaginal delivery on a 20-year-old G1 at 40 weeks for
maternal exhaustion. The patient had pushed for 3 hours with an epidural for pain management.
A second-degree episiotomy was cut to facilitate delivery. Eight hours after delivery, you are
called to see the patient because she is unable to void and complains of severe pain. On
examination you note a large fluctuant purple mass inside the vagina. What is the best
management for this patient?
a. Apply an ice pack to the perineum
b. Embolize the internal iliac artery
c. Incision and evacuation of the hematoma
d. Perform dilation and curettage to remove retained placenta
e. Place a vaginal pack for 24 hours

16
71/ . A 20-year-old G1 at 41 weeks has been pushing for 21/2 hours. The fetal head is at the introitus
and beginning to crown. It is necessary to cut an episiotomy. The tear extends through the
sphincter of the rectum, but the rectal mucosa is intact. How should you classify this type of
episiotomy?
a. First-degree
b. Second-degree
c. Third-degree
d. Fourth-degree
e. Mediolateral episiotomy
72/ . A 25-year-old G1P0 patient at 41 weeks presents to labor and delivery complaining of gross
rupture of membranes and painful uterine contractions every 2 to 3 minutes. On digital
examination, her cervix is 3 cm dilated and completely effaced with fetal feet palpable through
the cervix. The estimated weight of the fetus is about 6 lb, and the fetal heart rate tracing is
reactive. Which of the following is the best method to achieve delivery?
a. Deliver the fetus vaginally by breech extraction
b. Deliver the baby vaginally after external cephalic version
c. Perform an emergent cesarean section
d. Perform an internal podalic version
e. Perform a forceps-assisted vaginal delivery
73/ . A 25-year-old G1 at 37 weeks presents to labor and delivery with gross rupture of membranes.
The fluid is noted to be clear and the patient is noted to have regular painful contractions every
2 to 3 minutes lasting for 60 seconds each. The fetal heart rate tracing is reactive. On cervical
examination she is noted to be 4 cm dilated, 90% effaced with the presenting part a −3 station.
The presenting part is soft and felt to be the fetal buttock. A quick bedside ultrasound reveals a
breech presentation with both hips flexed and knees extended. What type of breech
presentation is described?
a. Frank
b. Incomplete, single footling
c. Complete
d. Double footling

The Puerperium, Lactation, and Immediate Care of the Newborn

74/ . On postoperative day 3 after an uncomplicated repeat cesarean delivery, the patient develops a
fever of 38.2°C (100.8°F). She has no com- plaints except for some fullness in her breasts. On
examination she appears in no distress; lung and cardiac examinations are normal. Her breast
exam- ination reveals full, firm breasts bilaterally slightly tender with no erythema or masses.
She is not breast-feeding. The abdomen is soft with firm, non- tender fundus at the umbilicus.
The lochia appears normal and is non- odorous. Urinalysis and white blood cell count are
normal. Which of the following is a characteristic of the cause of her puerperal fever?
a. Appears in less than 5% of postpartum women
b. Appears 3 to 4 days after the development of lacteal secretion
c. Is almost always painless
d. Fever rarely exceeds 37.8°C (99.8°F)
e. Is less severe and less common if lactation is suppressed
75/ . A 38-year-old G3P3 begins to breast-feed her 5-day-old infant. The baby latches on
appropriately and begins to suckle. In the mother, which of the following is a response to

17
suckling?
a. Decrease of oxytocin
b. Increase of prolactin-inhibiting factor
c. Increase of hypothalamic dopamine
d. Increase of hypothalamic prolactin
e. Increase of luteinizing hormone—releasing factor

76/ . A 24-year-old G1P1 presents for her routine postpartum visit 6 weeks after an uncomplicated
vaginal delivery. She states that she is having prob- lems sleeping and is feeling depressed over
the past 2 to 3 weeks. She reveals that she cries on most days and feels anxious about taking
care of her newborn son. She denies any weight loss or gain, but states she doesn’t feel like
eating or doing any of her normal activities. She denies suicidal or homicidal ideation. Which of
the following is true regarding this patient’s condition?
a. A history of depression is not a risk factor for developing postpartum depression.
b. Prenatal preventive intervention for patients at high risk for postpartum depression is
best managed alone by a mental health professional.
c. Young, multiparous patients are at highest risk.
d. Postpartum depression is a self-limiting process that lasts for a maximum of 3 months.
e. About 8% to 15% of women develop postpartum depression.

77/ . A 21-year-old G1 at 40 weeks, who underwent induction of labor for severe preeclampsia,
delivered a 3900-g male infant via vaginal delivery after pushing for 21/2 hours. A second-
degree midline laceration and side- wall laceration were repaired in the usual fashion under
local analgesia. The estimated blood loss was 450 cc. Magnesium sulfate is continued post-
partum for the seizure prophylaxis. Six hours after the delivery, the patient has difficulty
voiding. Which is the most likely cause of her problem?
a. Preeclampsia
b. Infusion of magnesium sulfate
c. Vulvar hematoma
d. Ureteral injury
e. Use of local analgesia for repair

78/ . A 32-year-old G2P2 develops fever and uterine tenderness 2 days after cesarean delivery for
nonreassuring fetal heart tones. She is placed on intravenous penicillin and gentamicin for her
infection. After 48 hours of antibiotics she remains febrile, and on examination she continues to
have uterine tenderness. Which of the following bacteria is resistant to these antibiotics and is
most likely to be responsible for this woman’s infection?
a. Proteus mirabilis
b. Bacteroides fragilis
c. Escherichia coli
d. α-Streptococci
e. Anaerobic streptococci

79/ . A 21-year-old G2P2 calls her physician 7 days postpartum because she is concerned that she is
still bleeding from the vagina. She describes the bleeding as light pink to bright red and less
heavy than the first few days postdelivery. She denies fever or any cramping pain. On
examination she is afebrile and has an appropriately sized, nontender uterus. The vagina con-
tains about 10 cc of old, dark blood. The cervix is closed. Which of the fol- lowing is the most

18
appropriate treatment?
a. Antibiotics for endometritis
b. High-dose oral estrogen for placental subinvolution
c. Oxytocin for uterine atony
d. Suction dilation and curettage for retained placenta
e. Reassurance
80/ . A 28-year-old G2P2 presents to the hospital 2 weeks after vaginal delivery with the complaint
of heavy vaginal bleeding that soaks a sanitary napkin every hour. Her pulse is 89 beats per
minute, blood pressure 120/76 mm Hg, and temperature 37.1°C (98.9°F). Her abdomen is non-
tender and her fundus is located above the symphysis pubis. On pelvic examination, her vagina
contained small blood clots and no active bleeding is noted from the cervix. Her uterus is about
12 to 14 weeks size and non- tender. Her cervix is closed. An ultrasound reveals an 8-mm
endometrial stripe. Her hemoglobin is 10.9, unchanged from the one at her vaginal delivery. β-
hCG is negative. Which of the following potential treatments would be contraindicated?
a. Methylergonovine maleate (Methergine)
b. Oxytocin injection (Pitocin)
c. Ergonovine maleate (Ergotrate)
d. Prostaglandins
e. Dilation and curettage
81/ . A 22-year-old G1P0 has just undergone a spontaneous vaginal delivery. As the placenta is being
delivered, a red fleshy mass is noted to be protruding out from behind the placenta. Which of
the following is the best next step in management of this patient?
a. Begin intravenous oxytocin infusion
b. Call for immediate assistance from other medical personnel
c. Continue to remove the placenta manually
d. Have the anesthesiologist administer magnesium sulfate
e. Shove the placenta back into the uterus
82/ . Following a vaginal delivery, a woman develops a fever, lower abdom- inal pain, and uterine
tenderness. She is alert, and her blood pressure and urine output are good. Large gram-positive
rods suggestive of clostridia are seen in a smear of the cervix. Which of the following is most
closely tied to a decision to proceed with hysterectomy?
a. Close observation for renal failure or hemolysis
b. Immediate radiographic examination for hydrosalpinx
c. High-dose antibiotic therapy
d. Fever of 103°F
e. Gas gangrene
83/ . Three days ago you delivered a 40-year-old G1P1 by cesarean section following arrest of
descent after 2 hours of pushing. Labor was also signif- icant for prolonged rupture of
membranes. The patient had an epidural, which was removed the day following delivery. The
nurse pages you to come to see the patient on the postpartum floor because she has a fever of
38.8°C (102°F) and is experiencing shaking chills. Her blood pressure is 120/70 mm Hg and her
pulse is 120 beats per minute. She has been eating a regular diet without difficulty and had a
normal bowel movement this morning. She is attempting to breast-feed, but says her milk has
not come in yet. On physical examination, her breasts are mildly engorged and ten- der
bilaterally. Her lungs are clear. Her abdomen is tender over the fundus, but no rebound is
present. Her incision has some serous drainage at the right apex, but no erythema is noted. Her
pelvic examination reveals uterine tenderness but no masses. Which of the following is the most
likely diagnosis?

19
a. Pelvic abscess
b. Septic pelvic thrombophlebitis
c. Wound infection
d. Endometritis
e. Atelectasis

84/ . You are doing postpartum rounds on a 23-year-old G1P1 who is postpartum day 2 after an
uncomplicated vaginal delivery. As you walk in the room, you note that she is crying. She states
she can’t seem to help it. She denies feeling sad or anxious. She has not been sleeping well
because of getting up every 2 to 3 hours to breast-feed her new baby. Her past medical history is
unremarkable. Which of the following is the most appropriate treatment recommendation?
a. Time and reassurance, because this condition is self-limited
b. Referral to psychiatry for counseling and antidepressant therapy
c. Referral to psychiatry for admission to a psychiatry ward and therapy with Haldol
d. A sleep aid
e. Referral to a psychiatrist who can administer electroconvulsive therapy
85/ . A 20-year-old G1P1 is postpartum day 2 after an uncomplicated vaginal delivery of a 6-lb 10-
oz baby boy. She is trying to decide whether to have you perform a circumcision on her
newborn. The boy is in the well- baby nursery and is doing very well. In counseling this patient,
you tell her which of the following recommendations from the American Pediatric
Association?
a. Circumcisions should be performed routinely because they decrease the incidence of
male urinary tract infections.
b. Circumcisions should be performed routinely because they decrease the incidence of
penile cancer.
c. Circumcisions should be performed routinely because they decrease the incidence of
sexually transmitted diseases.
d. Circumcisions should not be performed routinely because of insufficient data regarding
risks and benefits.
e. Circumcisions should not be performed routinely because it is a risky procedure and
complications such as bleeding and infection are common.
86/ . You are counseling a new mother and father on the risks and benefits of circumcision for their
1-day-old son. The parents ask if you will use analgesia during the circumcision. What do you
tell them regarding the recommendations for administering pain medicine for circumcisions?
a. Analgesia is not recommended because there is no evidence that newborns undergoing
circumcision experience pain.
b. Analgesia is not recommended because it is unsafe in newborns.
c. Analgesia in the form of oral Tylenol is the pain medicine of choice recom- mended for
circumcisions.
d. Analgesia in the form of a penile block is recommended.
e. The administration of sugar orally during the procedure will keep the neonate
preoccupied and happy.

87/ . You are asked to assist in the well-born nursery with neonatal care. Which of the following is a
part of routine care in a healthy infant?
a. Administration of ceftriaxone cream to the eyes for prophylaxis for gonorrhea and
chlamydia
b. Administration of vitamin A to prevent bleeding problems

20
c. Administration of hepatitis B vaccination for routine immunization
d. Cool-water bath to remove vernix
e. Placement of a computer chip in left buttock for identification purposes
88/ . You are making rounds on a 29-year-old G1P1 who underwent an uncomplicated vaginal
delivery at term on the previous day. The patient is still very confused about whether she wants
to breast-feed. She is a very busy lawyer and is planning on going back to work in 4 weeks, and
she does not think that she has the time and dedication that breast-feeding requires. She asks
you what you think is best for her to do. Which of the following is an accurate statement
regarding breast-feeding?
a. Breast-feeding decreases the time to return of normal menstrual cycles.
b. Breast-feeding is associated with a decreased incidence of sudden infant death syndrome.
c. Breast-feeding is a poor source of nutrients for required infant growth.
d. Breast-feeding is associated with an increased incidence of childhood obesity.
e. Breast-feeding is associated with a decreased incidence of childhood attention deficit
disorder.
89/ . A 22-year-old G1P1 who is postpartum day 2 and is bottle-feeding complains that her breasts
are very engorged and tender. She wants you to give her something to make the engorgement go
away. Which of the following is recommended to relieve her symptoms?
a. Breast binder
b. Bromocriptine
c. Estrogen-containing contraceptive pills
d. Pump her breasts
e. Use oral antibiotics
90/ . A 36-year-old G1P1 comes to see you for a routine postpartum exam- ination 6 weeks after an
uncomplicated vaginal delivery. She is currently nursing her baby without any major problems
and wants to continue to do so for at least 9 months. She is ready to resume sexual activity and
wants to know what her options are for birth control. She does not have any medical problems.
She is a nonsmoker and is not taking any medications except for her prenatal vitamins. Which
of the following methods may decrease her milk supply?
a. Intrauterine device
b. Progestin only pill
c. Depo-Provera
d. Combination oral contraceptives
e. Foam and condoms
91/ . A 30-year-old G3P3, who is 8 weeks postpartum and regularly breast-feeding calls you and is
very concerned because she is having pain with intercourse secondary to vaginal dryness.
Which of the following should you recommend to help her with this problem?
a. Instruct her to stop breast-feeding
b. Apply hydrocortisone cream to the perineum
c. Apply testosterone cream to the vulva and vagina
d. Apply estrogen cream to the vagina and vulva
e. Apply petroleum jelly to the perineum
92/ . A 25-year-old G1P1 comes to see you 6 weeks after an uncompli- cated vaginal delivery for a
routine postpartum examination. She denies any problems and has been breast-feeding her
newborn without any diffi- culties since leaving the hospital. During the bimanual examination,
you note that her uterus is irregular, firm, nontender, and about a 15-week size. Which of the
following is the most likely etiology for this enlarged uterus?
a. Subinvolution of the uterus

21
b. The uterus is appropriate size for 6 weeks postpartum
c. Fibroid uterus
d. Adenomyosis
e. Endometritis
93/ . A 39-year-old G3P3 comes to see you on day 5 after a second repeat cesarean delivery. She is
concerned because her incision has become very red and tender and pus started draining from a
small opening in the inci- sion this morning. She has been experiencing general malaise and
reports a fever of 38.8°C (102°F). Physical examination indicates that the Pfan- nenstiel
incision is indeed erythematous and is open about 1 cm at the left corner, and is draining a small
amount of purulent liquid. There is tender- ness along the wound edges. Which of the following
is the best next step in the management of this patient?
a. Apply Steri-Strips to close the wound
b. Administer antifungal medication
c. Probe the fascia
d. Take the patient to the OR for debridement and closure of the skin
e. Reapproximate the wound edge under local analgesia
94/ . A 30-year-old G3P3 is postoperative day 4 after a repeat cesarean delivery. During the surgery
she received 2 units of packed red blood cells for a hemorrhage related to uterine atony. She is
to be discharged home today. She complains of some yellowish drainage from her incision and
redness that just started earlier in the day. She states that she feels feverish. She is breast-
feeding. Her past medical history is significant for type 2 dia- betes mellitus and chronic
hypertension. She weighs 110 kg. Her vital signs are temperature 37.8°C (100.1°F), pulse 69
beats per minute, respi- ratory rate 18 breaths per minute, and blood pressure is 143/92 mm Hg.
Breast, lung, and cardiac examinations are normal. Her midline vertical skin incision is
erythematous and has a foul-smelling purulent discharge from the lower segment of the wound.
It is tender to touch. The uterine fundus is not tender. Which of the following is not a risk factor
for her condition?
a. Diabetes
b. Corticosteroid therapy
c. Preoperative antibiotic administration
d. Anemia
e. Obesity

22
Gynecology- Preventive Care and Health Maintenance

95/ . You are following up on the results of routine testing of a 68-year-old G4P3 for her well-
woman examination. Her physical examination was nor- mal for a postmenopausal woman. Her
Pap smear revealed parabasal cells, her mammogram was normal, lipid profile was normal, and
the urinalysis shows hematuria. Which of the following is the most appropriate next step in the
management of this patient?
a. Colposcopy
b. Endometrial biopsy
c. Renal sonogram
d. Urine culture
e. No further treatment/evaluation is necessary if the patient is asymptomatic.
96/ . A 74-year-old woman presents to your office for well-woman exam- ination. Her last Pap smear
and mammogram were 3 years ago. She has hypertension, high cholesterol, and osteoarthritis.
She stopped smoking 15 years ago, and denies alcohol use. Based on this patient’s history
which of the following medical conditions should be this patient’s biggest concern?
a. Alzheimer disease
b. Breast cancer
c. Cerebrovascular disease
d. Heart disease
e. Lung cancer
97/ . A 17-year-old G1P1 presents to your office for her yearly well- woman examination. She
had an uncomplicated vaginal delivery last year. She has been sexually active for the past 4
years and has had six different sexual partners. Her menses occurs every 28 days and lasts for 4
days. She denies any intermenstrual spotting, postcoital bleeding, or vaginal dis- charge. She
denies tobacco, alcohol, or illicit drug use. Which of the fol- lowing are appropriate screening
tests for this patient?
a. Pap test
b. Pap test and gonorrhea and chlamydia cervical cultures
c. Pap test and herpes simplex cultures
d. Pap test and hemoglobin level assessment
e. Pap test and hepatitis C antibody

98/ . A 26-year-old woman presents to your office for her well-woman examination. She denies any
medical problems or prior surgeries. She states that her cycles are monthly. She is sexually
active and uses oral contracep- tive pills for birth control. Her physical examination is normal.
As part of preventive health maintenance, you recommend breast self-examination and instruct
the patient how to do it. Which of the following is the best fre- quency and time to perform
breast self-examinations?
a. Monthly, in the week prior to the start of the menses
b. Monthly, in the week after cessation of menses
c. Monthly, during the menses
d. Every 3 months, in the week prior to the start of the menses
e. Every 6 months, in the week prior to the start of the menses

99/ . A married 41-year-old G5P3114 presents to your office for a routine examination. She reports

23
being healthy except for a history of migraine headaches. All her Pap smears have been normal. She
developed gestational diabetes in her last pregnancy. She drinks alcohol socially, and admits to
smoking occasionally. Her grandmother was diagnosed with ovarian cancer when she was in her
fifties. Her blood pressure is 140/90 mm Hg; height is 5 ft 5 in; weight is 150 lb. Which of the
following is the most common cause of death in women of this patient’s age?
a. HIV
b. Cardiac disease
c. Accidents
d. Suicide
e. Cancer

100/ . A 36-year-old G2P2 presents for her well-woman examination. She has had two spontaneous
vaginal deliveries without complications. Her largest child weighed 3500 g at birth. She uses
oral contraceptive pills and denies any history of an abnormal Pap smear. She does not smoke,
but drinks about four times per week. Her weight is 70 kg. Her vital signs are normal. After
place- ment of the speculum, you note a clear cyst approximately 2.5 cm in size on the lateral
wall of the vagina on the right side. The cyst is nontender and does not cause the patient any
dyspareunia or discomfort. Which of the following is the most likely diagnosis of this mass?
a. Bartholin duct cyst
b. Gartner duct cyst
c. Lipoma
d. Hematoma
e. Inclusion cyst
101/ . A 50-year-old G4P4 presents for her well-woman examination. She had one cesarean delivery
followed by three vaginal deliveries. Her menses stopped 1 year ago and she occasionally still
has a hot flash. She tells you that about 10 years ago she was treated with a laser conization for
carcinoma in situ of her cervix. Since that time, all of her Pap tests have been normal. What
recommendation should you make regarding how frequently she should undergo Pap smear
testing?
a. Every 3 months
b. Every 6 months
c. Every year
d. Every 2 years
e. Every 3 years
102/ . A 45-year-old G3P3 presents for her yearly examination. She last saw a doctor 7 years ago
after she had her last child. She had three vaginal deliveries, the last of which was complicated
by gestational diabetes and preeclampsia. She has not been sexually active in the past year. She
once had an abnormal Pap smear for which she underwent cryotherapy. She denies any medical
problems. Her family history is significant for coronary artery disease in her dad and a maternal
aunt who developed ovarian cancer at the age of 67. Which of the following is best screening
approach for this patient?
a. Pap smear
b. Pap smear and mammography
c. Pap smear, mammography, and cholesterol profile
d. Pap smear, mammography, cholesterol profile, and fasting blood sugar
e. Pap smear, mammography, cholesterol profile, fasting blood sugar, and serum CA-125
103/ . A 30-year-woman presents to your office with the fear of developing ovarian cancer. Her 70-
year-old grandmother recently died from ovarian cancer. You discuss with her the risks factors

24
and prevention for ovarian cancer. Which of the following can decrease a woman’s risk of
ovarian cancer?
a. Use of combination oral contraceptive therapy
b. Menopause after age 55
c. Nonsteroidal anti-inflammatory drugs
d. Nulliparity
e. Ovulation induction medications
104/ . A 42-year-old G4P3104 presents for her well-woman examination. She has had three vaginal
deliveries and one cesarean delivery for breech. She states her cycles are regular and denies any
sexually transmitted diseases. Currently she and her husband use condoms, but they hate the
hassle of a coital-dependent method. She is interested in a more effective contraception because
they do not want any more children. She reports occasional migraine headaches and had a
serious allergic reaction to anesthesia as a child when she underwent a tonsillectomy. She drinks
and smokes socially. She weighs 78 kg, and her blood pressure is 142/89 mm Hg. During her
office visit, you counsel the patient at length regarding birth control methods. Which of the
following is the most appropriate contraceptive method for this patient?
a. Intrauterine device
b. Bilateral tubal ligation
c. Combination oral contraceptives
d. Diaphragm
e. Transdermal patch
105/ . A 48-year-old G2P2 presents for her well-woman examination. She had two uneventful
vaginal deliveries. She had a vaginal hysterectomy for fibroids and menorrhagia. She denies any
medical problems, but has not seen a doctor in 6 years. Her family history is significant for
stroke, dia- betes, and high blood pressure. On examination she is a pleasant female, stands 5 ft
3 in tall, and weighs 85 kg. Her blood pressure is 150/92 mm Hg, pulse 70 beats per minute,
respiratory rate 14 breaths per minute, and tem- perature 37°C (98.4°F). Her breast, lung,
cardiac, abdomen, and pelvic examinations are normal. The next appropriate step in the
management of this patient’s blood pressure is which of the following?
a. Beta-blocker
b. Calcium channel blocker
c. Diuretic
d. Diet, exercise, weight loss, and repeat blood pressure in 2 months

Benign and Malignant Disorders of the Breast and Pelvis

106/ . A 50-year-old woman is diagnosed with cervical cancer. Which lymph node group would be
the first involved in metastatic spread of this disease beyond the cervix and uterus?
a. Common iliac nodes
b. Parametrial nodes
c. External iliac nodes
d. Paracervical or ureteral nodes
e. Para-aortic nodes

107/ . A 54-year-old woman undergoes a laparotomy because of a pelvic mass. At exploratory

25
laparotomy, a unilateral ovarian neoplasm is discovered that is accompanied by a large omental
metastasis. Frozen section diagnosis confirms metastatic serous cystadenocarcinoma. Which of
the following is the most appropriate intraoperative course of action?
a. Excision of the omental metastasis and ovarian cystectomy
b. Omentectomy and ovarian cystectomy
c. Excision of the omental metastasis and unilateral oophorectomy
d. Omentectomy and bilateral salpingo-oophorectomy
e. Omentectomy, total abdominal hysterectomy, and bilateral salpingo-oophorectomy
108/ . A 58-year-old woman is seen for evaluation of a swelling in her right vulva. She has also
noted pain in this area when walking and during coitus. At the time of pelvic examination, a
mildly tender, fluctuant mass is noted just outside the introitus in the right vulva in the region of
the Bartholin gland. Which of the following is the most appropriate treatment?
a. Marsupialization
b. Administration of antibiotics
c. Surgical excision
d. Incision and drainage
e. Observation
109/ . A 51-year-old woman is diagnosed with invasive cervical carcinoma by cone biopsy. Pelvic
examination and rectal-vaginal examination reveal the parametrium to be free of disease, but the
upper portion of the vagina is involved with tumor. Intravenous pyelography (IVP) and
sigmoidoscopy are negative, but a computed tomography (CT) scan of the abdomen and pelvis
shows grossly enlarged pelvic and periaortic nodes. This patient is classified at which of the
following stages?
a. IIa
b. IIb
c. IIIa
d. IIIb
e. IV
110/ . A 35-year-old G3P3 with a Pap smear showing high-grade squamous intraepithelial lesion of
the cervix (CIN III) has an inadequate colposcopy. Cone biopsy of the cervix shows squamous
cell cancer that has invaded only 1 mm beyond the basement membrane. There are no confluent
tongues of tumor, and there is no evidence of lymphatic or vascular invasion. The margins of
the cone biopsy specimen are free of disease.
How should you classify or stage this patient’s disease?
a. Carcinoma of low malignant potential
b. Microinvasive cancer, stage Ia1
c. Atypical squamous cells of undetermined significance
d. Carcinoma in situ
e. Invasive cancer, stage IIa
111/ . A 35-year-old G3P3 with a Pap smear showing high-grade squamous intraepithelial lesion of
the cervix (CIN III) has an inadequate colposcopy. Cone biopsy of the cervix shows squamous
cell cancer that has invaded only 1 mm beyond the basement membrane. There are no confluent
tongues of tumor, and there is no evidence of lymphatic or vascular invasion. The margins of
the cone biopsy specimen are free of disease.
The patient above now asks you for your advice on how to treat her cervical disease. Your
best recommendation is for the patient to undergo which of the following?
a. Treatment with external beam radiation
b. Implantation of radioactive cesium into the cervical canal c. Simple hysterectomy

26
d. Simple hysterectomy with pelvic lymphadenectomy
e. Radical hysterectomy

112/ . A pregnant 35-year-old patient is at highest risk for the concurrent development of which of
the following malignancies?
a. Cervix
b. Ovary
c. Breast
d. Vagina
e. Colon

113/ . A 22-year-old G3P0030 obese female comes to your office for a rou- tine gynecologic
examination. She is single, but is currently sexually active. She has a history of five sexual
partners in the past, and became sexually active at age 15. She has had three first-trimester
voluntary pregnancy ter- minations. She uses Depo-Provera for birth control, and reports
occasion- ally using condoms as well. She has a history of genital warts, but denies any prior
history of abnormal Pap smears. The patient denies use of any illicit drugs, but admits to
smoking about one pack of cigarettes a day. Her physical examination is normal. However, 3
weeks later you receive the results of her Pap smear, which shows a high-grade squamous
intraepithe- lial lesion (HGSIL). Which of the following factors in this patient’s history does not
increase her risk for cervical dysplasia?
a. Young age at initiation of sexual activity
b. Multiple sexual partners
c. History of genital warts
d. Use of Depo-Provera
e. Smoking

114/ . A 57-year-old menopausal patient presents to your office for evaluation of postmenopausal
bleeding. She is morbidly obese and has chronic hypertension and adult onset diabetes. An
endometrial sampling done in the office shows complex endometrial hyperplasia with atypia,
and a pelvic ultrasound done at the hospital demonstrates multiple, large uterine fibroids. Which
of the following is the best treatment option for this patient?
a. Myomectomy
b. Total abdominal hysterectomy
c. Oral contraceptives
d. Uterine artery embolization
e. Oral progesterone
Infertility, Endocrinology, and Menstrual Dysfunction
115/ . You see five postmenopausal patients in the clinic. Each patient has one of the conditions
listed, and each patient wishes to begin hormone replacement therapy today. Which one of the
following patients would you start on therapy at the time of this visit?
a. Mild essential hypertension
b. Liver disease with abnormal liver function tests
c. Malignant melanoma
d. Undiagnosed genital tract bleeding
e. Treated stage III endometrial cancer
116/ . A mother brings her 12-year-old daughter in to your office for consul- tation. She is concerned

27
because most of the other girls in her daughter’s class have already started their period. She
thinks her daughter hasn’t shown any evidence of going into puberty yet. Knowing the usual
first sign of the onset of puberty, you should ask the mother which of the following questions?
a. Has her daughter had any acne?
b. Has her daughter started to develop breasts?
c. Does her daughter have any axillary or pubic hair?
d. Has her daughter started her growth spurt?
e. Has her daughter had any vaginal spotting?

117/ . A 55-year-old woman presents to your office for consultation regard- ing her symptoms of
menopause. She stopped having periods 8 months ago and is having severe hot flushes. The hot
flushes are causing her considerable stress. What should you tell her regarding the
psychological symptoms of the climacteric?
a. They are not related to her changing levels of estrogen and progesterone.
b. They commonly include insomnia, irritability, frustration, and malaise.
c. They are related to a drop in gonadotropin levels.
d. They are not affected by environmental factors.
e. They are primarily a reaction to the cessation of menstrual flow.

118/ . While evaluating a 30-year-old woman for infertility, you diagnose a bicornuate uterus. You
explain that additional testing is necessary because of the woman’s increased risk of congenital
anomalies in which organ system?
a. Skeletal
b. Hematopoietic
c. Urinary
d. Central nervous
e. Tracheoesophageal
119/ . A 39-year-old G3P3 complains of severe, progressive secondary dysmenorrhea and
menorrhagia. Pelvic examination demonstrates a tender, diffusely enlarged uterus with no
adnexal tenderness. Results of endometrial biopsy are normal. Which of the following is the
most likely diagnosis?
a. Endometriosis
b. Endometritis
c. Adenomyosis
d. Uterine sarcoma
e. Leiomyoma
120/ . A 28-year-old G3P0 has a history of severe menstrual cramps, pro- longed, heavy periods,
chronic pelvic pain, and painful intercourse. All of her pregnancies were spontaneous abortions
in the first trimester. A hysterosalpingogram (HSG) she just had as part of the evaluation for
recurrent abortion showed a large uterine septum. You have recommended surgical repair of the
uterus. Of the patient’s symptoms, which is most likely to be corrected by resection of the
uterine septum?
a. Habitual abortion
b. Dysmenorrhea
c. Menometrorrhagia
d. Dyspareunia
e. Chronic pelvic pain
121/ . During the evaluation of infertility in a 25-year-old female, a hysterosalpingogram showed

28
evidence of Asherman syndrome. Which one of the following symptoms would you expect this
patient to have?
a. Hypomenorrhea
b. Oligomenorrhea
c. Menorrhagia
d. Metrorrhagia
e. Dysmenorrhea

122/ . A couple presents for evaluation of primary infertility. The evaluation of the woman is
completely normal. The husband is found to have a left varicocele. If the husband’s varicocele
is the cause of the couple’s infertility, what would you expect to see when evaluating the
husband’s semen analysis?
a. Decreased sperm count with an increase in the number of abnormal forms
b. Decreased sperm count with an increase in motility
c. Increased sperm count with an increase in the number of abnormal forms
d. Increased sperm count with absent motility
e. Azoospermia

123/ . A 25-year-old woman presents to your office for evaluation of primary infertility. She has
regular periods every 28 days. She has done testing at home with an ovulation kit, which
suggests she is ovulating. A hysterosalpingogram demonstrates patency of both fallopian tubes.
A progesterone level drawn in the mid–luteal phase is lower than expected. A luteal phase
defect is suspected to be the cause of this patient’s infertility. Which of the following studies
performed in the second half of the menstrual cycle is helpful in making this diagnosis?
a. Serum estradiol levels
b. Urinary pregnanetriol levels
c. Endometrial biopsy
d. Serum follicle-stimulating hormone (FSH) levels
e. Serum luteinizing hormone (LH) levels
124/ . A 45-year-old woman who had two normal pregnancies 15 and 18 years ago presents with the
complaint of amenorrhea for 7 months. She expresses the desire to become pregnant again.
After exclusion of pregnancy, which of the following tests is next indicated in the evaluation of
this patient’s amenorrhea?
a. Hysterosalpingogram
b. Endometrial biopsy
c. Thyroid function tests
d. Testosterone and DHAS levels
e. LH and FSH levels

125/ . Which of the following pubertal events in girls is not estrogen dependent?
a. Menses
b. Vaginal cornification
c. Hair growth
d. Reaching adult height
e. Production of cervical mucus

126/ . You suspect that your infertility patient has an inadequate luteal phase. She should undergo

29
an endometrial biopsy on which day of her menstrual cycle?
a. Day 3
b. Day 8
c. Day 14
d. Day 21
e. Day 26
127/ . You have recommended a postcoital test for your patient as part of her evaluation for
infertility. She and her spouse should have sexual intercourse on which day of her menstrual
cycle as part of postcoital testing?
a. Day 3
b. Day 8
c. Day 14
d. Day 21
e. Day 26
128/ . You ask a patient to call your office during her next menstrual cycle to schedule a
hysterosalpingogram as part of her infertility evaluation. Which day of the menstrual cycle is
best for performing the hysterosalpingogram?
a. Day 3
b. Day 8
c. Day 14
d. Day 21
e. Day 26
129/ . You have recommended that your infertility patient return to your office during her next
menstrual cycle to have her serum progesterone level checked. Which is the best day of the
menstrual cycle to check her proges- terone level if you are trying to confirm ovulation?
a. Day 3
b. Day 8
c. Day 14
d. Day 21
e. Day 26

Pelvic Relaxation and Urology


130/ . An 86-year-old woman presents to your office for her well-woman examination. She has no
complaints. On pelvic examination performed in the supine and upright positions, the patient
has second-degree prolapse of the uterus. Which of the following is the best next step in the
management of this patient?
a. Reassurance
b. Placement of a pessary
c. Vaginal hysterectomy
d. Le Fort procedure
e. Anterior colporrhaphy
131/ . An 81-year-old woman presents to your office complaining that her uterus fell out 2 months
ago. She has multiple medical problems, includ- ing chronic hypertension, congestive heart
failure, and osteoporosis. She is limited to sitting in a wheelchair because of her health
problems. Her fallen uterus causes significant pain. On physical examination, the patient is frail
and requires assistance with getting on the examination table. She has com- plete procidentia of
the uterus. Which of the following is the most appro- priate next step in the management of this

30
patient?
a. Reassurance
b. Placement of a pessary
c. Vaginal hysterectomy
d. Le Fort procedure
e. Anterior colporrhaphy
132/ . A 78-year-old woman with chronic obstructive pulmonary disease, chronic hypertension, and
history of myocardial infarction requiring angioplasty presents to your office for evaluation of
something hanging out of her vagina. She had a hysterectomy for benign indications at age 48.
For the past few months, she has been experiencing the sensation of pelvic pressure. Last month
she felt a bulge at the vaginal opening. Two weeks ago something fell out of the vagina. On
pelvic examination, the patient has total eversion of the vagina. There is a superficial ulceration
at the vaginal apex. Which of the following is the best next step in the management of this
patient?
a. Biopsy of the vaginal ulceration
b. Schedule abdominal sacral colpopexy
c. Place a pessary
d. Prescribe oral estrogen
e. Prescribe topical vaginal estrogen cream

133/ . A 28-year-old woman presents to your office with symptoms of a uri- nary tract infection.
This is her second infection in 2 months. You treated the last infection with Bactrim DS for 3
days. Her symptoms never really improved. Now she has worsening lower abdominal
discomfort, dysuria, and frequency. She has had no fever or flank pain. Physical examination
shows only mild suprapubic tenderness. Which of the following is the best next step in the
evaluation of this patient?
a. Urine culture
b. Intravenous pyelogram
c. Cystoscopy
d. Wet smear
e. CT scan of the abdomen with contrast

Human Sexuality and Contraception

134/ . A 28-year-old G3P3 presents to your office for contraceptive coun- seling. She denies any
medical problems or sexually transmitted diseases. You counsel her on the risks and benefits of
all contraceptive methods. Which of the following is the most common form of contraception
used by reproductive-age women in the United States?
a. Pills
b. Condom
c. Diaphragm
d. Intrauterine device (IUD)
e. Permanent sterilization

135/ . A 20-year-old woman presents to your office for her well-woman examination. She has

31
recently become sexually active and desires an effective contraceptive method. She has no
medical problems, but family history is significant for breast cancer in a maternal aunt at the
age of 42. She is worried about getting cancer from taking birth control pills. You discuss with
her the risks and benefits of contraceptive pills. You tell her that which of the following
neoplasms has been associated with the use of oral contraceptives?
a. Breast cancer
b. Ovarian cancer
c. Endometrial cancer
d. Hepatic cancer
e. Hepatic adenoma
136/ . An intrauterine pregnancy of approximately 10 weeks gestation is confirmed in a 30-year-old
G5P4 woman with an IUD in place. The patient expresses a strong desire for the pregnancy to
be continued. On examina- tion, the string of the IUD is noted to be protruding from the
cervical os. Which of the following is the most appropriate course of action?
a. Leave the IUD in place without any other treatment.
b. Leave the IUD in place and continue prophylactic antibiotics throughout pregnancy.
c. Remove the IUD immediately.
d. Terminate the pregnancy because of the high risk of infection.
e. Perform a laparoscopy to rule out a heterotopic ectopic pregnancy.

137/ . A 19-year-old woman presents for voluntary termination of pregnancy 6 weeks after her
expected (missed) menses. She previously had reg- ular menses every 28 days. Pregnancy is
confirmed by β-human chorionic gonadotropin (β-hCG), and ultrasound confirms expected
gestational age. Which of the following techniques for termination of pregnancy would be safe
and effective in this patient at this time?
a. Dilation and evacuation (D&E)
b. Hypertonic saline infusion
c. Suction dilation and curettage (D&C)
d. 15-methyl α-prostaglandin injection
e. Hysterotomy

138/ . A 22-year-old nulliparous woman has recently become sexually active. She consults you
because of painful coitus, with the pain located at the vaginal introitus. It is accompanied by
painful involuntary contraction of the pelvic muscles. Other than confirmation of these findings,
the pelvic examination is normal. Which of the following is the most common cause of this
condition?
a. Endometriosis
b. Psychogenic causes
c. Bartholin gland abscess
d. Vulvar atrophy
e. Ovarian cyst

139/ . Five patients present for contraceptive counseling, each requesting that an IUD be inserted.
Which of the following is a recognized contraindication to the insertion of an IUD?
a. Pelvic inflammatory disease
b. Previous pregnancy with an IUD
c. Dysfunctional uterine bleeding
d. Cervical conization

32
e. Chorioamnionitis in previous pregnancy

140/ . A couple presents to your office to discuss permanent sterilization. They have three children
and are sure they do not want any more. You discuss the risk and benefits of surgical
sterilization. Which of the following statements is true regarding surgical sterilizations?
a. They cannot be performed immediately postpartum.
b. They have become the second most common method of contraception for white couples
between 20 and 40 years of age in the United States.
c. They can be considered effective immediately in females (bilateral tubal ligation).
d. They can be considered effective immediately in males (vasectomy).
e. Tubal ligation should be performed in the secretory phase of the menstrual cycle.
141/ . A couple presents to your office to discuss sterilization. They are very happy with their four
children and do not want any more. You discuss with them the pros and cons of both female and
male sterilization. The 34-year- old male undergoes a vasectomy. Which of the following is the
most frequent immediate complication of this procedure?
a. Infection
b. Impotence
c. Hematoma
d. Spontaneous reanastomosis
e. Sperm granulomas
142/ . A woman with multiple sexual partners
For above female patient seeking contraception, select the method that is medically
contraindicated for that patient.
a. Oral contraceptives
b. IUD
c. Condoms
d. Laparoscopic tubal ligation
e. Diaphragm
143/ . A woman with a history of deep vein thrombosis
For above female patient seeking contraception, select the method that is medically
contraindicated for that patient.
a. Oral contraceptives
b. IUD
c. Condoms
d. Laparoscopic tubal ligation
e. Diaphragm
144/ . A woman with moderate cystocele
For above female patient seeking contraception, select the method that is medically
contraindicated for that patient.
a. Oral contraceptives
b. IUD
c. Condoms
d. Laparoscopic tubal ligation
e. Diaphragm
145/ . A woman with severely reduced functional capacity as a result of chronic obstructive lung
disease.
For above female patient seeking contraception, select the method that is medically
contraindicated for that patient.

33
a. Oral contraceptives
b. IUD
c. Condoms
d. Laparoscopic tubal ligation
e. Diaphragm
146/ . A woman with a known latex allergy.
For above female patient seeking contraception, select the method that is medically
contraindicated for that patient.
a. Oral contraceptives
b. IUD
c. Condoms
d. Laparoscopic tubal ligation
e. Diaphragm

147/ . Nausea during first cycle of pills


For above situation involving oral contraceptives, select the most appropriate response.
a. Stop pills and resume after 7 days.
b. Continue pills as usual.
c. Continue pills and use an additional form of contraception.
d. Take an additional pill.
e. Stop pills and seek a medical examination.

148/ . Pill forgotten for 1 day


For above situation involving oral contraceptives, select the most appropriate response.
a. Stop pills and resume after 7 days.
b. Continue pills as usual.
c. Continue pills and use an additional form of contraception.
d. Take an additional pill.
e. Stop pills and seek a medical examination.
149/ . Pill forgotten for 3 continuous days
For above situation involving oral contraceptives, select the most appropriate response.
a. Stop pills and resume after 7 days.
b. Continue pills as usual.
c. Continue pills and use an additional form of contraception.
d. Take an additional pill.
e. Stop pills and seek a medical examination.
150/ . Light bleeding at midcycle during first month on pill
For above situation involving oral contraceptives, select the most appropriate response.
a. Stop pills and resume after 7 days.
b. Continue pills as usual.
c. Continue pills and use an additional form of contraception.
d. Take an additional pill.
e. Stop pills and seek a medical examination.

34
Sexual Abuse and Domestic Violence

151/ . A 20-year-old woman presents to your office with the complaint of abdominal pain. Through
further questioning, the woman reveals that she was sexually assaulted at a party 3 weeks ago
by a male friend whom she recently started dating. She states that she has not revealed this to
anyone else and has not informed the police because she was drinking. Her abdominal and
pelvic examinations are normal. Which of the following is the best management to offer this
patient?
a. Counsel patient to sue male friend.
b. Provide an antidepressant.
c. Provide emergency contraception.
d. Test for and treat sexually transmitted infections.
e. Order CT of the abdomen and pelvis.

152/ . You are called to the emergency department to evaluate an 18-year- old woman for a vulvar
laceration. She is accompanied by her mother and father. The father explains that the injury was
caused by a fall onto the sup- port bar on her bicycle. You interview the woman alone and find
out that her father has been sexually assaulting her. Which of the following statements best
describes injuries related to sexual assault?
a. Most injuries are considered major and require surgical correction.
b. Most injuries require hospitalization.
c. More than 50% of victims will have an injury.
d. Most injuries occur after the assault has taken place.
e. Vaginal and vulvar lacerations are common in virginal victims.

153/ . You are an intern working the night shift in the emergency department. During the evaluation
of a sexual assault victim, your attending physician asks you to order the appropriate laboratory
tests. Which of the following tests should be ordered?
a. HIV, HBsAg, Pap smear, RPR, and urine culture
b. HIV, HBsAg, Pap smear, RPR, and urine pregnancy test
c. Chlamydia and gonorrhea cultures, complete blood count, HIV, HBsAg, Pap smear, and
RPR
d. Chlamydia and gonorrhea cultures, HIV, HBsAg, Pap smear, RPR, and urine pregnancy
test
e. Chlamydia and gonorrhea cultures, HIV, HBsAg, RPR, urine culture, and urine
pregnancy test

154/ . You are evaluating a 19-year-old woman for a sexual assault. She denies any medical
problems or allergies to medications. Her pregnancy test is negative. Which of the following
antibiotic prophylaxes do you recommend for sexually transmitted infections?
a. No antibiotic prophylaxis is indicated
b. Flagyl 500 mg PO twice daily for 7 days
c. Rocephin 250 mg IM
d. Doxycycline 100 mg PO twice daily for 7 days plus Rocephin 250 mg IM
e. Erythromycin 500 mg PO twice daily for 7 days

35
155/ . After your evaluation and treatment of a rape victim has been com- pleted, you
discharge the patient to home. When is the best time to schedule a follow-up
appointment for the patient?
a. 24
to 48
hours b.
1 week
c.
6
we
eks
d.
12
we
eks
e. There is no need for the patient to have any additional follow-up as long
as she feels well.

I- Choose the most appropriate answer:

1- Ovulation is apparently initiated by which of the following hormones:


a) Progesterone b) Luteinizing hormone
c) Follicle stimulating hormone d) Estrogen

2- Each month a developing graafian follicle (corpus luteum) produce estrogen, which
builds up the endomatrium of the uterus, this is called:
a) The proliferation phase b) The secretory phase
c) Menstrual phase d) Pre-menstrual phase

3- During secretory phase, the corpus luteum begin to secrete which of the following
hormones:
a) Progesterone b) Luteinizing hormone
c) LH d) FSH

4- Fertilization usually occurs in the:


a) Uterus b) Vagina
c) Medial portion of the fallopian tube d) Distal portion of the fallopian tube

5- Which of the following is/are responsible for the migration of the


fertilized ovum to the uterine cavity:
a) Peristaltic action in the fallopian tube
b) Cilia in the fallopian tube
c) Currents setup by peristaltic and ciliary actions in the fallopian tube
d) All of the above

6- How many chromosomes do the mature ovum contain:

36
b) 23 single e) 46 single
a) 23 pairs d) 46 pair
c) chro. chro.

7- The principal course of progesterone and estrogen during the first sixth to eighth
week of pregnancy is the:
a) Pituitary gland b) Adrenal cortex
c) Corpus luetum d) Placenta

8- Which of the following is the approximate time that the blastocyst spends
traveling to the uterus for implantation?
2 days 10 days

7 days 14 weeks

9- Infertility in a 25-year-old couple is defined as which of the following?


a) The couple's inability to conceive after 6 months of unprotected attempts
b) The couple's inability to sustain a pregnancy
c) The couple's inability to conceive after 1 year of unprotected attempts
d) A low sperm count and decreased motility

10- After ovulation has occurred, the ovum remains viable for a period of approximately:
a) 1 – 2 hours b) 10 – 12 hours
c) 3 – 6 days d) 24 – 48 hours

11- During the first 3 months, which of the following hormones is responsible for
maintaining pregnancy?
a) Human chorionic gonadotropin
b) Progesterone
(HCG)
c) Estrogen d) Relaxin
12- Which of the following symptoms are considered positive signs of
pregnancy:
a) Amenorrhea b) Frequency of urination
c) Heaviness and tingling in
d) None of them
breasts

13- Chadwick’s sign is described as:


a) A purplish discoloration of vagina
b) Decreased oxygen tension of maternal blood
c) Rapid growth of microorganisms in the vaginal secretions
d) Softening of the cervix

14- A primigravida may start to perceive quickening at:


10th week of pregnancy 18th week of pregnancy

26th week of pregnancy 34th week of pregnancy

37
15- Heartburn during pregnancy is thought to result from:
a) Gastritis due to hyperacidity and hyper-peristalsis.
b) Excessive secretion of hydrochloric acid by gastric mucosa
c) Friction of the pericardium against the diaphragm elevated by the enlarged
uterus
d) Tendency of acid content of the compressed and displaced stomach to
reflux into lower esophegus

16- Varicose veins of the lower extremities may develop during pregnancy
as a result of:
a) Systemic circulation microorganisms
b) Increased venous blood volume due to return of blood from the fetus
c) Compression of the abdominal veins by the progressively enlarged uterus
d) Increased cardiac metabolic load imposed by the oxygen needs of the
growing fetus.

17- Nausea and vomiting in early pregnancy has been linked with the
following except:
b) Increased metabolic rate of
a) High level of HCG
pregnancy
c) Habitual abortion d) Emotional reactions to pregnancy

18- A pregnant client's last menstrual period began on October 12. The nurse calculates the
estimated date of delivery (EDD) as:
a) June 5 b) June 19
c) July 5 d) July 19

Situation 1:

Mona is pregnant for the first time. She visited the clinic when she was thirty weeks
pregnant. She complained of swollen feet, legs, and hands. She also had headache,
blurred vision, excessive weight gain, blood pressure of 144/96 and +2 proteinuria.

19- What is Mona’s diagnosis?


a) Essential hypertension b) Pre-eclampsia
c) Eclampsia d) Clomeruonephritis

20- Increasing of Mona’s weight result from:


Retention of sodium and water Increased blood pressure

Increase progesterone level Kidney lesion

21- Mona’s hypertension occurred in response to:


a) A toxin proliferation by the uterus
b) A toxin produced in the kidney
c) Emotional tension

38
d) Generalized arteriolar spasm of unknown cause.
22- A client with pregnancy-induced hypertension (PIH) probably exhibits
which of the following symptoms?
a) Proteinuria, headaches, and vaginal bleeding
b) Headaches, double vision, and vaginal bleeding
c) Proteinuria, headaches, and double vision
d) Proteinuria, double vision, and uterine contractions

23- A pregnant client comes to the facility for her first prenatal visit. After obtaining her
health history and performing a physical examination, the nurse reviews the client's
laboratory test results. Which findings suggest iron deficiency anemia?
a) Hb 15 g/L; HCT 33% b) Hb 13 g/L; HCT 32%
c) Hb 9 g/L; HCT 30% d) Hb 10 g/L; HCT 35%

24- The nurse is caring for a 16-year-old pregnant client. The client is taking an iron
supplement. What should this client drink to increase the absorption of iron?
a) A glass of milk b) A cup of hot tea
c) A liquid antacid d) A glass of orange juice

25- A client is admitted to the facility with a suspected ectopic pregnancy.


When reviewing the client's health history for risk factors for this abnormal
condition, the nurse expects to find:
a) History of pelvic inflammatory disease.
b) Grand multiparity (five or more births).
c) Use of an intrauterine device for 1 year.
d) Use of an oral contraceptive for 5 years

26- A client is in the 8th month of pregnancy. To enhance cardiac output and renal function,
the nurse should advise her to use which body position?
a) Right lateral b) Left lateral
c) Supine d) Semi-Fowler's

27- The nurse is planning care for a 16-year-old client in the prenatal clinic. Adolescents are
prone to which complication during pregnancy?
a) Iron deficiency anemia b) Varicosities
c) Nausea and vomiting d) Gestational diabetes

28- A client who's 2 months pregnant complains of urinary frequency and


says she gets up several times at night to go to the bathroom. She denies
other urinary symptoms. How should the nurse intervene?
a) Advise the client to decrease her daily fluid intake.
b) Refer the client to a urologist for further investigation.
c) Explain that urinary frequency isn't a sign of urinary tract infection (UTI).
d) Explain that urinary frequency is expected during the first trimester.

29- A low-risk client who's 6 weeks pregnant comes to the clinic for her first
prenatal visit. At this time, the nurse should assign highest priority to:
a) Establishing a schedule of prenatal visits.
b) Scheduling an ultrasound test to confirm the pregnancy.
c) Enrolling the client in a childbirth class.

39
d) Scheduling genetic testing for the client.

30- The nurse is assessing a pregnant woman. Which signs or symptoms indicate a
hydatidiform mole?
a) Rapid fetal heart tones b) Abnormally high (HCG) levels
c) Slow uterine growth d) Lack of symptoms of pregnancy
31- During a routine prenatal visit, a pregnant client reports heartburn. To minimize her
discomfort, the nurse should include which suggestion in the plan of care?
a) Eat small, frequent meals b) Limit fluid intake sharply
c) Drink more citrus juice d) Take sodium bicarbonate

32- Station +2 means:


a) Presenting part is 2cm above the level of ischial spine.
b) Biparietal is at the level of ischial spine.
c) Presenting part is 2cm below the level of ischial spine.
d) Biparietal diameter is 2cm above the ischial spine.

33- Lightening can best described as:


a) Stretching of the cervix
b) First movement of the fetus
c) Bloody discharge from the superficial mucosa of the cervical canal.
d) Presenting part descends into the pelvis far enough to be fixed.

34- Which of the following is a characteristic of true labor contraction:


a) Cervical dilatation does not occur.
b) Discomfort is usually located in the abdomen
c) Contractions are more frequent and have some type of regular pattern.
d) Contraction may relieve by walking.

35- Which of the following is a possible explanation of the onset of labor.


a) Progesterone level rises at term to initiate contractions
b) The ovaries release additional estrogen at term.
c) Prostaglandin may be the causative factor of labor.
d) None of them.

36- Effacement is the process by which the cervix is:


a) Open to its widest diameter
b) Incorporate into the lower uterine segment
c) Expulsion of the mucus plug occluding it
d) Forced to produce into the upper vagina.

37- A client, 38 weeks pregnant, arrives in the emergency department


complaining of contractions. To help confirm that she's in true labor, the
nurse should assess for:
a) Irregular contractions.
b) Increased fetal movement.
c) Changes in cervical effacement and dilation after 1 to 2 hours.
d) Contractions that feel like pressure in the abdomen and groin.

40
38- For a client who's fully dilated, which of the following actions would be
inappropriate during the second stage of labor?
a) Positioning the mother for effective pushing
b) Preparing for delivery of the baby
c) Assessing vital signs every 15 minutes
d) Assessing for rupture of membranes

39- When caring for a client who's having her second baby, the nurse can
anticipate the client's labor will be which of the following?
a) Shorter than her first labor
b) About half as long as her first labor
c) About the same length of time as her first labor
d) A length of time that can't be determined based on her first labor
40- The nurse-midwife determines that a client is in the second stage of
labor and may start pushing. What marks the beginning of the second
stage, and what marks the end?
a) Cervical dilation of 7 to 8 cm; complete cervical dilation
b) Complete cervical dilation; delivery of the neonate
c) Cervical dilation of 7 to 8 cm; delivery of the placenta
d) Complete cervical dilation; delivery of the placenta

41- The nurse is caring for a client who's in the first stage of labor. What is the shortest
but most difficult part of this stage?
a) Active phase b) Complete phase
c) Latent phase d) Transitional phase

42- Because cervical effacement and dilation aren't progressing in a client


in labor, the physician orders I.V. administration of oxytocin (Pitocin). Why
must the nurse monitor the client's fluid intake and output closely during
oxytocin administration?
a) Oxytocin causes water intoxication.
b) Oxytocin causes excessive thirst.
c) Oxytocin is toxic to the kidneys.
d) Oxytocin has a diuretic effect.

43- When assessing a client 1 hour after vaginal delivery, the nurse notes blood gushing
from the vagina, pallor, and a rapid, thready pulse. What do these findings suggest?
a) Uterine involution b) Cervical laceration
c) Placental separation d) Postpartum hemorrhage

44- A client is in the second stage of labor. During this stage, how frequently should the
nurse assess her uterine contractions?
a) Every 5 minutes b) Every 15 minutes
c) Every 30 minutes d) Every 60 minutes

45- When caring for a client in the first stage of labor, the nurse documents cervical
dilation of 9 cm and intense contractions lasting 45 to 60 seconds and occurring about
every 2 minutes. Based on these findings, the nurse should recognize that the client is in
which phase of labor?

41
a) Active phase b) Latent phase
c) Descent phase d) Transitional phase
Women,s care during the first 2h after birth -
-Check B/P & pulse every 15minutes A
-Check temperature every hour B
-Check uterine contraction &fundal level every 15 minutes C
-All the above D
All of the following situation lead to cord prolapse except -2
A-presenting parte will fit on cervix
B-High parity
C-Prematurity or polyhydramnios
D-cord presentation
Management of cord presentation include all of the following except -3
A-reduce the vaginal examination
B-Help mother to change position (Knee chest position)
C-Stop fetal monitoring
D-Administering oxygen to mother by face mask to improve fetal oxygenation
4- The first step that a midwife would take when a prolapsed cord is detected is.

A-Place the mother in a position that reduces compression of the cord

B-Refer rapidly to the operation room

C-Handle a prolapsed cord and reinsert it

D-Start IV fluid

5-A woman with a ruptured uterus has which of the following signs and symptoms.

A-Rapid maternal pulse

B-Persistent abdominal pain and suprapubic tenderness

C-Fetal distress

D-All of the above

42
6- Cardiovascular signs and symptoms of amniotic fluid embolism includes:

A-Restlessness

B-Convulsion

C-Dyspnea

D-Tachycardia

7- Vaginal bleeding immediately after birth in the presence of a well contracted uterus is

most often due to:

A-Uterine atony

B-Endometritis

C-Genital trauma

D-Abnormal clotting mechanism

8- After repair of the cervical tear client hemoglobin is found to be 10 g/dL, and her

vital signs are stable. What is the most appropriate plan of care?

A-Begin transfusing blood

B-Send her home

C-Monitor her vital signs encourage breastfeeding

D-Continue administration of oxytocin for 24 hours

9-32 years old MG is admitted to the postpartum unit after a cesarean section , the
midwife plans to notify physician immediately if the client experiences
A-. Pain in her calf
B-.Hypotension
C- Dyspnea and chest pain
D- d.Anorexia
10-The first step that a midwife would take when a prolapsed cord is detected is.
A-Place the mother in a position that reduces compression of the cord

B-Refer rapidly to the operation room

C-Handle a prolapsed cord and reinsert it

D-Start IV fluid

I- Matching
A B

1. Corpus -------- Neck of the uterus

2. Fundus -------- Extend from either side of corpus

43
3. Bi-Ischial -------- Triangular body of the uterus

4. Isthmus -------- Distance between the ischial tuberosities

5. Cervix -------- Located above the insertion of fallopian tube

The nurse is assessing the fetal heart rate (FHR) of a client admitted to the labor and
delivery area at term. Which of the following should the nurse identify as the normal
range of the baseline FHR?
a) 60 to 80 beats/minute
b) 80 to 120 beats/minute
c) 120 to 160 beats/minute
d) 160 to 200 beats/minute
Rationale: In a full-term fetus, the baseline FHR normally ranges from 120 to 160
beats/minute. FHR below 120 beats/minute reflects bradycardia; above 160 beats/minute,
tachycardia.

Which of the following physiologic changes during labor makes it necessary for the
nurse to assess blood pressure frequently?
a) Blood pressure decreases as a sign of maternal pain.
b) Alterations in cardiovascular function affect the fetus.
c) Blood pressure decreases at the peak of each contraction.
d) Decreased blood pressure is the first sign of preeclampsia
Rationale: During contractions, blood pressure increases and blood flow to the intervillous
spaces decreases, compromising the fetal blood supply. Therefore, the nurse should
frequently assess the client's blood pressure to determine whether it returns to
precontraction levels and allows adequate fetal blood flow. During pain and contractions,
the maternal blood pressure usually increases, rather than decreases. Preeclampsia
causes the blood pressure to increase — not decrease.

Which of the following behaviors would cause the nurse to suspect that a client's
labor is moving quickly and that the physician should be notified?
a) An increased sense of rectal pressure
b) A decrease in intensity of contractions
c) An increase in fetal heart rate variability
d) Episodes of nausea and vomiting
Rationale: An increased sense of rectal pressure indicates that the client is moving into the
second stage of labor. The nurse should be able to discern that information by the client's
behavior. Contractions don't decrease in intensity, there isn't a change in fetal heart rate
variability, and nausea and vomiting don't usually occur.

Which of the following would be least likely to indicate anticipated bonding


behaviors by new parents?

a) The parents' willingness to touch and hold the newborn


b) The parents' expression of interest about the size of the newborn
c) The parents' indication that they want to see the newborn
II- The parents' interactions with each other
Rationale: Parental interaction will provide the nurse with a good assessment of the
stability of the family's home life but it has no indication for parental bonding. Willingness to

44
touch and hold the newborn, expressing interest about the newborn's size, and indicating a
desire to see the newborn are behaviors indicating parental bonding.

Initial client assessment information includes the following: blood pressure 160/110
mm Hg, pulse 88 beats/minute, respiratory rate 22 breaths/minute, reflexes +3/+4
with 2 beat clonus. Urine specimen reveals +3 protein, negative sugar and ketones.
Based on these findings, the nurse would expect the client to have which of the
following complaints?
a) Headache, blurred vision, and facial and extremity swelling
b) Abdominal pain, urinary frequency, and pedal edema
c) Diaphoresis, nystagmus, and dizziness
d) Lethargy, chest pain, and shortness of breath
Rationale: The client is exhibiting signs of preeclampsia. In addition to hypertension and
hyperreflexia, most preeclamptic clients have edema. Headache and blurred vision are
indications of the effects of the hypertension. Abdominal pain, urinary frequency,
diaphoresis, nystagmus, dizziness, lethargy, chest pain, and shortness of breath are
inconsistent with a diagnosis of preeclampsia.

After completing a second vaginal examination of a client in labor, the nurse-midwife


determines that the fetus is in the right occiput anterior position and at –1 station.
Based on these findings, the nurse-midwife knows that the fetal presenting part is:
a) 1 cm below the ischial spines.
b) Directly in line with the ischial spines.
c) 1 cm above the ischial spines.
d) In no relationship to the ischial spines.
Rationale: Fetal station — the relationship of the fetal presenting part to the maternal
ischial spines — is described in the number of centimeters above or below the spines. A
presenting part above the ischial spines is designated as –1, –2, or –3. A presenting part
below the ischial spines, as +1, +2, or +3.

Anxiety related to the facility environment

Fear related to a potentially difficult childbirth

Ineffective family coping: Compromised related to hospitalization

Pain related to labor contractions

Rationale: A client's ability to cope during labor and delivery may be hampered by fear of a
painful or difficult childbirth, fear of loss of control or self-esteem during childbirth, or fear of
fetal death. A previous negative experience may increase these fears. Therefore, Fear
related to a potentially difficult childbirth is the most appropriate nursing diagnosis. The
client's anxiety stems from her past history of a long labor, not from being in the facility;
therefore a diagnosis of Anxiety related to the facility environment isn't warranted. There is
no evidence of compromised ineffective family coping related to hospitalization. Although
pain related to labor contractions may be a problem, this isn't mentioned in the question.

The nurse is caring for a client in labor. Which assessment finding indicates fetal
distress?
45
a) Lack of meconium staining
b) Early decelerations in fetal heart rate during contractions
c) An increase in fetal heart rate with fetal scalp stimulation
d) Fetal blood pH less than 7.2
Rationale: A fetal blood pH less than 7.2 is an indication of fetal hypoxia. During labor, a
fetal pH range of 7.2 to 7.3 is considered normal. Fetal blood is sampled from the fetal
scalp through a dilated cervix. The other options are all normal findings.

When caring for a client with preeclampsia, which of the following actions is a
priority?

a) Monitoring the client's labor carefully and preparing for a fast delivery
b) Continually assessing the fetal tracing for signs of fetal distress
c) Checking vital signs every 15 minutes to watch for increasing blood pressure
d) Reducing visual and auditory stimulation
Rationale: A client with preeclampsia is at risk for seizure activity because her neurologic
system is overstimulated. Therefore, in addition to administering pharmacologic
interventions to reduce the possibility of seizures, the nurse should lessen auditory and
visual stimulation. Although the other actions are important, they're of a lesser priority.

A client in the first stage of labor enters the labor and delivery area. She seems
anxious and tells the nurse that she hasn't attended childbirth education classes.
Her husband, who accompanies her, is also unprepared for childbirth. Which
nursing intervention would be most effective for the couple at this time?
a) Teach the client progressive muscle relaxation.
b) Instruct the husband on touch, massage, and breathing patterns.
c) Use hypnosis on the client and her husband.
d) Teach the client and her husband about pain transmission
Rationale: If the unprepared client has a support person, the nurse should focus on that
person's supporting role, demonstrating touch, massage, and simple breathing patterns.
The other options are inappropriate at this time because they may make the client and her
husband more anxious.

A nulliparous client has been in the latent phase of the first stage of labor for several
hours. Despite continued uterine contractions, her cervix hasn't dilated further since
the initial examination. Her latent phase may be considered prolonged after:

a) 6 hours.
b) 10 hours.
c) 14 hours.
d) 20 hours.
Rationale: Based on research, the latent phase may be considered prolonged if it exceeds
20 hours in a nulliparous client or 14 hours in a multiparous client.

A client's husband asks the nurse whether he can do anything more to help his wife
during labor. What should the nurse suggest?
a) Encourage her to talk during contractions.
b) Frequently ask her how she feels.
c) Provide helpful distractions.
d) Ask about the progress of other clients

46
Rationale: A partner can be most helpful to a client in labor by providing helpful
distractions, timing contractions, coaching breathing, providing a calming influence,
reducing loneliness, and communicating the client's needs to health care professionals.
Encouraging her to talk and frequently asking her how she feels would be more of a
hindrance than a help. Asking about the progress of other clients doesn't address his
concerns about his wife.

A client who has been in the latent phase of the first stage of labor enters the
transition to the active phase. During the transition, the nurse expects to see which
client behavior?

a) A desire for personal contact and touch


b) A full response to teaching
c) Fatigue, a desire for touch, and quietness
d) Withdrawal, irritability, and resistance to touch
Rationale: During the transition to the active phase of the first stage of labor, increased
pain typically makes the client withdrawn, irritable, and resistant to touch. During the latent
phase (the early part of the first stage of labor), when contractions aren't intensely painful,
the client typically desires personal contact and touch and responds to teaching and
interventions. Fatigue, a desire for touch, and quietness are common during the third and
fourth stages of labor.

A client who's a gravida 1 para 0 has been admitted to the perinatal admission unit
and is in early labor. The client's cervical examination would reveal which of the
following?
a) cm dilated; 100% effaced at 0 station
b) 4 to 5 cm dilated; 80% effaced at –1 station
c) 2 cm dilated; 50% effaced at +1 station
d) 3 cm dilated; 50% effaced at 0 station
Rationale: The nurse must distinguish between the primigravida and multigravida cervical
dilation to make a plan of care for the laboring client. Primigravidas will efface and then
dilate, while multigravidas will efface and dilate at the same time.

A pregnant client arrives at the health care facility, stating that her bed linens were
wet when she woke up this morning. She says no fluid is leaking but complains of
mild abdominal cramps and lower back discomfort. Vaginal examination reveals
cervical dilation of 3 cm, 100% effacement, and positive ferning. Based on these
findings, the nurse concludes that the client is in which phase of the first stage of
labor?

a) Active phase
b) Latent phase
c) Expulsive phase
d) Transitional phase
Rationale: The latent phase of the first stage of labor is associated with irregular, short,
mild contractions; cervical dilation of 3 to 4 cm; and abdominal cramps or lower back
discomfort. During the active phase, the cervix dilates to 7 cm and moderately intense
contractions of 40 to 50 seconds' duration occur every 2 to 5 minutes. Fetal descent
continues throughout the active phase and into the transitional phase, when the cervix

47
dilates from 8 to 10 cm and intense contractions of 45 to 60 seconds' duration occur every
1½ to 2 minutes. The first stage of labor doesn't include an expulsive phase.

The third stage of labor ends with which of the following?

a) The birth of the baby


b) When the client is fully dilated
c) After the birth of the placenta
d) When the client is transferred to her postpartum bed
Rationale: The definition of the third stage of labor is the birth of the placenta. The first
stage of labor ends with complete cervical dilation and effacement. The second stage of
labor ends with the birth of the baby. The fourth stage of labor includes the first 4 hours
after birth

When assessing a client who has just delivered a neonate, the nurse finds that the
fundus is boggy and deviated to the right. What should the nurse do?

a) Have the client void.


b) Assess the client's vital signs.
c) Evaluate lochia characteristics.
d) Massage the fundus
Rationale: Having the client void can determine whether the boggy, deviated fundus
results from a full bladder — the most common cause of these fundal findings. Vital sign
assessment is unnecessary unless the nurse suspects hemorrhage from delayed
involution. In a client who doesn't have a full bladder, the nurse should evaluate lochia
characteristics to detect possible hemorrhage. If the client has a full bladder, massaging
the fundus won't stimulate uterine contractions (which aid uterine involution) or prevent
uterine atony — a possible cause of hemorrhage.

A client is in the third stage of labor. Which finding indicates impending placental
separation?

a) Increased maternal anxiety


b) Severe rectal pressure
c) Increased bloody show
d) Umbilical cord lengthening
Rationale: Signs of placental separation include lengthening of the umbilical cord, a
sudden gush of dark blood from the vagina, and a change in uterine shape from disklike to
globular. Increased maternal anxiety occurs during the transitional phase of the first stage
of labor. Severe rectal pressure and increased bloody show are common during the second
stage.

At 40 weeks' gestation, a client is admitted to the labor and delivery area. She and
her husband are worried about the fetus's health because she had problems during
her previous childbirth. The nurse reassures them that the fetus will be monitored
closely with an electronic fetal monitor (EFM). On the fetal monitor strip, what is the
single most reliable indicator of fetal well-being?
a) Normal long-term variability
b) Normal short-term variability
c) Normal baseline fetal heart rate (FHR)
d) Normal contraction sequence

48
Rationale: Normal short-term variability — 2 to 3 beats per amplitude — is the single most
reliable indicator of fetal well-being on an EFM strip. It represents actual beat-to-beat
fluctuations in the FHR. Normal long-term variability, although a helpful indicator, takes into
account larger periodic and rhythmic deviations above and below the baseline FHR.
Baseline FHR serves only as a reference for all subsequent FHR readings taken during
labor. Contraction sequence provides no information about fetal well-being, although it
does give some indication of maternal well-being and progress.

During the fourth stage of labor, the client should be assessed carefully for:

a) Uterine atony.
b) Complete cervical dilation.
c) Placental expulsion.
d) Umbilical cord prolapse.
Rationale: Uterine atony should be carefully assessed during the fourth stage. The second
stage of labor begins with complete cervical dilation and ends with birth. The third stage
begins immediately after birth and ends with the separation and expulsion of the placenta.
Immediately after delivery, the placenta is evaluated carefully for completeness, and the
client is assessed for excessive bleeding or a relaxed uterus. Umbilical cord prolapse,
displacement of the umbilical cord to a position at or below the fetus's presenting part,
occurs most commonly when amniotic membranes rupture before fetal descent. The client
should be assessed for a visible or palpable umbilical cord in the birth canal, violent fetal
activity, or fetal bradycardia with variable deceleration during contractions. The presence of
umbilical cord prolapse requires an emergency delivery.

During assessment for admission to the labor and delivery area, a client and her
husband ask the nurse whether their sons, ages 8 and 10, can witness the childbirth.
Before answering this question, the nurse should consider which guideline?
a) The children and client should share a support person during the childbirth.
b) Children should attend childbirth only if it takes place at home.
c) Children shouldn't attend childbirth because it will frighten them.
d) Each child attending the childbirth should have a separate support person.
Rationale: Each child attending the childbirth should have a support person — one who
isn't also serving as the client's support person. The support person explains what is
happening, reassures the child, and removes the child from the area if an emergency
occurs or if the child becomes frightened. Children can attend childbirth in any setting. The
decision to have a child present hinges on the child's developmental level, ability to
understand the experience, and amount of preparation

The nurse is caring for a client who's in labor. The physician still isn't present. After
the baby's head is delivered, which nursing intervention would be most appropriate?

a) Checking for the umbilical cord around the baby's neck


b) Placing antibiotic ointment in the baby's eyes
c) Turning the baby's head to the side to drain secretions
d) Assessing the baby for respirations
Rationale: After the baby's head is delivered, the nurse should check for the cord around
the baby's neck. If the cord is around the neck, it should be gently lifted over the baby's
head. Antibiotic ointment is administered to the baby after birth, not during delivery of the
49
head, to prevent gonorrheal conjunctivitis. The baby's head isn't turned during delivery.
After delivery, the baby is held with the head lowered to help with drainage of secretions. If
a bulb syringe is available, it can be used to gently suction the baby's mouth. Assessing the
baby's respiratory status should be done immediately after delivery.

A client who's being admitted to labor and delivery has the following assessment
findings: gravida 2 para 1, estimated 40 weeks' gestation, contractions 2 minutes
apart, lasting 45 seconds, vertex +4 station. Which of the following would be the
priority at this time?
a) Placing the client in bed to begin fetal monitoring
b) Preparing for immediate delivery
c) Checking for ruptured membranes
d) Providing comfort measures
Rationale: This question requires an understanding of station as part of the intrapartal
assessment process. Based on the client's assessment findings, this client is ready for
delivery, which is the nurse's top priority. Placing the client in bed, checking for ruptured
membranes, and providing comfort measures could be done, but the priority here is
immediate delivery.

When assessing the fetal heart rate tracing, the nurse becomes concerned about the
fetal heart rate pattern. In response to the loss of variability, the nurse repositions
the client to her left side and administers oxygen. These actions are likely to improve
which of the following?
a) Fetal hypoxia
b) The contraction pattern
c) The status of a trapped cord
d) Maternal comfort
Rationale: These actions, which will improve fetal hypoxia, increase the amount of
maternal circulating oxygen by taking pressure created by the uterus off the aorta and
improving blood flow. These actions won't improve the contraction pattern, free a trapped
cord, or improve maternal comfort.

Labor is divided into how many stages?

a) Five
b) Three
c) Two
d) Four
Rationale: Labor is divided into four stages: first stage, onset of labor to full dilation;
second stage, full dilation to birth of the baby; third stage, birth of the placenta; and fourth
stage, 1-hour postpartum. The first stage is divided into three phases: early, active, and
transition.

The nurse assesses a client for evidence of postpartum hemorrhage during the third
stage of labor. Early signs of this postpartum complication include:
a) An increased pulse rate, decreased respiratory rate, and increased blood
pressure.
b) A decreased pulse rate, increased respiratory rate, and increased blood
pressure.

50
c) A decreased pulse rate, decreased respiratory rate, and increased blood
pressure.
d) An increased pulse rate, increased respiratory rate, and decreased blood
pressure.
Rationale: An increased pulse rate followed by an increased respiratory rate and
decreased blood pressure may be the first signs of postpartum hemorrhage and
hypovolemic shock

The nurse is caring for a woman receiving a lumbar epidural anesthetic block to
control labor pain. What should the nurse do to prevent hypotension?

a) Administer ephedrine to raise her blood pressure.


b) Administer oxygen using a mask.
c) Place the woman flat on her back with her legs raised.
d) Ensure adequate hydration before the anesthetic is administered.
Rationale: Because the woman is in a state of relative hypovolemia, administering fluids
I.V. before the epidural anesthetic is given may prevent hypotension. Administration of an
epidural anesthetic may lead to hypotension because blocking the sympathetic fibers in the
epidural space reduces peripheral resistance. Ephedrine may be administered after an
epidural block if a woman becomes hypotensive and shows evidence of cardiovascular
decompensation. However, ephedrine isn't administered to prevent hypotension. Oxygen is
administered to a woman who becomes hypotensive, but it won't prevent hypotension.
Placing a pregnant woman in the supine position can contribute to hypotension because of
uterine pressure on the great vessels

Which of the following describes the term fetal position?

a) Relationship of the fetus's presenting part to the mother's pelvis


b) Fetal posture
c) Fetal head or breech at cervical os
d) Relationship of the fetal long axis to the mother's long axis
Rationale: Fetal position refers to the relationship of the fetus's presenting part to the
mother's pelvis. Fetal posture refers to "attitude." Presentation refers to the part of the fetus
at the cervical os. Lie refers to the relationship of the fetal long axis to that of the mother's
long axis.

A primigravid client is admitted to the labor and delivery area. Assessment reveals
that she's in the early part of the first stage of labor. Her pain is likely to be most
intense

a) Around the pelvic girdle.


b) Around the pelvic girdle and in the upper legs.
c) Around the pelvic girdle and at the perineum.
d) At the perineum.
Rationale: During most of the first stage of labor, pain centers around the pelvic girdle.
During the late part of this stage and the early part of the second stage, pain spreads to the

51
upper legs and perineum. During the late part of the second stage and during childbirth,
intense pain occurs at the perineum.

A client with intrauterine growth retardation is admitted to the labor and delivery unit
and started on an I.V. infusion of oxytocin (Pitocin). Which of the following is least
likely to be included in her plan of care?

a) Carefully titrating the oxytocin based on her pattern of labor


b) Monitoring vital signs, including assessment of fetal well-being, every 15 to 30
minutes
c) Allowing the client to ambulate as tolerated
d) Helping the client use breathing exercises to manage her contractions
Rationale: Because the fetus is at risk for complications, frequent and close monitoring is
necessary. Therefore, the client shouldn't be allowed to ambulate. Carefully titrating the
oxytocin, monitoring vital signs, including fetal well-being, and assisting with breathing
exercises are appropriate actions to include.

When caring for a client during the second stage of labor, which of the following
actions would be least appropriate?

a) Assisting the client with pushing


b) Ensuring the client's legs are positioned appropriately
c) Allowing the client clear liquids
d) Monitoring the fetal heart rate
Rationale: During this time, the client is usually offered ice chips rather than clear liquids.
Nursing care for the client during the second stage of labor should include assisting the
mother with pushing, helping position her legs for maximum pushing effectiveness, and
monitoring the fetal heart rate.

After admission to the labor and delivery area, a client undergoes routine tests,
including a complete blood count, urinalysis, Venereal Disease Research Laboratory
test, and gonorrhea culture. The gonorrhea culture is positive, although the client
lacks signs and symptoms of this disease. What is the significance of this finding?

a) Maternal gonorrhea may cause a neural tube defect in the fetus.


b) Maternal gonorrhea may cause an eye infection in the neonate.
c) Maternal gonorrhea may cause acute liver changes in the fetus.
d) Maternal gonorrhea may cause anemia in the neonate
Rationale: Gonorrhea in the cervix may cause neonatal eye infection during delivery as
well as a serious puerperal infection in the client. Maternal gonorrhea isn't associated with
neural tube defects, acute fetal liver changes, or neonatal anemia

During labor, a client asks the nurse why her blood pressure must be measured so
often. Which explanation should the nurse provide?

52
a) Blood pressure reflects changes in cardiovascular function, which may affect the
fetus.
b) Increased blood pressure indicates that the client is experiencing pain.
c) Increased blood pressure signals the acme of the contraction.
d) Medications given during labor affect blood pressure.
Rationale: Frequent blood pressure measurement helps determine whether maternal
cardiovascular function is adequate. During contractions, blood flow to the intervillous
spaces changes, compromising fetal blood supply. Increased blood pressure is expected
during pain and contractions. Measuring blood pressure frequently helps determine
whether blood pressure has returned to precontraction levels, ensuring adequate blood
flow to the fetus. Although medications given during labor can affect blood pressure, the
main purpose of measuring blood pressure is to verify adequate fetal status.

A client is in the second stage of labor. During this stage, how frequently should the
nurse assess her uterine contractions?

a) Every 5 minutes
b) Every 15 minutes
c) Every 30 minutes
d) Every 60 minutes
Rationale: During the second stage of labor, the nurse should assess the strength,
frequency, and duration of contractions every 15 minutes. If maternal or fetal problems are
detected, more frequent monitoring is necessary. An interval of 30 to 60 minutes between
assessments is too long because of variations in the length and duration of a client's labor.

After a client enters the second stage of labor, the nurse notes that her amniotic
fluid is port-wine colored. What does this finding suggest?
a) Increased bloody show
b) Normal amniotic fluid
c) Abruptio placentae
d) Meconium
Rationale: Port-wine-colored amniotic fluid isn't normal and may indicate abruptio
placentae. Increased bloody show is a normal finding and causes light pink amniotic fluid.
Meconium turns amniotic fluid green.

A multiparous client is admitted to the labor and delivery area with painless vaginal
bleeding. Ultrasonography shows that an edge of her placenta meets but doesn't
occlude the rim of the cervical os. This finding suggests:

a) Partial placenta previa.


b) A low-lying placenta.
c) Marginal placenta previa.
d) Abruptio placentae.
Rationale: Marginal placenta previa is present when the edge of the placenta meets the
rim of the cervical os but doesn't occlude it. In partial placenta previa, the placenta covers
the os partially. In a low-lying placenta, the placenta implants in the lower uterine

53
segment and a placental edge lies close to the cervical os. In abruptio placentae, the
placenta separates from the uterine wall.

a) A decreased urge to push


b) Decreased bloody show
c) Fetal heart rate (FHR) accelerations
d) Bulging of the vaginal introitus
Rationale: Signs and symptoms of transition to the second stage of labor include bulging
of the vaginal introitus, an increased urge to push, increased bloody show, and grunting.
FHR accelerations may occur at any time during labor

A client in labor is attached to an electronic fetal monitor (EFM). Which of the


following data provided by an EFM most reliably indicates adequate uteroplacental
and fetal perfusion?

a) Fetal heart rate variability within an acceptable range


b) Persistent fetal bradycardia
c) Late decelerations
d) Variable decelerations and sinusoidal pattern
Rationale: Fetal heart rate variability most reliably indicates uteroplacental and fetal
perfusion; an average variability of 6 to 10 beats per minute is considered acceptable.
Persistent fetal bradycardia may signal hypoxia, arrhythmias, or fetal cord compression.
Late decelerations indicate decreased blood flow and oxygen to the intervillous spaces
during uterine contractions — a nonreassuring pattern. Variable decelerations suggest
umbilical cord compression; a sinusoidal pattern signals severe fetal anemia or
asphyxiation.

Assessment of a client in active labor reveals meconium-stained amniotic fluid and


fetal heart sounds in the upper right quadrant. Which of the following is the most
likely cause of this situation?

a) Breech position
b) Late decelerations
c) Entrance into the second stage of labor
d) Multiple gestation
Rationale: Fetal heart sounds in the upper right quadrant and meconium-stained amniotic
fluid indicate a breech presentation. The staining is usually caused by the squeezing
actions of the uterus on a fetus in the breech position, although late decelerations, entrance
into the second stage of labor, and multiple gestation may contribute to meconium-stained
amniotic fluid

When administering magnesium sulfate to a client with preeclampsia, the nurse


understands that this drug is given to do which of the following?

a) Prevent seizures
b) Reduce blood pressure
c) Slow the process of labor
d) Increase diuresis
Rationale: The chemical makeup of magnesium is similar to that of calcium and, therefore,
magnesium will act like calcium in the body. As a result, magnesium will block seizure
54
activity in a hyperstimulated neurologic system by interfering with signal transmission at the
neural musculature junction. Reducing blood pressure, slowing labor, and increasing
diuresis are secondary effects of magnesium.

After delivering a neonate, a client delivers the placenta. At this time, where does the
nurse expect to palpate the uterine fundus?
a) At the midline, 0.4" to 0.8" (1 to 2 cm) above the umbilicus
b) At the midline, 0.4" to 0.8" (1 to 2 cm) below the umbilicus
c) Left of the midline, 0.8" to 1" (2 to 3 cm) above the umbilicus
d) Left of the midline, 0.8" to 1" (2 to 3 cm) below the umbilicus
Rationale: After delivery of the placenta, the fundus is normally firmly contracted at the
midline, 0.4" to 0.8" (1 to 2 cm) below the umbilicus.

A client with moderate pregnancy-induced hypertension (PIH) is a poor candidate for


regional anesthesia during labor and delivery. If she were to receive this form of
anesthesia, she might experience:

a) Hypotension.
b) Hypertension.
c) Seizures.
d) Renal toxicity
Rationale: In a client with PIH, uteroplacental perfusion may be inadequate and gas
exchange may be poor. Regional anesthesia increases the risk of hypotension resulting
from sympathetic blockade, possibly causing fetal and maternal hypoxia. Hypertension,
seizures, and renal toxicity aren't associated with regional anesthesia

During a childbirth education class, a nurse-educator discusses pain control


techniques used during labor and delivery. Which technique most effectively helps a
client cope with the pain of uterine contractions?
a) Controlled breathing
b) Distraction
c) Cutaneous stimulation
d) Hypnosis
Rationale: Used by the client or her coach, cutaneous stimulation, such as effleurage,
creates organized, controlled sensory input that reduces local irritability. In effleurage, the
client concentrates on the sensation of light fingertip stroking of the abdomen or back,
rather than the pain of the contraction, to help counteract the perception of pain. Controlled
breathing is primarily used to enhance relaxation; although relaxation can reduce pain, it's
typically less effective than cutaneous stimulation. Distraction isn't helpful because it diverts
the client from the task of labor. Hypnosis is a trancelike state used to reduce attention to
external stimuli.

When caring for a client who's having her second baby, the nurse can anticipate the
client's labor will be which of the following?
a) Shorter than her first labor
b) About half as long as her first labor
c) About the same length of time as her first labor
d) A length of time that can't be determined based on her first labor
Rationale: A woman having her second baby can anticipate a labor about half as long as
her first labor. The other options are incorrect

55
The nurse is administering oxytocin (Pitocin) to a client in labor. During oxytocin
therapy, why must the nurse monitor the client's fluid intake and output closely?

a) Because oxytocin causes water intoxication


b) Because oxytocin causes excessive thirst
c) Because oxytocin has a diuretic effect
d) Because oxytocin is toxic to the kidney
Rationale: Oxytocin has an antidiuretic effect; prolonged I.V. infusion may lead to severe
water intoxication, resulting in seizures, coma, and even death. Excessive thirst results
from the work of labor and lack of oral fluids, not oxytocin administration. Oxytocin isn't
toxic to the kidney.

A client is recovering in the labor and delivery area after delivering a 6-lb, 3-oz boy.
On assessment, the nurse finds that the client's fundus is firm and located two
fingerbreadths below the umbilicus. Although she didn't have an episiotomy, her
perineal pad reveals a steady trickle of blood. What is the probable cause of these
assessment findings?

a) A boggy uterus
b) Normal involution
c) A vaginal laceration
d) A clotting problem
Rationale: A steady trickle of blood on the perineal pad of a client with a well-contracted
uterus may indicate a vaginal, cervical, or perineal laceration. A boggy uterus would be
palpable above the umbilicus and would be soft and poorly contracted. With normal
involution, the perineal pad would show only lochia, not a trickle of blood. A clotting
problem causes more than a steady trickle of blood and probably would have been
identified earlier during labor.

The nurse is caring for a primigravid client in the labor and delivery area. Which
condition would place the client at risk for disseminated intravascular coagulation
(DIC)?

a) Intrauterine fetal death


b) Placenta accreta
c) Dysfunctional labor
d) Premature rupture of the membranes
Rationale: Intrauterine fetal death, abruptio placentae, septic shock, and amniotic fluid
embolism may trigger normal clotting mechanisms; if clotting factors are depleted, DIC may
occur. Placenta accreta, dysfunctional labor, and premature rupture of the membranes
aren't associated with DIC.

When reviewing a fetal monitor strip, the nurse looks for reassuring and
nonreassuring fetal heart rate (FHR) patterns. Which pattern is nonreassuring?

a) FHR that accelerates to baseline tachycardia


b) Baseline FHR that doesn't increase
c) Variable FHR pattern that begins and ends abruptly
d) Short-term variability that doesn't decrease
Rationale: In a nonreassuring pattern, the FHR accelerates to baseline tachycardia as the
fetus attempts to compensate for a growing oxygen deficit. A reassuring variable pattern
56
has an abrupt onset and end. The baseline FHR doesn't increase and short-term variability
doesn't decrease.

A client who comes to the labor and delivery area tells the nurse she believes her
membranes have ruptured. When obtaining her history, what should the nurse ask
about first?
a) The time of membrane rupture
b) The frequency of contractions
c) The presence of back pain
d) The presence of bloody show
Rationale: First, the nurse should ask the client when her membranes ruptured because
the risk of perinatal infection increases with the time elapsed between membrane rupture
and the onset of contractions. After determining the time of membrane rupture, the nurse
should ask about the frequency of contractions and find out whether the client has back
pain or bloody show

The nurse is caring for a client who's in the first stage of labor. What is the shortest
but most difficult part of this stage?
a) Active phase
b) Complete phase
c) Latent phase
d) Transitional phase
Rationale: The transitional phase, which lasts 1 to 3 hours, is the shortest but most difficult
part of the first stage of labor. This phase is characterized by intense uterine contractions
that occur every 1½ to 2 minutes and last 45 to 90 seconds. The active phase lasts 4½ to 6
hours; it's characterized by contractions that start out moderately intense, grow stronger,
and last about 60 seconds. The complete phase occurs during the second, not first, stage
of labor. The latent phase lasts 5 to 8 hours and is marked by mild, short, irregular
contractions

After several hours of using music and imagery to promote comfort, a client in labor
becomes discouraged and frustrated. What should the nurse suggest?
a) Continue to use these methods until they work.
b) Ask the support person to leave the room temporarily.
c) Use an alternate method of promoting comfort.
d) Encourage the client to get some sleep.
Rationale: If a client becomes discouraged or frustrated during labor, the nurse should
reassure her and help her choose an alternate method of promoting comfort. The other
options would increase the client's frustration

A 32-year-old multipara is admitted to the birthing room after her initial examination
reveals her cervix to be at 8 cm, completely effaced (100%), and at 0 station. What
phase of labor is she in?

a) Active phase
b) Latent phase
c) Expulsive phase
d) Transitional phase
Rationale: The transitional phase of labor extends from 8 to 10 cm; it's the shortest but
most difficult and intense for the client. The latent phase extends from 0 to 3 cm; it's mild in

57
nature. The active phase extends from 4 to 7 cm; it's moderate for the client. The expulsive
phase begins immediately after the birth and ends with separation and expulsion of the
placenta.

A client, age 19, goes into labor at 40 weeks' gestation. When assessing the fetal
monitor strip, the nurse sees that the fetal heart rate (FHR) has decreased to 60
beats/minute and that the waveforms sometimes resemble a U and begin and end
abruptly. The nurse should interpret this pattern as:

a) Variable decelerations.
b) Decreased short-term variability.
c) Increased long-term variability.
d) Early decelerations.
Rationale: On a fetal monitor strip, variable decelerations are characterized by an FHR
that commonly decreases to 60 beats/minute; waveform shapes that vary and may
resemble the letter U, V, or W; and deceleration waveforms with an abrupt onset and
recovery. Decreased short-term variability manifests as fewer than 2 to 3 beats/amplitude
of the baseline FHR. Increased long-term variability manifests as more than 5 to 20
beats/minute of the baseline FHR in rhythmic fluctuation. Early decelerations are seen as
the descent, peak, and recovery of the deceleration waveform that mirrors the contraction
waveform.

A client is admitted to the labor and delivery department in preterm labor. To help manage
preterm labor the nurse would expect to administer:
a) ritodrine (Yutopar).
b) bromocriptine (Parlodel).
c) magnesium sulfate.
d) betamethasone (Celestone).
Rationale: Ritodrine reduces frequency and intensity of uterine contractions by stimulating
B2 receptors in the uterine smooth muscle. It's the drug of choice when trying to inhibit
labor. Bromocriptine, a dopamine receptor agonist and an ovulation stimulant, is used to
inhibit lactation in the postpartum period. Magnesium sulfate, an anticonvulsant, is used to
treat preeclampsia and eclampsia — a life-threatening form of pregnancy-induced
hypertension. Betamethasone, a synthetic corticosteroid, is used to stimulate fetal
pulmonary surfactant (administered to the mother).

When assessing a pregnant client with a history of cardiac dysfunction, the nurse
discovers that the client has been taking propranolol (Inderal), a beta-adrenergic
blocker, to treat hypertension. During labor, the nurse should stay alert for which
adverse effect of this drug?

a) Uterine hypotonus
b) Uterine hypertonus
c) Hypotension
d) Tachyarrhythmias
Rationale: Propranolol, used to treat hypertension and tachyarrhythmias, may cause
constant uterine hypertonus, resulting in preterm labor. It's unlikely that the drug would
cause hypotension during labor, and it doesn't cause hypertension and tachyarrhythmias.

58
To promote comfort during labor, the nurse advises a client to assume certain
positions and avoid others. Which position may cause maternal hypotension and
fetal hypoxia?
a) Lateral position
b) Squatting position
c) Supine position
d) Standing position
Rationale: The supine position causes compression of the client's aorta and inferior vena
cava by the fetus. This, in turn, inhibits maternal circulation, leading to maternal
hypotension and, ultimately, fetal hypoxia. The other positions promote comfort and aid
labor progress. For instance, the lateral, or side-lying, position improves maternal and fetal
circulation, enhances comfort, increases maternal relaxation, reduces muscle tension, and
eliminates pressure points. The squatting position promotes comfort by taking advantage of
gravity. The standing position also takes advantage of gravity and aligns the fetus with the
pelvic angle.

At 28 weeks' gestation, a client is admitted to the labor and delivery area in preterm
labor. An I.V. infusion of ritodrine (Yutopar) is started. Which client outcome reflects
the nurse's awareness of an adverse effect of ritodrine?

a) The client remains free from tachycardia."


b) "The client remains free from polyuria."
c) "The client remains free from hypertension."
d) "The client remains free from hyporeflexia."
Rationale: Ritodrine and other beta-adrenergic agonists may cause tachycardia,
hypotension, bronchial dilation, increased plasma volume, increased cardiac output,
arrhythmias, myocardial ischemia, reduced urine output, restlessness, headache, nausea,
and vomiting. These drugs aren't associated with polyuria, hypertension, or hyporeflexia.

A client in labor receives epidural anesthesia. The nurse should assess carefully for which adverse
reaction to the anesthetic agent?
a) Hypotensive crisis
b) Fetal tachycardia
c) Renal toxicity
d) Increased beat-to-beat variability in the fetal heart rate (FHR)
Rationale: Hypotensive crisis may occur after epidural anesthesia administration as the
anesthetic agent spreads through the spinal canal, blocking sympathetic innervation. Other
signs and symptoms of hypotensive crisis associated with epidural anesthesia may include
fetal bradycardia (not tachycardia) and decreased (not increased) beat-to-beat variability in
the FHR. Urine retention, not renal toxicity, may occur during the postpartum period.

When magnesium sulfate is administered to a client in labor, its action occurs at


which of the following sites?

a) Neural-muscular junctions
b) Distal renal tubules
c) Central nervous system (CNS)
d) Myocardial fibers
Rationale: Because magnesium has chemical properties similar to those of calcium, it will
assume the role of calcium at the neural muscular junction. It doesn't act on the distal renal
tubules, CNS, or myocardial fibers.

59
For a client who's fully dilated, which of the following actions would be inappropriate
during the second stage of labor?

a) Positioning the mother for effective pushing


b) Preparing for delivery of the baby
c) Assessing vital signs every 15 minutes
d) Assessing for rupture of membranes
Rationale: In most cases, the membranes have ruptured (spontaneously or artificially) by
this stage of labor. Positioning for effective pushing, preparing for delivery, and assessing
vital signs every 15 minutes are appropriate actions at this time.

A primigravid client is admitted to the labor and delivery area, where the nurse
evaluates her. Which assessment finding may indicate the need for cesarean
delivery?
a) Insufficient perineal stretching
b) Rapid, progressive labor
c) Umbilical cord prolapse
d) Fetal prematurity
Rationale: Indications for cesarean delivery include umbilical cord prolapse, breech
presentation, fetal distress, dystocia, previous cesarean delivery, herpes simplex infection,
condyloma acuminatum, placenta previa, abruptio placentae, and unsuccessful labor
induction. Insufficient perineal stretching; rapid, progressive labor; and fetal prematurity
aren't indications for cesarean delivery.

A client with Rh isoimmunization delivers a neonate with an enlarged heart and


severe, generalized edema. Which nursing diagnosis is most appropriate for this
client?

a) Ineffective denial related to a socially unacceptable infection


b) Deficient parenting related to the neonate's transfer to the intensive care unit
c) Deficient fluid volume related to severe edema
d) Fear related to removal and loss of the neonate by statute
Rationale: Because the neonate is severely ill and needs to be placed in the neonatal
intensive care unit, the client may have a nursing diagnosis of Deficient parenting related to
the neonate's transfer to the neonatal intensive care unit. (Another pertinent nursing
diagnosis may be Ineffective family coping: Compromised related to lack of opportunity for
bonding.) Rh isoimmunization isn't a socially unacceptable infection. This condition causes
a fluid volume excess (not deficit) related to cardiac problems. Rh isoimmunization doesn't
lead to loss of the neonate by statute.

The nurse is caring for a client with mild active bleeding from placenta previa. Which
assessment factor indicates that an emergency cesarean section may be necessary?

a) Increased maternal blood pressure of 150/90 mm Hg


b) Decreased amount of vaginal bleeding
c) Fetal heart rate of 80 beats/minute
d) Maternal heart rate of 65 beats/minute
Rationale: A drop in fetal heart rate signals fetal distress and may indicate the need for a
cesarean section to prevent neonatal death. Maternal blood pressure, pulse rate,
respiratory rate, intake and output, and description of vaginal bleeding are all important

60
assessment factors; however, changes in these factors don't always necessitate the
delivery of the neonate.

A client is admitted to the labor and delivery area. The nurse-midwife checks for fetal
descent, flexion, internal rotation, extension, external rotation, and expulsion. What
do these terms describe?

a) Phases of the first stage of labor


b) Cardinal movements of labor
c) Factors affecting labor
d) Factors that determine fetal position
Rationale: Cardinal movements of labor refer to the typical sequence of positions assumed
by the fetus during labor and delivery. These positions are most commonly called descent,
flexion, internal rotation, extension, external rotation, and expulsion. Phases of the first
stage of labor include the latent, active, and transitional phases. Factors affecting labor
include the passenger, passageway, powers, placental position and function, and
psychological response. Factors that determine fetal position include the landmark of the
fetal presenting part, whether the landmark faces the left or right side of the maternal
pelvis, and whether the landmark faces the front, back, or side of the maternal pelvis.

For a client who's moving into the active phase of labor, the nurse should include
which of the following as the priority of care?

a) Offer support by reviewing the short-pant form of breathing.


b) Administer narcotic analgesia.
c) Allow the mother to walk around the unit.
d) Watch for rupture of the membranes.
Rationale: By helping the client use the pant form of breathing, the nurse can help the
client manage her contractions and reduce the need for narcotics and other forms of pain
relief, which can have an effect on fetal outcome. In the active phase, the mother most
likely is too uncomfortable to walk around the unit. The nurse will observe for rupture of
membranes and may administer narcotic analgesia but these don't take priority.

A client in labor tells the nurse-midwife that she feels a strong urge to push.
Physical examination reveals that her cervix is not completely dilated. The nurse-
midwife tells her not to push yet. What is the rationale for this instruction?

a) Early pushing may cause edema and impede fetal descent.


b) The nurse-midwife isn't ready to assist her.
c) The fetus hasn't rotated into the proper position.
d) Pushing at this time may cause rupture of the membranes
Rationale: Pushing (bearing down) before the cervix is completely dilated may cause
edema and tissue damage and may impede fetal descent. Telling the client not to push
because the nurse-midwife isn't ready to assist is inappropriate and unprofessional. If the
cervix were completely dilated, the nurse-midwife could assist the client in changing
position to help reposition the fetus. The client's membranes should have ruptured already.

During labor, a client's cervix fails to dilate progressively, despite her uncomfortable
uterine contractions. To augment labor, the physician orders oxytocin (Pitocin).
When preparing the client for oxytocin administration, the nurse describes the

61
contractions the client is likely to feel when she starts to receive the drug. Which
description is accurate?
a) Contractions will be stronger and more uncomfortable and will peak more
abruptly.
b) Contractions will be weaker, longer, and more effective.
c) Contractions will be stronger, shorter, and less uncomfortable.
d) Contractions will be stronger and shorter and will peak more slowly.
Rationale: Oxytocin administration causes stronger, more uncomfortable contractions,
which peak more abruptly than spontaneous contractions. Oxytocin doesn't affect the
duration of contractions

The nurse is assessing a woman in labor. Her cervix is dilated 8 cm. Her
contractions are occurring every 2 minutes. She's irritable and in considerable pain.
What type of breathing should the nurse instruct the woman to use during the peak
of a contraction?
a) Deep breathing
b) Shallow chest breathing
c) Deep, cleansing breaths
d) Chest panting
Rationale: Shallow chest breathing is used during the peak of a contraction during the
transitional phase of labor. Deep breathing can cause a woman to hyperventilate and feel
light-headed, with numbness or tingling in her fingers or toes. A deep, cleansing breath
taken at the beginning and end of each breathing exercise can help prevent
hyperventilation. Chest panting may be used to prevent a woman from pushing before the
cervix is fully dilated.

A client with active genital herpes is admitted to the labor and delivery area during
the first stage of labor. Which type of birth should the nurse anticipate for this
client?

a) Mid forceps
b) Low forceps
c) Induction
d) Cesarean
Rationale: For a client with active genital herpes, cesarean birth helps avoid infection
transmission to the neonate, which would occur during a vaginal birth. Mid forceps and low
forceps are types of vaginal births that could transmit the herpes infection to the neonate.
Induction is used only during vaginal birth; therefore, it's inappropriate for this client.

An assisted birth using forceps or a vacuum extractor may be performed for


ineffective pushing, for large infants, to shorten the second stage of labor, or for a
malpresentation. The nurse caring for the mother following an assisted birth should
keep which of the following in mind?

a) A vacuum extractor is safer than forceps because it causes less trauma to the
baby and the mother's perineum.
b) The baby will develop a cephalohematoma as a result of the instrumentation.
c) The use of instruments during the birth process is a fairly rare occurrence.
d) Additional nursing interventions are needed to ensure an uncomplicated
postpartum.
Rationale: When used properly, a vacuum extractor is a safer delivery with fewer
complications for the mother and the baby than a forceps delivery. Cephalohematomas
62
occur more often in assisted births than in unassisted births. Instruments are used during
delivery when individually necessary. No additional nursing interventions are needed during
the postpartum period.

A client is admitted to the maternity area in active labor. Her cervix is dilated 4 cm.
The physician prescribes etidocaine (Duranest), 150 mg via epidural catheter. What
might account for the physician's choice of etidocaine over other local anesthetic
agents?
a) It produces no vasoconstrictor effects.
b) It's least likely to cross the placenta.
c) It has the fastest onset of action.
d) It's least likely to cause cardiac arrhythmias
Rationale: Etidocaine is least likely to cross the placenta. Local anesthetics, such as
etidocaine, don't cause vasoconstriction. If vasoconstriction is needed, the local anesthetic
must be combined with a drug such as epinephrine. Although etidocaine has an onset of
action of 2 to 8 minutes, prilocaine's onset of action is less than 2 minutes. Local
anesthetics, including etidocaine, can cause adverse cardiac arrhythmias if high doses are
given.

A client in labor for the past 10 hours shows no change in cervical dilation and has
stayed at 5 to 6 cm for the past 2 hours. Her contractions remain regular at 2-minute
intervals, lasting 40 to 45 seconds. Which of the following would be the nurse's
initial action?
a) Assess for presence of a full bladder.
b) Suggest the placement of an internal uterine pressure catheter to determine
adequacy of contractions.
c) Encourage the mother to relax by assisting her with appropriate breathing
techniques.
d) Suggest to the physician that oxytocin augmentation be started to stimulate
labor.
Rationale: A full bladder will slow or stop cervical dilation and produce symptoms that
could be misdiagnosed as arrest in labor. Other strategies, such as internal uterine
monitoring, relaxation, and oxytocin augmentation, would be appropriate later, but
assessing the bladder first is key.

A client is to have an epidural block to relieve labor pain. The nurse anticipates that
the anesthesiologist will inject the anesthetic agent into the:

a) subarachnoid space.
b) area between the subarachnoid space and the dura mater.
c) area between the dura mater and the ligamentum flavum.
d) ligamentum flavum.
Rationale: For an epidural block, the nurse should anticipate that the anesthesiologist will
inject a local anesthetic agent into the epidural space, located between the dura mater and
the ligamentum flavum in the lumbar region of the spinal column. When administering a
spinal block, the anesthesiologist injects the anesthetic agent into the subarachnoid space.
The ligamentum flavum and the area between the subarachnoid space and the dura mater
are inappropriate injection sites.

63
The nurse is assessing a client who has been admitted to the labor and delivery
area. Which technique does the nurse use to determine fetal position and
presentation?

a) Abdominal ultrasonography
b) Fetal heart tone auscultation
c) Palpation of contractions
d) Leopold's maneuvers
Rationale: Leopold's maneuvers, a series of abdominal palpations, are used to determine
fetal position, presentation, and lie. Abdominal ultrasonography is used throughout
pregnancy to determine the positions of the uterus and cervix, detect an abnormal fetus,
determine the number of fetuses, take fetal measurements, and estimate gestational age.
Fetal heart tone auscultation is done to assess the fetal heart rate. Palpation of
contractions helps to distinguish true labor contractions from Braxton Hicks contractions.

A client in labor is receiving oxytocin (Pitocin). The electronic fetal monitoring strip
shows contractions occurring every 30 seconds to 2 minutes, with an intensity of 90
mm Hg and increasing resting tone. How should the nurse respond to these
findings?

a) Administer oxygen as prescribed.


b) Call the physician.
c) Check the fetal heart rate (FHR).
d) Discontinue the oxytocin infusion.
Rationale: Oxytocin should be discontinued when contractions occur less than 2 minutes
apart or last longer than 90 seconds. The nurse can stop oxytocin infusion independently
without seeking permission from the physician — an action that would waste valuable time.
This client isn't oxygen deprived and, therefore, doesn't need supplemental oxygen.
Checking the FHR isn't appropriate in this situation because the decelerations occur and
resolve with each contraction, independent of oxytocin administration.

The nurse applies an external electronic fetal monitor (EFM) to assess a client's
uterine contractions and evaluate the fetal heart rate (FHR). However, the client is
uncomfortable and changes positions frequently, making FHR hard to assess.
Consequently, the physician decides to switch to an internal EFM. Before internal
monitoring can begin, which of the following must occur?

a) The membranes must rupture.


b) The client must receive anesthesia.
c) The cervix must be fully dilated.
d) The fetus must be at 0 station.
Rationale: Internal EFM can be used only after the client's membranes rupture, when the
cervix is dilated at least 2 cm and when the presenting part is at least at –1 station.
Anesthesia isn't required for internal EFM.

Which of the following should be the nurse's initial action immediately following the
birth of the baby?

a) Aspirating mucus from the infant's nose and mouth


b) Drying the infant to stabilize the infant's temperature
c) Promoting parental bonding
d) Identifying the newborn

64
Rationale: The nurse's first action is to dry the baby and stabilize the infant's temperature.
Aspiration of the infant's nose and mouth occurs at the time of delivery. Promoting parental
bonding and identifying the newborn are appropriate after the baby has been dried.

During the active phase of the first stage of labor, a client undergoes an amniotomy.
After this procedure, which nursing diagnosis takes the highest priority?
a) Deficient knowledge related to amniotomy
b) Ineffective fetal cerebral tissue perfusion related to cord compression
c) Pain related to increasing strength of contractions
d) Risk for infection related to rupture of membranes
Rationale: Amniotomy increases the risk of cord prolapse. If the prolapsed cord is
compressed by the presenting fetal part, the fetal blood supply may be impaired,
jeopardizing the fetal oxygen supply. Because lack of oxygen to the fetus may cause fetal
death, the nursing diagnosis of Ineffective fetal cerebral tissue perfusion takes priority over
diagnoses of Deficient knowledge, Pain, and Risk for infection.

When assessing a client 1 hour after vaginal delivery, the nurse notes blood gushing
from the vagina, pallor, and a rapid, thready pulse. What do these findings suggest?

a) Uterine involution
b) Cervical laceration
c) Placental separation
d) Postpartum hemorrhage
Rationale: Postpartum hemorrhage results in excessive vaginal bleeding and signs of
shock, such as pallor and a rapid, thready pulse. Placental separation causes a sudden
gush or trickle of blood from the vagina, rise of the fundus in the abdomen, increased
umbilical cord length at the introitus, and a globe-shaped uterus. Uterine involution causes
a firmly contracted uterus, which can't occur until the placenta is delivered. Cervical
lacerations produce a steady flow of bright red blood in a client with a firmly contracted
uterus.

Several minutes after a vaginal delivery, nursing assessment reveals blood gushing
from the client's vagina, umbilical cord lengthening, and a globular-shaped uterus.
The nurse should suspect which condition?

a) Cervical or vaginal laceration


b) Placental separation
c) Postpartum hemorrhage
d) Uterine involution
Rationale: Placental separation is characterized by a sudden gush or trickle of blood from
the vagina, further protrusion of the umbilical cord from the vagina, a globular-shaped
uterus, and an increase in fundal height. With cervical or vaginal laceration, the nurse notes
a consistent flow of bright red blood from the vagina. With postpartum hemorrhage, usually
caused by uterine atony, the uterus isn't globular. Uterine involution can't begin until the
placenta has been delivered.

The physician decides to artificially rupture the membranes. Following this


procedure, the nurse checks the fetal heart tones for which the following reasons?

a) To determine fetal well-being


b) To assess for prolapsed cord
c) To assess fetal position
65
d) To prepare for an imminent delivery
Rationale: After a client has an amniotomy, the nurse should assure that the cord isn't
prolapsed and that the baby tolerated the procedure well. The most effective way to do this
is to check the fetal heart rate. Fetal well-being is assessed via a nonstress test. Fetal
position is determined by vaginal examination. Artificial rupture of membranes doesn't
indicate an imminent delivery.

A client is admitted to the labor and delivery area. How can the nurse most
effectively determine the duration of the client's contractions?

a) By timing the period between one contraction and the beginning of the next
contraction
b) By timing the period from the onset of uterine tightening to uterine relaxation
c) By timing the period from the increment (building-up) phase to the acme (peak)
phase
d) By timing the period from the acme (peak) phase to the decrement (letting-down)
phase
Rationale: To determine the duration of contractions, the nurse should time the period
from the onset of uterine tightening to uterine relaxation. Timing the period between one
contraction and the beginning of the next contraction helps determine the frequency of
contractions. Timing the period from the increment to the acme or from the acme to the
decrement supplies only partial information about contractions.

When caring for a client who's a primigravida, the nurse would expect that the
second stage would normally last how long?
a) Approximately 2 hours
b) Less than 1 hour
c) 4 hours
d) 3 hours
Rationale: The average length of time a primigravida needs to push is approximately 2
hours. Longer than that might mean the client is experiencing an arrest in descent. Few
primigravidas have a second stage of labor shorter than 1 hour

During labor, a primigravid client receives epidural anesthesia, and the nurse assists
in monitoring maternal and fetal status. Which finding suggests an adverse reaction
to the anesthesia?
a) Increased variability
b) Maternal hypotension
c) Fetal tachycardia
d) Anuria
Rationale: As the epidural anesthetic agent spreads through the spinal canal, it may
produce hypotensive crisis, which is characterized by maternal hypotension, decreased
variability, and fetal bradycardia. Although the client may experience some postpartum
urine retention, anuria isn't associated with epidural anesthesia

Because cervical effacement and dilation aren't progressing in a client in labor, the
physician orders I.V. administration of oxytocin (Pitocin). Why must the nurse
monitor the client's fluid intake and output closely during oxytocin administration?
a) Oxytocin causes water intoxication.
b) Oxytocin causes excessive thirst.

66
c) Oxytocin is toxic to the kidneys.
d) Oxytocin has a diuretic effect.
Rationale: The nurse should monitor fluid intake and output because prolonged oxytocin
infusion may cause severe water intoxication, leading to seizures, coma, and death.
Excessive thirst results from the work of labor and limited oral fluid intake — not oxytocin.
Oxytocin has no nephrotoxic or diuretic effects. In fact, it produces an antidiuretic effect.

In the first stage of labor, a client with a full-term pregnancy has an electronic fetal
monitoring (EFM) device in place. Which EFM pattern suggests adequate
uteroplacental-fetal perfusion?
a) Persistent fetal bradycardia
b) Variable decelerations
c) Average variability
d) Late decelerations
Rationale: A fetal heart rate with average variability (6 to 10 beats/minute) accurately
predicts adequate uteroplacental-fetal perfusion. Persistent fetal bradycardia may indicate
hypoxia, arrhythmia, or umbilical cord compression. Variable decelerations also suggest
umbilical cord compression. Late decelerations may reflect decreased blood flow and
oxygen to the intervillous spaces during contractions.

A woman in labor shouts to the nurse, "My baby is coming right now! I feel like I
have to push!" An immediate nursing assessment reveals that the head of the fetus
is crowning. After asking another staff member to notify the physician and setting up
for delivery, which nursing intervention is most appropriate?
a) Gently pulling at the baby's head as it's delivered
b) Holding the baby's head back until the physician arrives
c) Applying gentle pressure to the baby's head as it's delivered
d) Placing the mother in the Trendelenburg position until the physician arrives
Rationale: Gentle pressure applied to the baby's head as it's delivered prevents rapid
expulsion, which can cause brain damage to the baby and perineal tearing in the mother.
Never pull at the baby's head or hold the head back. Placing the mother in the
Trendelenburg position won't halt labor and may cause respiratory difficulties.

A diabetic client in labor tells the nurse she has had trouble controlling her blood
glucose level recently. She says she didn't take her insulin when the contractions
began because she felt nauseated; about an hour later, when she felt better, she ate
some soup and crackers but didn't take insulin. Now, she reports increased nausea
and a flushed feeling. The nurse notes a fruity odor to her breath. What do these
findings suggest?
a) Diabetic ketoacidosis
b) Hypoglycemia
c) Infection
d) Transition to the active phase of labor
Rationale: Signs and symptoms of diabetic ketoacidosis include nausea and vomiting, a
fruity or acetone breath odor, signs of dehydration (such as flushed, dry skin),
hyperglycemia, ketonuria, hypotension, deep and rapid respirations, and a decreased level
of consciousness. In contrast, hypoglycemia causes sweating, tremors, palpitations, and
behavioral changes. Infection causes a fever. Transition to the active phase of labor is
signaled by cervical dilation of up to 7 cm and contractions every 2 to 5 minutes.

67
Late in the first stage of labor, a client receives a spinal block to relieve discomfort.
A short time later, her husband tells the nurse that his wife feels dizzy and is
complaining of numbness around her lips. What do the client's symptoms suggest?

a) Anesthesia overdose
b) Transition to the second stage of labor
c) Anxiety
d) Dehydration
Rationale: Dizziness, circumoral numbness, and slurred speech indicate anesthesia
overdose. Transition to the second stage of labor is marked by an increased urge to push,
an increase in bloody show, grunting, gaping of the anus, involuntary defecation, thrashing
about, loss of control over breathing techniques, and nausea and vomiting. Anxiety and
dehydration rarely cause dizziness or circumoral numbness.

An expected fetal adverse reaction to meperidine (Demerol) during labor is:


a) Decreased beat-to-beat variability.
b) Bradycardia.
c) Late decelerations.
d) None known.
Rationale: Possible fetal adverse reactions include both moderate central nervous system
depression and decreased beat-to-beat variability. Bradycardia and late decelerations don't
occur as a result of meperidine administration.

A client is progressing through the first stage of labor. Which finding signals the
beginning of the second stage of labor?
a) Passage of the mucus plug
b) Bearing-down reflex
c) Change in uterine shape
d) Gush of dark blood
Rationale: The second stage of labor is heralded by a bearing-down reflex with each
contraction, increased bloody show, severe rectal pressure, and rupture of the membranes
(if this hasn't already occurred). Passage of the mucus plug typically occurs during the
latent phase of the first stage of labor. A change in uterine shape and a gush of dark blood
occur during the placental separation phase of the third stage of labor.

Which of the following is the most serious adverse effect associated with oxytocin
(Pitocin) administration during labor?
a) Tetanic contractions
b) Elevated blood pressure
c) Early decelerations of fetal heart rate
d) Water intoxication
Rationale: Tetanic contractions are the most serious adverse effect associated with
administering oxytocin. When tetanic contractions occur, the fetus is at high risk for hypoxia
and the mother is at risk for uterine rupture. The client may be at risk for pulmonary edema
if large amounts of oxytocin have been administered, and this drug can also increase blood
pressure. However, pulmonary edema and increased blood pressure aren't the most
serious adverse effects. Early decelerations of fetal heart rate aren't associated with
oxytocin administration.

68
When assessing the fetal heart rate tracing, the nurse assesses the fetal heart rate at
170 beats/minute. This rate is considered fetal tachycardia if which of the following
occurs?
a) The fetal heart rate remains at greater than 160 beats/minute for 5 minutes.
b) The fetal heart rate remains at greater than 160 beats/minute for 10 minutes.
c) The fetal heart rate remains at greater than 160 beats/minute for more than 20
minutes.
d) The fetal heart rate is at least 170 beats/minute at any time.
Rationale: The normal parameter for the fetal heart rate is 120 to 160 beats/minute.
Tachycardia is defined as a fetal heart rate greater than 160 beats/minute for more than 10
minutes. This definition takes into account the difference between tachycardia and
acceleration.

The nurse is caring for a client in labor. The external fetal monitor shows a pattern of
variable decelerations in fetal heart rate. What should the nurse do first?
a) Change the client's position.
b) Prepare for emergency cesarean section.
c) Check for placenta previa.
d) Administer oxygen.
Rationale: Variable decelerations in fetal heart rate are an ominous sign, indicating
compression of the umbilical cord. Changing the client's position from supine to side-lying
may immediately correct the problem. An emergency cesarean section is necessary only if
other measures, such as changing position and amnioinfusion with sterile saline, prove
unsuccessful. Administering oxygen may be helpful, but the priority is to change the
woman's position and relieve cord compression.

A primigravid client is admitted to the labor and delivery area. Assessment reveals
fetal malpresentation, yellow amniotic fluid, and a fetal heart rate (FHR) of 98
beats/minute. What should the nurse do?
a) Increase the I.V. oxytocin flow rate, as ordered, to hasten labor and delivery.
b) Reassess the client for continued normal findings in 15 minutes.
c) Help the client into the lithotomy position for delivery.
d) Notify the physician and surgical team of an emergency.
Rationale: Because the abnormal FHR and amniotic fluid color suggest fetal distress, the
nurse should notify the physician and surgical team. The other options describe actions
that wouldn't address this emergency situation.

During the fourth stage of labor, the nurse notes that the client's fundus is boggy
and located above the umbilicus. How should the nurse intervene?
a) Let the condition resolve spontaneously.
b) Massage the client's fundus.
c) Instruct the client to bear down.
d) Notify the physician or nurse-midwife.
Rationale: A boggy (soft and poorly contracted) fundus signals uterine atony. To correct
this condition, the nurse should massage the fundus until it becomes firm and clots are
expressed. Allowing a boggy fundus to persist would place the client at high risk for
postpartum hemorrhage. Bearing down doesn't affect uterine involution. The nurse should
notify the physician or nurse-midwife only if the client's fundus doesn't respond to massage.

69
A primigravid client, age 20, has just completed a difficult, forceps-assisted delivery
of twins. Her labor was unusually long and required oxytocin (Pitocin) augmentation.
The nurse who's caring for her should stay alert for:
a) Uterine inversion.
b) Uterine atony.
c) Uterine involution.
d) Uterine discomfort.
Rationale: Multiple fetuses, extended labor stimulation with oxytocin, and traumatic
delivery commonly are associated with uterine atony, which may lead to postpartum
hemorrhage. Uterine inversion may precede or follow delivery and commonly results from
apparent excessive traction on the umbilical cord and attempts to deliver the placenta
manually. Uterine involution and some uterine discomfort are normal after delivery.

Accompanied by her husband, a client seeks admission to the labor and delivery
area. She states that she's in labor and says she attended the facility clinic for
prenatal care. Which question should the nurse ask her first?
a) "Do you have any chronic illnesses?"
b) "Do you have any allergies?"
c) "What is your expected due date?"
d) "Who will be with you during labor?"
Rationale: When obtaining the history of a client who may be in labor, the nurse's highest
priority is to determine her current status, particularly her due date, gravidity, and parity.
Gravidity and parity affect the duration of labor and the potential for labor complications.
Later, the nurse should ask about chronic illnesses, allergies, and support persons.

The nurse-midwife determines that a client is in the second stage of labor and may
start pushing. What marks the beginning of the second stage, and what marks the
end?
a) Cervical dilation of 7 to 8 cm; complete cervical dilation
b) Complete cervical dilation; delivery of the neonate
c) Cervical dilation of 7 to 8 cm; delivery of the placenta
d) Complete cervical dilation; delivery of the placenta
Rationale: The second stage of labor begins with complete cervical dilation and ends with
delivery of the neonate.

During labor, a client tells the nurse that her last baby "came out really fast." The
nurse can help control a precipitous delivery by:
a) Applying counterpressure to the fetus's head.
b) Encouraging the client to push.
c) Massaging and supporting the perineum.
d) Instructing the client to contract the perineal muscles.
Rationale: The nurse can help control a precipitous delivery by stretching the labia, such
as by massaging and bracing the perineum with gentle back pressure. This helps prevent
perineal lacerations — the primary maternal complication of precipitous delivery. Applying
counterpressure to the fetus's head reduces perineal stress temporarily; however, delivery
proceeds when the client pushes with uterine contractions. Pushing puts further stress on
the perineum, promoting delivery. When the fetus's head exerts pressure on the perineum,
contracting the perineal muscles is virtually impossible.

70
When caring for a client in the first stage of labor, the nurse documents cervical
dilation of 9 cm and intense contractions lasting 45 to 60 seconds and occurring
about every 2 minutes. Based on these findings, the nurse should recognize that the
client is in which phase of labor?
a) Active phase
b) Latent phase
c) Descent phase
d) Transitional phase
Rationale: In the transitional phase, the cervix dilates from 8 to 10 cm, and intense
contractions occur every 1½ to 2 minutes and last for 45 to 90 seconds. In the active
phase, the cervix dilates from 5 to 7 cm, and moderate contractions progress to strong
contractions that last 60 seconds. In the latent phase, the cervix dilates 3 to 4 cm, and
contractions are short, irregular, and mild. No descent phase exists. (Fetal descent may
begin several weeks before labor but usually doesn't occur until the second stage of labor.)

The nurse observes a late deceleration. It's characterized by and indicates which of
the following?

a) U-shaped deceleration occurring after the first half of the contraction, indicating
uteroplacental insufficiency
b) U-shaped deceleration occurring with the contraction, indicating cord
compression
c) V-shaped deceleration occurring after the contraction, indicating uteroplacental
insufficiency
d) Deep U-shaped deceleration occurring before the contraction, indicating head
compression
Rationale: A late deceleration is U-shaped and occurs after the first half of the contraction,
indicating uteroplacental insufficiency. It's an ominous pattern and requires immediate
action — such as administering oxygen, repositioning the mother, and increasing the I.V.
infusion rate — to correct the problem. U- and V-shaped decelerations are variable
decelerations occurring at unpredictable times during contractions and are related to
umbilical cord compression. Deep U-shaped deceleration occurring before the contraction
is early deceleration.

For a client in active labor, the nurse-midwife plans to use an internal electronic fetal
monitoring (EFM) device. What must occur before the internal EFM can be applied?
a) The membranes must rupture.
b) The fetus must be at 0 station.
c) The cervix must be dilated fully.
d) The client must receive anesthesia.
Rationale: Internal EFM can be applied only after the client's membranes have ruptured,
when the fetus is at least at the –1 station, and when the cervix is dilated at least 2 cm.
Although the client may receive anesthesia, it isn't required before application of an internal
EFM device.

To obtain a good monitor tracing on a client in labor, the mother lies on her back.
Suddenly, she complains of feeling light-headed and becomes diaphoretic. Which of
the following should be the nurse's first action?
a) Reposition the client to her left side.
b) Immediately take the client's blood pressure and summon the physician.
c) Start oxygen at 6 L via nasal cannula.
d) Increase the I.V. fluids to correct the client's dehydration.
71
Rationale: This client is hypotensive because of decreased blood flow through the aorta.
By turning the client to her left side, the nurse removes the weight of the uterus from the
aorta and increases the maternal blood flow. Taking blood pressure, summoning the
physician, starting oxygen, and increasing I.V. fluids aren't necessary unless repositioning
doesn't relieve the symptoms.

When a client states that her "water broke," which of the following actions would be
inappropriate for the nurse to do?
a) Observing for pooling of straw-colored fluid
b) Checking vaginal discharge with nitrazine paper
c) Conducting a bedside ultrasound for an amniotic fluid index
d) Observing for flakes of vernix in the vaginal discharge
Rationale: It isn't within a nurse's scope of practice to perform and interpret a bedside
ultrasound under these conditions and without specialized training. Observing for pooling of
straw-colored fluid, checking vaginal discharge with nitrazine paper, and observing for
flakes of vernix are appropriate assessments for determining whether a client has ruptured
membranes.

At 39 weeks' gestation, a pregnant client is admitted to the labor and delivery area in
active labor. During the admission interview, she reports that her membranes
haven't ruptured. Her history reveals that this is her third pregnancy, she previously
experienced a stillbirth at 38 weeks' gestation, and she has one child at home. Which
of these findings indicates the need for electronic fetal monitoring (EFM)?
a) Third pregnancy
b) Intact membranes
c) 39 weeks' gestation
d) Previous stillbirth
Rationale: Previous stillbirth is one of several maternal factors that indicate the need for
EFM. Other indications for EFM include certain fetal factors, such as meconium staining;
certain pregnancy factors, such as amnionitis; and certain uterine factors, such as regional
anesthesia. A third pregnancy, intact membranes, and 39 weeks' gestation don't place the
client at risk for problems that require close fetal monitoring.

Which of the following would be an inappropriate indication of placental


detachment?

a) An abrupt lengthening of the cord


b) An increase in the number of contractions
c) Relaxation of the uterus
d) Increased vaginal bleeding
Rationale: Relaxation isn't an indication for detachment of the placenta. An abrupt
lengthening of the cord, an increase in the number of contractions, and an increase in
vaginal bleeding are all indications that the placenta has detached from the wall of the
uterus.

1. When assessing the adequacy of sperm for conception to occur, which of the following is
the most useful criterion?

A. Sperm count
B. Sperm motility
72
C. Sperm maturity
D. Semen volume
2. A couple who wants to conceive but has been unsuccessful during the last 2 years has
undergone many diagnostic procedures. When discussing the situation with the nurse, one
partner states, “We know several friends in our age group and all of them have their own
child already, Why can’t we have one?”. Which of the following would be the most pertinent
nursing diagnosis for this couple?

A. Fear related to the unknown


B. Pain related to numerous procedures.
C. Ineffective family coping related to infertility.
D. Self-esteem disturbance related to infertility.
3. Which of the following urinary symptoms does the pregnant woman most frequently
experience during the first trimester?

A. Dysuria
B. Frequency
C. Incontinence
D. Burning
4. Heartburn and flatulence, common in the second trimester, are most likely the result of
which of the following?

A. Increased plasma HCG levels


B. Decreased intestinal motility
C. Decreased gastric acidity
D. Elevated estrogen levels
5. On which of the following areas would the nurse expect to observe chloasma?

A. Breast, areola, and nipples


B. Chest, neck, arms, and legs
C. Abdomen, breast, and thighs
D. Cheeks, forehead, and nose
6. A pregnant client states that she “waddles” when she walks. The nurse’s explanation is
based on which of the following as the cause?

A. The large size of the newborn


B. Pressure on the pelvic muscles
C. Relaxation of the pelvic joints
D. Excessive weight gain
7. Which of the following represents the average amount of weight gained during
pregnancy?

A. 12 to 22 lb
B. 15 to 25 lb
C. 24 to 30 lb
D. 25 to 40 lb
8. When talking with a pregnant client who is experiencing aching swollen, leg veins, the
nurse would explain that this is most probably the result of which of the following?

A. Thrombophlebitis
B. Pregnancy-induced hypertension
73
C. Pressure on blood vessels from the enlarging uterus
D. The force of gravity pulling down on the uterus
9. Cervical softening and uterine souffle are classified as which of the following?

A. Diagnostic signs
B. Presumptive signs
C. Probable signs
D. Positive signs
10. Which of the following would the nurse identify as a presumptive sign of pregnancy?

A. Hegar sign
B. Nausea and vomiting
C. Skin pigmentation changes
D. Positive serum pregnancy test
11. Which of the following common emotional reactions to pregnancy would the nurse
expect to occur during the first trimester?

A. Introversion, egocentrism, narcissism


B. Awkwardness, clumsiness, and unattractiveness
C. Anxiety, passivity, extroversion
D. Ambivalence, fear, fantasies
12. During which of the following would the focus of classes be mainly on physiologic
changes, fetal development, sexuality, during pregnancy, and nutrition?

A. Prepregnant period
B. First trimester
C. Second trimester
D. Third trimester
13. Which of the following would be disadvantage of breast feeding?

A. Involution occurs more rapidly


B. The incidence of allergies increases due to maternal antibodies
C. The father may resent the infant’s demands on the mother’s body
D. There is a greater chance for error during preparation
14. Which of the following would cause a false-positive result on a pregnancy test?

A. The test was performed less than 10 days after an abortion


B. The test was performed too early or too late in the pregnancy
C. The urine sample was stored too long at room temperature
D. A spontaneous abortion or a missed abortion is impending
15. FHR can be auscultated with a fetoscope as early as which of the following?

A. 5 weeks gestation
B. 10 weeks gestation
C. 15 weeks gestation
D. 20 weeks gestation
16. A client LMP began July 5. Her EDD should be which of the following?

A. January 2
B. March 28
C. April 12
D. October 12

74
17. Which of the following fundal heights indicates less than 12 weeks’ gestation when the
date of the LMP is unknown?

A. Uterus in the pelvis


B. Uterus at the xiphoid
C. Uterus in the abdomen
D. Uterus at the umbilicus
18. Which of the following danger signs should be reported promptly during the antepartum
period?

A. Constipation
B. Breast tenderness
C. Nasal stuffiness
D. Leaking amniotic fluid
19. Which of the following prenatal laboratory test values would the nurse consider as
significant?

A. Hematocrit 33.5%
B. Rubella titer less than 1:8
C. White blood cells 8,000/mm3
D. One hour glucose challenge test 110 g/dL
20. Which of the following characteristics of contractions would the nurse expect to find in a
client experiencing true labor?

A. Occurring at irregular intervals


B. Starting mainly in the abdomen
C. Gradually increasing intervals
D. Increasing intensity with walking
21. During which of the following stages of labor would the nurse assess “crowning”?

A. First stage
B. Second stage
C. Third stage
D. Fourth stage
22. Barbiturates are usually not given for pain relief during active labor for which of the
following reasons?

A. The neonatal effects include hypotonia, hypothermia, generalized drowsiness, and


reluctance to feed for the first few days.
B. These drugs readily cross the placental barrier, causing depressive effects in the newborn 2
to 3 hours after intramuscular injection.
C. They rapidly transfer across the placenta, and lack of an antagonist make them generally
inappropriate during labor.
D. Adverse reactions may include maternal hypotension, allergic or toxic reaction or partial or
total respiratory failure
23. Which of the following nursing interventions would the nurse perform during the third
stage of labor?

A. Obtain a urine specimen and other laboratory tests.


B. Assess uterine contractions every 30 minutes.
C. Coach for effective client pushing
D. Promote parent-newborn interaction.
75
24. Which of the following actions demonstrates the nurse’s understanding about the
newborn’s thermoregulatory ability?

A. Placing the newborn under a radiant warmer.


B. Suctioning with a bulb syringe
C. Obtaining an Apgar score
D. Inspecting the newborn’s umbilical cord
25. Immediately before expulsion, which of the following cardinal movements occur?

A. Descent
B. Flexion
C. Extension
D. External rotation
26. Before birth, which of the following structures connects the right and left auricles of the
heart?

A. Umbilical vein
B. Foramen ovale
C. Ductus arteriosus
D. Ductus venosus
27. Which of the following when present in the urine may cause a reddish stain on the
diaper of a newborn?

A. Mucus
B. Uric acid crystals
C. Bilirubin
D. Excess iron
28. When assessing the newborn’s heart rate, which of the following ranges would be
considered normal if the newborn were sleeping?

A. 80 beats per minute


B. 100 beats per minute
C. 120 beats per minute
D. 140 beats per minute
29. Which of the following is true regarding the fontanels of the newborn?

A. The anterior is triangular shaped; the posterior is diamond shaped.


B. The posterior closes at 18 months; the anterior closes at 8 to 12 weeks.
C. The anterior is large in size when compared to the posterior fontanel.
D. The anterior is bulging; the posterior appears sunken.
30. Which of the following groups of newborn reflexes below are present at birth and
remain unchanged through adulthood?

A. Blink, cough, rooting, and gag


B. Blink, cough, sneeze, gag
C. Rooting, sneeze, swallowing, and cough
D. Stepping, blink, cough, and sneeze
31. Which of the following describes the Babinski reflex?

A. The newborn’s toes will hyperextend and fan apart from dorsiflexion of the big toe when
one side of foot is stroked upward from the ball of the heel and across the ball of the foot.

76
B. The newborn abducts and flexes all extremities and may begin to cry when exposed to
sudden movement or loud noise.
C. The newborn turns the head in the direction of stimulus, opens the mouth, and begins to
suck when cheek, lip, or corner of mouth is touched.
D. The newborn will attempt to crawl forward with both arms and legs when he is placed on his
abdomen on a flat surface
32. Which of the following statements best describes hyperemesis gravidarum?

A. Severe anemia leading to electrolyte, metabolic, and nutritional imbalances in the absence of
other medical problems.
B. Severe nausea and vomiting leading to electrolyte, metabolic, and nutritional imbalances in
the absence of other medical problems.
C. Loss of appetite and continuous vomiting that commonly results in dehydration and
ultimately decreasing maternal nutrients
D. Severe nausea and diarrhea that can cause gastrointestinal irritation and possibly internal
bleeding
33. Which of the following would the nurse identify as a classic sign of PIH?

A. Edema of the feet and ankles


B. Edema of the hands and face
C. Weight gain of 1 lb/week
D. Early morning headache
34. In which of the following types of spontaneous abortions would the nurse assess dark
brown vaginal discharge and a negative pregnancy tests?

A. Threatened
B. Imminent
C. Missed
D. Incomplete
35. Which of the following factors would the nurse suspect as predisposing a client to
placenta previa?

A. Multiple gestation
B. Uterine anomalies
C. Abdominal trauma
D. Renal or vascular disease
36. Which of the following would the nurse assess in a client experiencing abruptio
placenta?

A. Bright red, painless vaginal bleeding


B. Concealed or external dark red bleeding
C. Palpable fetal outline
D. Soft and nontender abdomen
37. Which of the following is described as premature separation of a normally implanted
placenta during the second half of pregnancy, usually with severe hemorrhage?

A. Placenta previa
B. Ectopic pregnancy
C. Incompetent cervix
D. Abruptio placentae
38. Which of the following may happen if the uterus becomes overstimulated by oxytocin
during the induction of labor?
77
A. Weak contraction prolonged to more than 70 seconds
B. Tetanic contractions prolonged to more than 90 seconds
C. Increased pain with bright red vaginal bleeding
D. Increased restlessness and anxiety
39. When preparing a client for cesarean delivery, which of the following key concepts
should be considered when implementing nursing care?

A. Instruct the mother’s support person to remain in the family lounge until after the delivery
B. Arrange for a staff member of the anesthesia department to explain what to expect
postoperatively
C. Modify preoperative teaching to meet the needs of either a planned or emergency cesarean
birth
D. Explain the surgery, expected outcome, and kind of anesthetics
40. Which of the following best describes preterm labor?

A. Labor that begins after 20 weeks gestation and before 37 weeks gestation
B. Labor that begins after 15 weeks gestation and before 37 weeks gestation
C. Labor that begins after 24 weeks gestation and before 28 weeks gestation
D. Labor that begins after 28 weeks gestation and before 40 weeks gestation
41. When PROM occurs, which of the following provides evidence of the nurse’s
understanding of the client’s immediate needs?

A. The chorion and amnion rupture 4 hours before the onset of labor.
B. PROM removes the fetus most effective defense against infection
C. Nursing care is based on fetal viability and gestational age.
D. PROM is associated with malpresentation and possibly incompetent cervix
42. Which of the following factors is the underlying cause of dystocia?

A. Nurtional
B. Mechanical
C. Environmental
D. Medical
43. When uterine rupture occurs, which of the following would be the priority?

A. Limiting hypovolemic shock


B. Obtaining blood specimens
C. Instituting complete bed rest
D. Inserting a urinary catheter
44. Which of the following is the nurse’s initial action when umbilical cord prolapse occurs?

A. Begin monitoring maternal vital signs and FHR


B. Place the client in a knee-chest position in bed
C. Notify the physician and prepare the client for delivery
D. Apply a sterile warm saline dressing to the exposed cord
45. Which of the following amounts of blood loss following birth marks the criterion for
describing postpartum hemorrhage?

A. More than 200 ml


B. More than 300 ml
C. More than 400 ml
D. More than 500 ml
46. Which of the following is the primary predisposing factor related to mastitis?

78
A. Epidemic infection from nosocomial sources localizing in the lactiferous glands and ducts
B. Endemic infection occurring randomly and localizing in the periglandular connective tissue
C. Temporary urinary retention due to decreased perception of the urge to avoid
D. Breast injury caused by overdistention, stasis, and cracking of the nipples
47. Which of the following best describes thrombophlebitis?

A. Inflammation and clot formation that result when blood components combine to form an
aggregate body
B. Inflammation and blood clots that eventually become lodged within the pulmonary blood
vessels
C. Inflammation and blood clots that eventually become lodged within the femoral vein
D. Inflammation of the vascular endothelium with clot formation on the vessel wall
48. Which of the following assessment findings would the nurse expect if the client
develops DVT?

A. Midcalf pain, tenderness and redness along the vein


B. Chills, fever, malaise, occurring 2 weeks after delivery
C. Muscle pain the presence of Homans sign, and swelling in the affected limb
D. Chills, fever, stiffness, and pain occurring 10 to 14 days after delivery
49. Which of the following are the most commonly assessed findings in cystitis?

A. Frequency, urgency, dehydration, nausea, chills, and flank pain


B. Nocturia, frequency, urgency dysuria, hematuria, fever and suprapubic pain
C. Dehydration, hypertension, dysuria, suprapubic pain, chills, and fever
D. High fever, chills, flank pain nausea, vomiting, dysuria, and frequency
50. Which of the following best reflects the frequency of reported postpartum “blues”?

A. Between 10% and 40% of all new mothers report some form of postpartum blues
B. Between 30% and 50% of all new mothers report some form of postpartum blues
C. Between 50% and 80% of all new mothers report some form of postpartum blues
D. Between 25% and 70% of all new mothers report some form of postpartum blues
Answers and Rationales
1. B. Although all of the factors listed are important, sperm motility is the most significant
criterion when assessing male infertility. Sperm count, sperm maturity, and semen volume are all
significant, but they are not as significant sperm motility.
2. D. Based on the partner’s statement, the couple is verbalizing feelings of inadequacy and
negative feelings about themselves and their capabilities. Thus, the nursing diagnosis of self-esteem
disturbance is most appropriate. Fear, pain, and ineffective family coping also may be present but as
secondary nursing diagnoses.
3. B. Pressure and irritation of the bladder by the growing uterus during the first trimester is
responsible for causing urinary frequency. Dysuria, incontinence, and burning are symptoms
associated with urinary tract infections.
4. C. During the second trimester, the reduction in gastric acidity in conjunction with pressure
from the growing uterus and smooth muscle relaxation, can cause heartburn and flatulence. HCG
levels increase in the first, not the second, trimester. Decrease intestinal motility would most likely
be the cause of constipation and bloating. Estrogen levels decrease in the second trimester.
5. D. Chloasma, also called the mask of pregnancy, is an irregular hyperpigmented area found
on the face. It is not seen on the breasts, areola, nipples, chest, neck, arms, legs, abdomen, or thighs.
6. C. During pregnancy, hormonal changes cause relaxation of the pelvic joints, resulting in
the typical “waddling” gait. Changes in posture are related to the growing fetus. Pressure on the
surrounding muscles causing discomfort is due to the growing uterus. Weight gain has no effect on
gait.
7. C. The average amount of weight gained during pregnancy is 24 to 30 lb. This weight gain
consists of the following: fetus – 7.5 lb; placenta and membrane – 1.5 lb; amniotic fluid – 2 lb;
79
uterus – 2.5 lb; breasts – 3 lb; and increased blood volume – 2 to 4 lb; extravascular fluid and fat – 4
to 9 lb. A gain of 12 to 22 lb is insufficient, whereas a weight gain of 15 to 25 lb is marginal. A
weight gain of 25 to 40 lb is considered excessive.
8. C. Pressure of the growing uterus on blood vessels results in an increased risk for venous
stasis in the lower extremities. Subsequently, edema and varicose vein formation may occur.
Thrombophlebitis is an inflammation of the veins due to thrombus formation. Pregnancy-induced
hypertension is not associated with these symptoms. Gravity plays only a minor role with these
symptoms.
9. C. Cervical softening (Goodell sign) and uterine soufflé are two probable signs of
pregnancy. Probable signs are objective findings that strongly suggest pregnancy. Other probable
signs include Hegar sign, which is softening of the lower uterine segment; Piskacek sign, which is
enlargement and softening of the uterus; serum laboratory tests; changes in skin pigmentation; and
ultrasonic evidence of a gestational sac. Presumptive signs are subjective signs and include
amenorrhea; nausea and vomiting; urinary frequency; breast tenderness and changes; excessive
fatigue; uterine enlargement; and quickening.
10. B. Presumptive signs of pregnancy are subjective signs. Of the signs listed, only nausea and
vomiting are presumptive signs. Hegar sign,skin pigmentation changes, and a positive serum
pregnancy test are considered probably signs, which are strongly suggestive of pregnancy.
11. D. During the first trimester, common emotional reactions include ambivalence, fear,
fantasies, or anxiety. The second trimester is a period of well-being accompanied by the increased
need to learn about fetal growth and development. Common emotional reactions during this
trimester include narcissism, passivity, or introversion. At times the woman may seem egocentric
and self-centered. During the third trimester, the woman typically feels awkward, clumsy, and
unattractive, often becoming more introverted or reflective of her own childhood.
12. B. First-trimester classes commonly focus on such issues as early physiologic changes, fetal
development, sexuality during pregnancy, and nutrition. Some early classes may include pregnant
couples. Second and third trimester classes may focus on preparation for birth, parenting, and
newborn care.
13. C. With breast feeding, the father’s body is not capable of providing the milk for the
newborn, which may interfere with feeding the newborn, providing fewer chances for bonding, or he
may be jealous of the infant’s demands on his wife’s time and body. Breast feeding is advantageous
because uterine involution occurs more rapidly, thus minimizing blood loss. The presence of
maternal antibodies in breast milk helps decrease the incidence of allergies in the newborn. A greater
chance for error is associated with bottle feeding. No preparation is required for breast feeding.
14. A. A false-positive reaction can occur if the pregnancy test is performed less than 10 days
after an abortion. Performing the tests too early or too late in the pregnancy, storing the urine sample
too long at room temperature, or having a spontaneous or missed abortion impending can all produce
false-negative results.
15. D. The FHR can be auscultated with a fetoscope at about 20 week’s gestation. FHR usually
is ausculatated at the midline suprapubic region with Doppler ultrasound transducer at 10 to 12
week’s gestation. FHR, cannot be heard any earlier than 10 weeks’ gestation.
16. C. To determine the EDD when the date of the client’s LMP is known use Nagele rule. To
the first day of the LMP, add 7 days, subtract 3 months, and add 1 year (if applicable) to arrive at the
EDD as follows: 5 + 7 = 12 (July) minus 3 = 4 (April). Therefore, the client’s EDD is April 12.
17. A. When the LMP is unknown, the gestational age of the fetus is estimated by uterine size or
position (fundal height). The presence of the uterus in the pelvis indicates less than 12 weeks’
gestation. At approximately 12 to 14 weeks, the fundus is out of the pelvis above the symphysis
pubis. The fundus is at the level of the umbilicus at approximately 20 weeks’ gestation and reaches
the xiphoid at term or 40 weeks.
18. D. Danger signs that require prompt reporting leaking of amniotic fluid, vaginal bleeding,
blurred vision, rapid weight gain, and elevated blood pressure. Constipation, breast tenderness, and
nasal stuffiness are common discomforts associated with pregnancy.
19. B. A rubella titer should be 1:8 or greater. Thurs, a finding of a titer less than 1:8 is
significant, indicating that the client may not possess immunity to rubella. A hematocrit of 33.5% a
white blood cell count of 8,000/mm3, and a 1 hour glucose challenge test of 110 g/dl are with
normal parameters.

80
20. D. With true labor, contractions increase in intensity with walking. In addition, true labor
contractions occur at regular intervals, usually starting in the back and sweeping around to the
abdomen. The interval of true labor contractions gradually shortens.
21. B. Crowing, which occurs when the newborn’s head or presenting part appears at the
vaginal opening, occurs during the second stage of labor. During the first stage of labor, cervical
dilation and effacement occur. During the third stage of labor, the newborn and placenta are
delivered. The fourth stage of labor lasts from 1 to 4 hours after birth, during which time the mother
and newborn recover from the physical process of birth and the mother’s organs undergo the initial
readjustment to the nonpregnant state.
22. C. Barbiturates are rapidly transferred across the placental barrier, and lack of an antagonist
makes them generally inappropriate during active labor. Neonatal side effects of barbiturates include
central nervous system depression, prolonged drowsiness, delayed establishment of feeding (e.g. due
to poor sucking reflex or poor sucking pressure). Tranquilizers are associated with neonatal effects
such as hypotonia, hypothermia, generalized drowsiness, and reluctance to feed for the first few
days. Narcotic analgesic readily cross the placental barrier, causing depressive effects in the
newborn 2 to 3 hours after intramuscular injection. Regional anesthesia is associated with adverse
reactions such as maternal hypotension, allergic or toxic reaction, or partial or total respiratory
failure.
23. D. During the third stage of labor, which begins with the delivery of the newborn, the nurse
would promote parent-newborn interaction by placing the newborn on the mother’s abdomen and
encouraging the parents to touch the newborn. Collecting a urine specimen and other laboratory tests
is done on admission during the first stage of labor. Assessing uterine contractions every 30 minutes
is performed during the latent phase of the first stage of labor. Coaching the client to push
effectively is appropriate during the second stage of labor.
24. A. The newborn’s ability to regulate body temperature is poor. Therefore, placing the
newborn under a radiant warmer aids in maintaining his or her body temperature. Suctioning with a
bulb syringe helps maintain a patent airway. Obtaining an Apgar score measures the newborn’s
immediate adjustment to extrauterine life. Inspecting the umbilical cord aids in detecting cord
anomalies.
25. D. Immediately before expulsion or birth of the rest of the body, the cardinal movement of
external rotation occurs. Descent flexion, internal rotation, extension, and restitution (in this order)
occur before external rotation.
26. B. The foramen ovale is an opening between the right and left auricles (atria) that should
close shortly after birth so the newborn will not have a murmur or mixed blood traveling through the
vascular system. The umbilical vein, ductus arteriosus, and ductus venosus are obliterated at birth.
27. B. Uric acid crystals in the urine may produce the reddish “brick dust” stain on the diaper.
Mucus would not produce a stain. Bilirubin and iron are from hepatic adaptation.
28. B. The normal heart rate for a newborn that is sleeping is approximately 100 beats per
minute. If the newborn was awake, the normal heart rate would range from 120 to 160 beats per
minute.
29. C. The anterior fontanel is larger in size than the posterior fontanel. Additionally, the
anterior fontanel, which is diamond shaped, closes at 18 months, whereas the posterior fontanel,
which is triangular shaped, closes at  8 to 12 weeks. Neither fontanel should appear bulging, which
may indicate increased intracranial pressure, or sunken, which may indicate dehydration.
30. B. Blink, cough, sneeze, swallowing and gag reflexes are all present at birth and remain
unchanged through adulthood. Reflexes such as rooting and stepping subside within the first year.
31. A. With the babinski reflex, the newborn’s toes hyperextend and fan apart from dorsiflexion
of the big toe when one side of foot is stroked upward form the heel and across the ball of the foot.
With the startle reflex, the newborn abducts and flexes all extremities and may begin to cry when
exposed to sudden movement of loud noise. With the rooting and sucking reflex, the newborn turns
his head in the direction of stimulus, opens the mouth, and begins to suck when the cheeks, lip, or
corner of mouth is touched. With the crawl reflex, the newborn will attempt to crawl forward with
both arms and legs when he is placed on his abdomen on a flat surface.
32. B. The description of hyperemesis gravidarum includes severe nausea and vomiting, leading
to electrolyte, metabolic, and nutritional imbalances in the absence of other medical problems.
Hyperemesis is not a form of anemia. Loss of appetite may occur secondary to the nausea and

81
vomiting of hyperemesis, which, if it continues, can deplete the nutrients transported to the fetus.
Diarrhea does not occur with hyperemesis.
33. B. Edema of the hands and face is a classic sign of PIH. Many healthy pregnant woman
experience foot and ankle edema. A weight gain of 2 lb or more per week indicates a problem. Early
morning headache is not a classic sign of PIH.
34. C. In a missed abortion, there is early fetal intrauterine death, and products of conception are
not expelled. The cervix remains closed; there may be a dark brown vaginal discharge, negative
pregnancy test, and cessation of uterine growth and breast tenderness. A threatened abortion is
evidenced with cramping and vaginal bleeding in early pregnancy, with no cervical dilation. An
incomplete abortion presents with bleeding, cramping, and cervical dilation. An incomplete abortion
involves only expulsion of part of the products of conception and bleeding occurs with cervical
dilation.
35. A. Multiple gestation is one of the predisposing factors that may cause placenta previa.
Uterine anomalies abdominal trauma, and renal or vascular disease may predispose a client to
abruptio placentae.
36. B. A client with abruptio placentae may exhibit concealed or dark red bleeding, possibly
reporting sudden intense localized uterine pain. The uterus is typically firm to boardlike, and the
fetal presenting part may be engaged. Bright red, painless vaginal bleeding, a palpable fetal outline
and a soft nontender abdomen are manifestations of placenta previa.
37. D. Abruptio placentae is described as premature separation of a normally implanted placenta
during the second half of pregnancy, usually with severe hemorrhage. Placenta previa refers to
implantation of the placenta in the lower uterine segment, causing painless bleeding in the third
trimester of pregnancy. Ectopic pregnancy refers to the implantation of the products of conception in
a site other than the endometrium. Incompetent cervix is a conduction characterized by painful
dilation of the cervical os without uterine contractions.
38. B. Hyperstimulation of the uterus such as with oxytocin during the induction of labor may
result in tetanic contractions prolonged to more than 90seconds, which could lead to such
complications as fetal distress, abruptio placentae, amniotic fluid embolism, laceration of the cervix,
and uterine rupture. Weak contractions would not occur. Pain, bright red vaginal bleeding, and
increased restlessness and anxiety are not associated with hyperstimulation.
39. C. A key point to consider when preparing the client for a cesarean delivery is to modify the
preoperative teaching to meet the needs of either a planned or emergency cesarean birth, the depth
and breadth of instruction will depend on circumstances and time available. Allowing the mother’s
support person to remain with her as much as possible is an important concept, although doing so
depends on many variables. Arranging for necessary explanations by various staff members to be
involved with the client’s care is a nursing responsibility. The nurse is responsible for reinforcing the
explanations about the surgery, expected outcome, and type of anesthetic to be used. The
obstetrician is responsible for explaining about the surgery and outcome and the anesthesiology staff
is responsible for explanations about the type of anesthesia to be used.
40. A. Preterm labor is best described as labor that begins after 20 weeks’ gestation and before
37 weeks’ gestation. The other time periods are inaccurate.
41. B. PROM can precipitate many potential and actual problems; one of the most serious is the
fetus loss of an effective defense against infection. This is the client’s most immediate need at this
time. Typically, PROM occurs about 1 hour, not 4 hours, before labor begins. Fetal viability and
gestational age are less immediate considerations that affect the plan of care. Malpresentation and an
incompetent cervix may be causes of PROM.
42. B. Dystocia is difficult, painful, prolonged labor due to mechanical factors involving the
fetus (passenger), uterus (powers), pelvis (passage), or psyche. Nutritional, environment, and
medical factors may contribute to the mechanical factors that cause dystocia.
43. A. With uterine rupture, the client is at risk for hypovolemic shock. Therefore, the priority is
to prevent and limit hypovolemic shock. Immediate steps should include giving oxygen, replacing
lost fluids, providing drug therapy as needed, evaluating fetal responses and preparing for surgery.
Obtaining blood specimens, instituting complete bed rest, and inserting a urinary catheter are
necessary in preparation for surgery to remedy the rupture.
44. B. The immediate priority is to minimize pressure on the cord. Thus the nurse’s initial action
involves placing the client on bed rest and then placing the client in a knee-chest position or
lowering the head of the bed, and elevating the maternal hips on a pillow to minimize the pressure
82
on the cord. Monitoring maternal vital signs and FHR, notifying the physician and preparing the
client for delivery, and wrapping the cord with sterile saline soaked warm gauze are important. But
these actions have no effect on minimizing the pressure on the cord.
45. D. Postpartum hemorrhage is defined as blood loss of more than 500 ml following birth.
Any amount less than this not considered postpartum hemorrhage.
46. D. With mastitis, injury to the breast, such as overdistention, stasis, and cracking of the
nipples, is the primary predisposing factor. Epidemic and endemic infections are probable sources of
infection for mastitis. Temporary urinary retention due to decreased perception of the urge to void is
a contributory factor to the development of urinary tract infection, not mastitis.
47. D. Thrombophlebitis refers to an inflammation of the vascular endothelium with clot
formation on the wall of the vessel. Blood components combining to form an aggregate body
describe a thrombus or thrombosis. Clots lodging in the pulmonary vasculature refers to pulmonary
embolism; in the femoral vein, femoral thrombophlebitis.
48. C. Classic symptoms of DVT include muscle pain, the presence of Homans sign, and
swelling of the affected limb. Midcalf pain, tenderness, and redness, along the vein reflect
superficial thrombophlebitis. Chills, fever and malaise occurring 2 weeks after delivery reflect pelvic
thrombophlebitis. Chills, fever,  stiffness and pain occurring 10 to 14 days after delivery suggest
femoral thrombophlebitis.
49. B. Manifestations of cystitis include, frequency, urgency, dysuria, hematuria nocturia, fever,
and suprapubic pain. Dehydration, hypertension, and chills are not typically associated with cystitis.
High fever chills, flank pain, nausea, vomiting, dysuria, and frequency are associated with
pvelonephritis.
50. C. According to statistical reports, between 50% and 80% of all new mothers report some
form of postpartum blues. The ranges of 10% to 40%, 30% to 50%, and 25% to 70% are incorrect.

Text Mode – Text version of the exam


1. A postpartum nurse is preparing to care for a woman who has just delivered a healthy
newborn infant. In the immediate postpartum period the nurse plans to take the woman’s
vital signs:

A. Every 30 minutes during the first hour and then every hour for the next two hours.
B. Every 15 minutes during the first hour and then every 30 minutes for the next two hours.
C. Every hour for the first 2 hours and then every 4 hours
D. Every 5 minutes for the first 30 minutes and then every hour for the next 4 hours.
2. A postpartum nurse is taking the vital signs of a woman who delivered a healthy
newborn infant 4 hours ago. The nurse notes that the mother’s temperature is 100.2*F.
Which of the following actions would be most appropriate?

A. Retake the temperature in 15 minutes


B. Notify the physician
C. Document the findings
D. Increase hydration by encouraging oral fluids
3. The nurse is assessing a client who is 6 hours PP after delivering a full-term healthy
infant. The client complains to the nurse of feelings of faintness and dizziness. Which of the
following nursing actions would be most appropriate?

A. Obtain hemoglobin and hematocrit levels


B. Instruct the mother to request help when getting out of bed
C. Elevate the mother’s legs
D. Inform the nursery room nurse to avoid bringing the newborn infant to the mother until the
feelings of lightheadedness and dizziness have subsided.

83
4. A nurse is preparing to perform a fundal assessment on a postpartum client. The initial
nursing action in performing this assessment is which of the following?

A. Ask the client to turn on her side


B. Ask the client to lie flat on her back with the knees and legs flat and straight.
C. Ask the mother to urinate and empty her bladder
D. Massage the fundus gently before determining the level of the fundus.
5. The nurse is assessing the lochia on a 1 day PP patient. The nurse notes that the lochia
is red and has a foul-smelling odor. The nurse determines that this assessment finding is:

A. Normal
B. Indicates the presence of infection
C. Indicates the need for increasing oral fluids
D. Indicates the need for increasing ambulation
6. When performing a PP assessment on a client, the nurse notes the presence of clots in
the lochia. The nurse examines the clots and notes that they are larger than 1 cm. Which of
the following nursing actions is most appropriate?

A. Document the findings


B. Notify the physician
C. Reassess the client in 2 hours
D. Encourage increased intake of fluids.
7. A nurse in a PP unit is instructing a mother regarding lochia and the amount of expected
lochia drainage. The nurse instructs the mother that the normal amount of lochia may vary
but should never exceed the need for:

A. One peripad per day


B. Two peripads per day
C. Three peripads per day
D. Eight peripads per day
8. A PP nurse is providing instructions to a woman after delivery of a healthy newborn
infant. The nurse instructs the mother that she should expect normal bowel elimination to
return:

A. One the day of the delivery


B. 3 days PP
C. 7 days PP
D. within 2 weeks PP
9. Select all of the physiological maternal changes that occur during the PP period.

A. Cervical involution ceases immediately


B. Vaginal distention decreases slowly
C. Fundus begins to descend into the pelvis after 24 hours
D. Cardiac output decreases with resultant tachycardia in the first 24 hours
E. Digestive processes slow immediately.
10. A nurse is caring for a PP woman who has received epidural anesthesia and is
monitoring the woman for the presence of a vulva hematoma. Which of the following
assessment findings would best indicate the presence of a hematoma?

A. Complaints of a tearing sensation


84
B. Complaints of intense pain
C. Changes in vital signs
D. Signs of heavy bruising
11. A nurse is developing a plan of care for a PP woman with a small vulvar hematoma.
The nurse includes which specific intervention in the plan during the first 12 hours following
the delivery of this client?

A. Assess vital signs every 4 hours


B. Inform health care provider of assessment findings
C. Measure fundal height every 4 hours
D. Prepare an ice pack for application to the area.
12. A new mother received epidural anesthesia during labor and had a forceps delivery
after pushing 2 hours. At 6 hours PP, her systolic blood pressure has dropped 20 points,
her diastolic BP has dropped 10 points, and her pulse is 120 beats per minute. The client is
anxious and restless. On further assessment, a vulvar hematoma is verified. After notifying
the health care provider, the nurse immediately plans to:

A. Monitor fundal height


B. Apply perineal pressure
C. Prepare the client for surgery.
D. Reassure the client
13. A nurse is monitoring a new mother in the PP period for signs of hemorrhage. Which of
the following signs, if noted in the mother, would be an early sign of excessive blood loss?

A. A temperature of 100.4*F
B. An increase in the pulse from 88 to 102 BPM
C. An increase in the respiratory rate from 18 to 22 breaths per minute
D. A blood pressure change from 130/88 to 124/80 mm Hg
14. A nurse is preparing to assess the uterine fundus of a client in the immediate
postpartum period. When the nurse locates the fundus, she notes that the uterus feels soft
and boggy. Which of the following nursing interventions would be most appropriate initially?

A. Massage the fundus until it is firm


B. Elevate the mothers legs
C. Push on the uterus to assist in expressing clots
D. Encourage the mother to void
15. A PP nurse is assessing a mother who delivered a healthy newborn infant by C-section.
The nurse is assessing for signs and symptoms of superficial venous thrombosis. Which of
the following signs or symptoms would the nurse note if superficial venous thrombosis were
present?

A. Paleness of the calf area


B. Enlarged, hardened veins
C. Coolness of the calf area
D. Palpable dorsalis pedis pulses
16. A nurse is providing instructions to a mother who has been diagnosed with mastitis.
Which of the following statements if made by the mother indicates a need for further
teaching?

85
A. “I need to take antibiotics, and I should begin to feel better in 24-48 hours.”
B. “I can use analgesics to assist in alleviating some of the discomfort.”
C. “I need to wear a supportive bra to relieve the discomfort.”
D. “I need to stop breastfeeding until this condition resolves.”
17. A PP client is being treated for DVT. The nurse understands that the client’s response
to treatment will be evaluated by regularly assessing the client for:

A. Dysuria, ecchymosis, and vertigo


B. Epistaxis, hematuria, and dysuria
C. Hematuria, ecchymosis, and epistaxis
D. Hematuria, ecchymosis, and vertigo
18. A nurse performs an assessment on a client who is 4 hours PP. The nurse notes that
the client has cool, clammy skin and is restless and excessively thirsty. The nurse prepares
immediately to:

A. Assess for hypovolemia and notify the health care provider


B. Begin hourly pad counts and reassure the client
C. Begin fundal massage and start oxygen by mask
D. Elevate the head of the bed and assess vital signs
19. A nurse is assessing a client in the 4th stage if labor and notes that the fundus is firm
but that bleeding is excessive. The initial nursing action would be which of the following?

A. Massage the fundus


B. Place the mother in the Trendelenburg’s position
C. Notify the physician
D. Record the findings
20. A nurse is caring for a PP client with a diagnosis of DVT who is receiving a continuous
intravenous infusion of heparin sodium. Which of the following laboratory results will the
nurse specifically review to determine if an effective and appropriate dose of the heparin is
being delivered?

A. Prothrombin time
B. International normalized ratio
C. Activated partial thromboplastin time
D. Platelet count
21. A nurse is preparing a list of self-care instructions for a PP client who was diagnosed
with mastitis. Select all instructions that would be included on the list.

A. Take the prescribed antibiotics until the soreness subsides.


B. Wear supportive bra
C. Avoid decompression of the breasts by breastfeeding or breast pump
D. Rest during the acute phase
E. Continue to breastfeed if the breasts are not too sore.
22. Methergine or pitocin is prescribed for a woman to treat PP hemorrhage. Before
administration of these medications, the priority nursing assessment is to check the:

A. Amount of lochia
B. Blood pressure
C. Deep tendon reflexes
D. Uterine tone

86
23. Methergine or pitocin are prescribed for a client with PP hemorrhage. Before
administering the medication(s), the nurse contacts the health provider who prescribed the
medication(s) in which of the following conditions is documented in the client’s medical
history?

A. Peripheral vascular disease


B. Hypothyroidism
C. Hypotension
D. Type 1 diabetes
24. Which of the following factors might result in a decreased supply of breastmilk in a PP
mother?

A. Supplemental feedings with formula


B. Maternal diet high in vitamin C
C. An alcoholic drink
D. Frequent feedings
25. Which of the following interventions would be helpful to a breastfeeding mother who is
experiencing engorged breasts?

A. Applying ice
B. Applying a breast binder
C. Teaching how to express her breasts in a warm shower
D. Administering bromocriptine (Parlodel)
26. On completing a fundal assessment, the nurse notes the fundus is situated on the
client’s left abdomen. Which of the following actions is appropriate?

A. Ask the client to empty her bladder


B. Straight catheterize the client immediately
C. Call the client’s health provider for direction
D. Straight catheterize the client for half of her uterine volume
27. The nurse is about the give a Type 2 diabetic her insulin before breakfast on her first
day postpartum. Which of the following answers best describes insulin requirements
immediately postpartum?

A. Lower than during her pregnancy


B. Higher than during her pregnancy
C. Lower than before she became pregnant
D. Higher than before she became pregnant
28. Which of the following findings would be expected when assessing the postpartum
client?

A. Fundus 1 cm above the umbilicus 1 hour postpartum


B. Fundus 1 cm above the umbilicus on postpartum day 3
C. Fundus palpable in the abdomen at 2 weeks postpartum
D. Fundus slightly to the right; 2 cm above umbilicus on postpartum day 2
29. A client is complaining of painful contractions, or afterpains, on postpartum day 2.
Which of the following conditions could increase the severity of afterpains?

A. Bottle-feeding
B. Diabetes
87
C. Multiple gestation
D. Primiparity
30. On which of the postpartum days can the client expect lochia serosa?

A. Days 3 and 4 PP
B. Days 3 to 10 PP
C. Days 10-14 PP
D. Days 14 to 42 PP
31. Which of the following behaviors characterizes the PP mother in the taking inphase?

A. Passive and dependant


B. Striving for independence and autonomy
C. Curious and interested in care of the baby
D. Exhibiting maximum readiness for new learning
32. Which of the following complications may be indicated by continuous seepage of blood
from the vagina of a PP client, when palpation of the uterus reveals a firm uterus 1 cm
below the umbilicus?

A. Retained placental fragments


B. Urinary tract infection
C. Cervical laceration
D. Uterine atony
33. What type of milk is present in the breasts 7 to 10 days PP?

A. Colostrum
B. Hind milk
C. Mature milk
D. Transitional milk
34. Which of the following complications is most likely responsible for a delayed postpartum
hemorrhage?

A. Cervical laceration
B. Clotting deficiency
C. Perineal laceration
D. Uterine subinvolution
35. Before giving a PP client the rubella vaccine, which of the following facts should the
nurse include in client teaching?

A. The vaccine is safe in clients with egg allergies


B. Breast-feeding isn’t compatible with the vaccine
C. Transient arthralgia and rash are common adverse effects
D. The client should avoid getting pregnant for 3 months after the vaccine because the vaccine
has teratogenic effects
36. Which of the following changes best described the insulin needs of a client with type 1
diabetes who has just delivered an infant vaginally without complications?

A. Increase
B. Decrease
C. Remain the same as before pregnancy
D. Remain the same as during pregnancy

88
37. Which of the following responses is most appropriate for a mother with diabetes who
wants to breastfeed her infant but is concerned about the effects of breastfeeding on her
health?

A. Mothers with diabetes who breastfeed have a hard time controlling their insulin needs
B. Mothers with diabetes shouldn’t breastfeed because of potential complications
C. Mothers with diabetes shouldn’t breastfeed; insulin requirements are doubled.
D. Mothers with diabetes may breastfeed; insulin requirements may decrease from
breastfeeding.
38. On the first PP night, a client requests that her baby be sent back to the nursery so she
can get some sleep. The client is most likely in which of the following phases?

A. Depression phase
B. Letting-go phase
C. Taking-hold phase
D. Taking-in phase
39. Which of the following physiological responses is considered normal in the early
postpartum period?

A. Urinary urgency and dysuria


B. Rapid diuresis
C. Decrease in blood pressure
D. Increase motility of the GI system
40. During the 3rd PP day, which of the following observations about the client would the
nurse be most likely to make?

A. The client appears interested in learning about neonatal care


B. The client talks a lot about her birth experience
C. The client sleeps whenever the neonate isn’t present
D. The client requests help in choosing a name for the neonate.
41. Which of the following circumstances is most likely to cause uterine atony and lead to
PP hemorrhage?

A. Hypertension
B. Cervical and vaginal tears
C. Urine retention
D. Endometritis
42. Which type of lochia should the nurse expect to find in a client 2 days PP?

A. Foul-smelling
B. Lochia serosa
C. Lochia alba
D. Lochia rubra
43. After expulsion of the placenta in a client who has six living children, an infusion of
lactated ringer’s solution with 10 units of pitocin is ordered. The nurse understands that this
is indicated for this client because:

A. She had a precipitate birth


B. This was an extramural birth
C. Retained placental fragments must be expelled
D. Multigravidas are at increased risk for uterine atony.
89
44. As part of the postpartum assessment, the nurse examines the breasts of a
primiparous breastfeeding woman who is one day postpartum.  An expected finding would
be:

A. Soft, non-tender; colostrum is present


B. Leakage of milk at let down
C. Swollen, warm, and tender upon palpation
D. A few blisters and a bruise on each areola
45. Following the birth of her baby, a woman expresses concern about the weight she
gained during pregnancy and how quickly she can lose it now that the baby is born.  The
nurse, in describing the expected pattern of weight loss, should begin by telling this woman
that:

A. Return to pre pregnant weight is usually achieved by the end of the postpartum period
B. Fluid loss from diuresis, diaphoresis, and bleeding accounts for about a 3 pound weight loss
C. The expected weight loss immediately after birth averages about 11 to 13 pounds
D. Lactation will inhibit weight loss since caloric intake must increase to support milk
production
46. Which of the following findings would be a source of concern if noted during the
assessment of a woman who is 12 hours postpartum?

A. Postural hypotension
B. Temperature of 100.4°F
C. Bradycardia — pulse rate of 55 BPM
D. Pain in left calf with dorsiflexion of left foot
47. The nurse examines a woman one hour after birth.  The woman’s fundus is boggy,
midline, and 1 cm below the umbilicus.  Her lochial flow is profuse, with two plum-sized
clots.  The nurse’s initial action would be to:

A. Place her on a bedpan to empty her bladder


B. Massage her fundus
C. Call the physician
D. Administer Methergine 0.2 mg IM which has been ordered prn
48. When performing a postpartum check, the nurse should:

A. Assist the woman into a lateral position with upper leg flexed forward to facilitate the
examination of her perineum
B. Assist the woman into a supine position with her arms above her head and her legs extended
for the examination of her abdomen
C. Instruct the woman to avoid urinating just before the examination since a full bladder will
facilitate fundal palpation
D. Wash hands and put on sterile gloves before beginning the check
49. Perineal care is an important infection control measure.  When evaluating a postpartum
woman’s perineal care technique, the nurse would recognize the need for further
instruction if the woman:

A. Uses soap and warm water to wash the vulva and perineum
B. Washes from symphysis pubis back to episiotomy
C. Changes her perineal pad every 2 – 3 hours
D. Uses the peribottle to rinse upward into her vagina

90
50. Which measure would be least effective in preventing postpartum hemorrhage?

A. Administer Methergine 0.2 mg every 6 hours for 4 doses as ordered


B. Encourage the woman to void every 2 hours
C. Massage the fundus every hour for the first 24 hours following birth
D. Teach the woman the importance of rest and nutrition to enhance healing

51. When making a visit to the home of a postpartum woman one week after birth, the
nurse should recognize that the woman would characteristically:

A. Express a strong need to review events and her behavior during the process of labor and
birth
B. Exhibit a reduced attention span, limiting readiness to learn
C. Vacillate between the desire to have her own nurturing needs met and the need to take
charge of her own care and that of her newborn
D. Have reestablished her role as a spouse/partner
52. Four hours after a difficult labor and birth, a primiparous woman refuses to feed her
baby, stating that she is too tired and just wants to sleep.  The nurse should:

A. Tell the woman she can rest after she feeds her baby
B. Recognize this as a behavior of the taking-hold stage
C. Record the behavior as ineffective maternal-newborn attachment
D. Take the baby back to the nursery, reassuring the woman that her rest is a priority at this
time
53. Parents can facilitate the adjustment of their other children to a new baby by:

A. 53-Having the children choose or make a gift to give to the new baby upon its arrival home
B. Emphasizing activities that keep the new baby and other children together
C. Having the mother carry the new baby into the home so she can show the other children the
new baby
D. Reducing stress on other children by limiting their involvement in the care of the new baby
54. A primiparous woman is in the taking-in stage of psychosocial recovery and adjustment
following birth.  The nurse, recognizing the needs of women during this stage, should:

A. Foster an active role in the baby’s care


B. Provide time for the mother to reflect on the events of and her behavior during childbirth
C. Recognize the woman’s limited attention span by giving her written materials to read when
she gets home rather than doing a teaching session now
D. Promote maternal independence by encouraging her to meet her own hygiene and comfort
needs
55. All of the following are important in the immediate care of the premature neonate. 
Which nursing activity should have the greatest priority?

A. Instillation of antibiotic in the eyes


B. Identification by bracelet and foot prints
C. Placement in a warm environment
D. Neurological assessment to determine gestational age
Answers and Rationales
1. Answer: B. Every 15 minutes during the first hour and then every 30 minutes for the
next two hours.
91
2. Answer: D. Increase hydration by encouraging oral fluids. The mother’s temperature
may be taken every 4 hours while she is awake. Temperatures up to 100.4 (38 C) in the first 24
hours after birth are often related to the dehydrating effects of labor. The most appropriate action is
to increase hydration by encouraging oral fluids, which should bring the temperature to a normal
reading. Although the nurse would document the findings, the most appropriate action would be to
increase the hydration.
3. Answer: B. Instruct the mother to request help when getting out of bed. Orthostatic
hypotension may be evident during the first 8 hours after birth. Feelings of faintness or dizziness are
signs that should caution the nurse to be aware of the client’s safety. The nurse should advise the
mother to get help the first few times the mother gets out of bed. Obtaining an H/H requires a
physicians order.
4. Answer: C. Ask the mother to urinate and empty her bladder. Before starting the fundal
assessment, the nurse should ask the mother to empty her bladder so that an accurate assessment can
be done. When the nurse is performing fundal assessment, the nurse asks the woman to lie flat on
her back with the knees flexed. Massaging the fundus is not appropriate unless the fundus is boggy
and soft, and then it should be massaged gently until firm.
5. Answer: B. Indicates the presence of infection. Lochia, the discharge present after birth, is
red for the first 1 to 3 days and gradually decreases in amount. Normal lochia has a fleshy odor. Foul
smelling or purulent lochia usually indicates infection, and these findings are not normal.
Encouraging the woman to drink fluids or increase ambulation is not an accurate nursing
intervention.
6. Answer: B. Notify the physician. Normally, one may find a few small clots in the first 1 to
2 days after birth from pooling of blood in the vagina. Clots larger than 1 cm are considered
abnormal. The cause of these clots, such as uterine atony or retained placental fragments, needs to be
determined and treated to prevent further blood loss. Although the findings would be documented,
the most appropriate action is to notify the physician.
7. Answer: D. Eight peripads per day. The normal amount of lochia may vary with the
individual but should never exceed 4 to 8 peripads per day. The average number of peripads is 6 per
day.
8. Answer: B. 3 days PP. After birth, the nurse should auscultate the woman’s abdomen in all
four quadrants to determine the return of bowel sounds. Normal bowel elimination usually returns 2
to 3 days PP. Surgery, anesthesia, and the use of narcotics and pain control agents also contribute to
the longer period of altered bowel function.
9. Answer: A and C. In the PP period, cervical healing occurs rapidly and cervical
involution occurs.After 1 week the muscle begins to regenerate and the cervix feels firm and the
external os is the width of a pencil. Although the vaginal mucosa heals and vaginal distention
decreases, it takes the entire PP period for complete involution to occur and muscle tone is never
restored to the pregravid state. The fundus begins to descent into the pelvic cavity after 24 hours, a
process known as involution. Despite blood loss that occurs during delivery of the baby, a transient
increase in cardiac output occurs. The increase in cardiac output, which persists about 48 hours after
childbirth, is probably caused by an increase in stroke volume because Bradycardia is often noted
during the PP period. Soon after childbirth, digestion begins to begin to be active and the new
mother is usually hungry because of the energy expended during labor.
10. Answer: C. Changes in vital signs. Because the woman has had epidural anesthesia and is
anesthetized, she cannot feel pain, pressure, or a tearing sensation. Changes in vitals indicate
hypovolemia in the anesthetized PP woman with vulvar hematoma. Heavy bruising may be
visualized, but vital sign changes indicate hematoma caused by blood collection in the perineal
tissues.
11. Answer: D. Prepare an ice pack for application to the area. Application of ice will
reduce swelling caused by hematoma formation in the vulvar area. The other options are not
interventions that are specific to the plan of care for a client with a small vulvar hematoma.
12. Answer: C. Prepare the client for surgery. The use of an epidural, prolonged second stage
labor and forceps delivery are predisposing factors for hematoma formation, and a collection of up
to 500 ml of blood can occur in the vaginal area. Although the other options may be implemented,
the immediate action would be to prepare the client for surgery to stop the bleeding.
13. Answer: B. An increase in the pulse from 88 to 102 BPM. During the 4th stage of labor,
the maternal blood pressure, pulse, and respiration should be checked every 15 minutes during the
92
first hour. A rising pulse is an early sign of excessive blood loss because the heart pumps faster to
compensate for reduced blood volume. The blood pressure will fall as the blood volume diminishes,
but a decreased blood pressure would not be the earliest sign of hemorrhage. A slight rise in
temperature is normal. The respiratory rate is increased slightly.
14. Answer: A. Massage the fundus until it is firm. If the uterus is not contracted firmly, the
first intervention is to massage the fundus until it is firm and to express clots that may have
accumulated in the uterus. Pushing on an uncontracted uterus can invert the uterus and cause
massive hemorrhage. Elevating the client’s legs and encouraging the client to void will not assist in
managing uterine atony. If the uterus does not remain contracted as a result of the uterine massage,
the problem may be distended bladder and the nurse should assist the mother to urinate, but this
would not be the initial action.
15. Answer: B. Enlarged, hardened veins. Thrombosis of the superficial veins is usually
accompanied by signs and symptoms of inflammation. These include swelling of the involved
extremity and redness, tenderness, and warmth.
16. Answer: D. “I need to stop breastfeeding until this condition resolves.” In most cases,
the mother can continue to breastfeed with both breasts. If the affected breast is too sore, the mother
can pump the breast gently. Regular emptying of the breast is important to prevent abscess
formation. Antibiotic therapy assists in resolving the mastitis within 24-48 hours. Additional
supportive measures include ice packs, breast supports, and analgesics.
17. Answer: C. Hematuria, ecchymosis, and epistaxis. The treatment for DVT is
anticoagulant therapy. The nurse assesses for bleeding, which is an adverse effect of anticoagulants.
This includes hematuria, ecchymosis, and epistaxis. Dysuria and vertigo are not associated
specifically with bleeding.
18. Answer: A. Assess for hypovolemia and notify the health care provider. Symptoms of
hypovolemia include cool, clammy, pale skin, sensations of anxiety or impending doom,
restlessness, and thirst. When these symptoms are present, the nurse should further assess for
hypovolemia and notify the health care provider.
19. Answer: C. Notify the physician. If the bleeding is excessive, the cause may be laceration
of the cervix or birth canal. Massaging the fundus if it is firm will not assist in controlling the
bleeding. Trendelenburg’s position is to be avoided because it may interfere with cardiac function.
20. Answer: C. Activated partial thromboplastin time. Anticoagulation therapy may be used
to prevent the extension of thrombus by delaying the clotting time of the blood. Activated partial
thromboplastin time should be monitored, and a heparin dose should be adjusted to maintain a
therapeutic level of 1.5 to 2.5 times the control. The prothrombin time and the INR are used to
monitor coagulation time when warfarin (Coumadin) is used.
21. Answer: B, D, and E. Mastitis are an infection of the lactating breast. Client instructions
include resting during the acute phase, maintaining a fluid intake of at least 3 L a day, and taking
analgesics to relieve discomfort. Antibiotics may be prescribed and are taken until the complete
prescribed course is finished. They are not stopped when the soreness subsides. Additional
supportive measures include the use of moist heat or ice packs and wearing a supportive bra.
Continued decompression of the breast by breastfeeding or pumping is important to empty the breast
and prevent formation of an abscess.
22. Answer: B. Blood pressure. Methergine and pitocin are agents that are used to prevent or
control postpartum hemorrhage by contracting the uterus. They cause continuous uterine
contractions and may elevate blood pressure. A priority nursing intervention is to check blood
pressure. The physician should be notified if hypertension is present.
23. Answer: A. Peripheral vascular disease. These medications are avoided in clients with
significant cardiovascular disease, peripheral disease, hypertension, eclampsia, or preeclampsia.
These conditions are worsened by the vasoconstriction effects of these medications.
24. Answer: A. Supplemental feedings with formula. Routine formula supplementation may
interfere with establishing an adequate milk volume because decreased stimulation to the mother’s
nipples affects hormonal levels and milk production.
25. Answer: C. Teaching how to express her breasts in a warm shower. Teaching the client
how to express her breasts in a warm shower aids with let-down and will give temporary relief. Ice
can promote comfort by vasoconstriction, numbing, and discouraging further letdown of milk.

93
26. Answer: A. Ask the client to empty her bladder. A full bladder may displace the uterine
fundus to the left or right side of the abdomen. Catheterization is unnecessary invasive if the woman
can void on her own.
27. Answer: C. Lower than before she became pregnant. PP insulin requirements are usually
significantly lower than pre pregnancy requirements. Occasionally, clients may require little to no
insulin during the first 24 to 48 hours postpartum.
28. Answer: A. Fundus 1 cm above the umbilicus 1 hour postpartum. Within the first 12
hours postpartum, the fundus usually is approximately 1 cm above the umbilicus. The fundus should
be below the umbilicus by PP day 3. The fundus shouldn’t be palpated in the abdomen after day 10.
29. Answer: C. Multiple gestation. Multiple gestation, breastfeeding, multiparity, and
conditions that cause overdistention of the uterus will increase the intensity of after-pains. Bottle-
feeding and diabetes aren’t directly associated with increasing severity of afterpains unless the client
has delivered a macrosomic infant.
30. Answer: B. Days 3 to 10 PP. On the third and fourth PP days, the lochia becomes a pale
pink or brown and contains old blood, serum, leukocytes, and tissue debris. This type of lochia
usually lasts until PP day 10. Lochia rubra usually last for the first 3 to 4 days PP. Lochia alba,
which contain leukocytes, decidua, epithelial cells, mucus, and bacteria, may continue for 2 to 6
weeks PP.
31. Answer: A. Passive and dependant. During the taking in phase, which usually lasts 1-3
days, the mother is passive and dependent and expresses her own needs rather than the neonate’s
needs. The taking hold phase usually lasts from days 3-10 PP. During this stage, the mother strives
for independence and autonomy; she also becomes curious and interested in the care of the baby and
is most ready to learn.
32. Answer: C. Cervical laceration. Continuous seepage of blood may be due to cervical or
vaginal lacerations if the uterus is firm and contracting. Retained placental fragments and uterine
atony may cause subinvolution of the uterus, making it soft, boggy, and larger than expected. UTI
won’t cause vaginal bleeding, although hematuria may be present.
33. Answer: D. Transitional milk. Transitional milk comes after colostrum and usually lasts
until 2 weeks PP.
34. Answer: D. Uterine subinvolution. Late postpartum bleeding is often the result of
subinvolution of the uterus. Retained products of conception or infection often cause subinvolution.
Cervical or perineal lacerations can cause an immediate postpartum hemorrhage. A client with a
clotting deficiency may also have an immediate PP hemorrhage if the deficiency isn’t corrected at
the time of delivery.
35. Answer: D. The client should avoid getting pregnant for 3 months after the vaccine
because the vaccine has teratogenic effects. The client must understand that she must not become
pregnant for 3 months after the vaccination because of its potential teratogenic effects. The rubella
vaccine is made from duck eggs so an allergic reaction may occur in clients with egg allergies. The
virus is not transmitted into the breast milk, so clients may continue to breastfeed after the
vaccination. Transient arthralgia and rash are common adverse effects of the vaccine.
36. Answer: B. Decrease. The placenta produces the hormone human placental lactogen, an
insulin antagonist. After birth, the placenta, the major source of insulin resistance, is gone. Insulin
needs decrease and women with type 1 diabetes may only need one-half to two-thirds of the prenatal
insulin during the first few PP days.
37. Answer: D. Mothers with diabetes may breastfeed; insulin requirements may decrease
from breastfeeding. Breastfeeding has an antidiabetogenic effect. Insulin needs are decreased
because carbohydrates are used in milk production. Breastfeeding mothers are at a higher risk of
hypoglycemia in the first PP days after birth because the glucose levels are lower. Mothers with
diabetes should be encouraged to breastfeed.
38. Answer: D. Taking-in phase. The taking-in phase occurs in the first 24 hours after birth.
The mother is concerned with her own needs and requires support from staff and relatives. The
taking-hold phase occurs when the mother is ready to take responsibility for her care as well as the
infants care. The letting-go phase begins several weeks later, when the mother incorporates the new
infant into the family unit.
39. Answer: B. Rapid diuresis. In the early PP period, there’s an increase in the glomerular
filtration rate and a drop in the progesterone levels, which result in rapid diuresis. There should be

94
no urinary urgency, though a woman may feel anxious about voiding. There’s a minimal change in
blood pressure following childbirth, and a residual decrease in GI motility.
40. Answer: A. The client appears interested in learning about neonatal care. The third to
tenth days of PP care are the “taking-hold” phase, in which the new mother strives for independence
and is eager for her neonate. The other options describe the phase in which the mother relives her
birth experience.
41. Answer: C. Urine retention. Urine retention causes a distended bladder to displace the
uterus above the umbilicus and to the side, which prevents the uterus from contracting. The uterus
needs to remain contracted if bleeding is to stay within normal limits. Cervical and vaginal tears can
cause PP hemorrhage but are less common occurrences in the PP period.
42. Answer: D. Lochia rubra
43. Answer: D. Multigravidas are at increased risk for uterine atony. Multiple full-term
pregnancies and deliveries result in overstretched uterine muscles that do not contract efficiently and
bleeding may ensue.
44. Answer: A. Soft, non-tender; colostrum is present. Breasts are essentially unchanged for
the first two to three days after birth. Colostrum is present and may leak from the nipples.
45. Answer: C. The expected weight loss immediately after birth averages about 11 to 13
pounds. Prepregnant weight is usually achieved by 2 to 3 months after birth, not within the 6-week
postpartum period. Weight loss from diuresis, diaphoresis, and bleeding is about 9 pounds. Weight
loss continues during breastfeeding since fat stores developed during pregnancy and extra calories
consumed are used as part of the lactation process.
46. Answer: D. Pain in left calf with dorsiflexion of left foot. Responses A and C are
expected related to circulatory changes after birth.  A temperature of 100.4°F in the first 24 hours is
most likely indicative of dehydration which is easily corrected by increasing oral fluid intake.  The
findings in response D indicate a positive Homan sign and are suggestive of thrombophlebitis and
should be investigated further.
47. Answer: B. Massage her fundus. A boggy or soft fundus indicates that uterine atony is
present. This is confirmed by the profuse lochia and passage of clots. The first action would be to
massage the fundus until firm, followed by 3 and 4, especially if the fundus does not become or
remain firm with massage. There is no indication of a distended bladder since the fundus is midline
and below the umbilicus.
48. Answer: A. Assist the woman into a lateral position with upper leg flexed forward to
facilitate the examination of her perineum. While the supine position is best for examining the
abdomen, the woman should keep her arms at her sides and slightly flex her knees in order to relax
abdominal muscles and facilitate palpation of the fundus. The bladder should be emptied before the
check. A full bladder alters the position of the fundus and makes the findings inaccurate. Although
hands are washed before starting the check, clean (not sterile) gloves are put on just before the
perineum and pad are assessed to protect from contact with blood and secretions.
49. Answer: D. Uses the peribottle to rinse upward into her vagina.  Responses A, B, and
C are all appropriate measures. The peribottle should be used in a backward direction over the
perineum. The flow should never be directed upward into the vagina since debris would be forced
upward into the uterus through the still-open cervix.
50. Answer: C. Massage the fundus every hour for the first 24 hours following birth. The
fundus should be massaged only when boggy or soft.  Massaging a firm fundus could cause it to
relax.  Responses A, B, and D are all effective measures to enhance and maintain contraction of the
uterus and to facilitate healing.
51. Answer: C. Vacillate between the desire to have her own nurturing needs met and the
need to take charge of her own care and that of her newborn. One week after birth the woman
should exhibit behaviors characteristic of the taking-hold stage as described in response C. This
stage lasts for as long as 4 to 5 weeks after birth. Responses A and B are characteristic of the taking-
in stage, which lasts for the first few days after birth. Response D reflects the letting-go stage, which
indicates that psychosocial recovery is complete.
52. Answer: D. Take the baby back to the nursery, reassuring the woman that her rest is a
priority at this time. Response A does not take into consideration the need for the new mother to be
nurtured and have her needs met during the taking-in stage.  The behavior described is typical of this
stage and not a reflection of ineffective attachment unless the behavior persists.  Mothers need to
reestablish their own well-being in order to effectively care for their baby.
95
53. Answer: A. Having the children choose or make a gift to give to the new baby upon its
arrival home. Special time should be set aside just for the other children without interruption from
the newborn.  Someone other than the mother should carry the baby into the home so she can give
full attention to greeting her other children.  Children should be actively involved in the care of the
baby according to their ability without overwhelming them.
54. Answer: B. Provide time for the mother to reflect on the events of and her behavior
during childbirth. The focus of the taking-in stage is nurturing the new mother by meeting her
dependency needs for rest, comfort, hygiene, and nutrition.  Once they are met, she is more able to
take an active role, not only in her own care but also the care of her newborn.  Women express a
need to review their childbirth experience and evaluate their performance.  Short teaching sessions,
using written materials to reinforce the content presented, are a more effective approach.
55. Answer: C. Placement in a warm environment

1. A nurse is caring for a client in labor. The nurse determines that the client is beginning in
the 2nd stage of labor when which of the following assessments is noted?

A. The client begins to expel clear vaginal fluid


B. The contractions are regular
C. The membranes have ruptured
D. The cervix is dilated completely
2. A nurse in the labor room is caring for a client in the active phases of labor. The nurse is
assessing the fetal patterns and notes a late deceleration on the monitor strip. The most
appropriate nursing action is to:

A. Place the mother in the supine position


B. Document the findings and continue to monitor the fetal patterns
C. Administer oxygen via face mask
D. Increase the rate of pitocin IV infusion
3. A nurse is performing an assessment of a client who is scheduled for a cesarean
delivery. Which assessment finding would indicate a need to contact the physician?

A. Fetal heart rate of 180 beats per minute


B. White blood cell count of 12,000
C. Maternal pulse rate of 85 beats per minute
D. Hemoglobin of 11.0 g/dL
4.  A client in labor is transported to the delivery room and is prepared for a cesarean
delivery. The client is transferred to the delivery room table, and the nurse places the client
in the:

A. Trendelenburg’s position with the legs in stirrups


B. Semi-Fowler position with a pillow under the knees
C. Prone position with the legs separated and elevated
D. Supine position with a wedge under the right hip
5. A nurse is caring for a client in labor and prepares to auscultate the fetal heart rate by
using a Doppler ultrasound device. The nurse most accurately determines that the fetal
heart sounds are heard by:

A. Noting if the heart rate is greater than 140 BPM


B. Placing the diaphragm of the Doppler on the mother abdomen
C. Performing Leopold’s maneuvers first to determine the location of the fetal heart
D. Palpating the maternal radial pulse while listening to the fetal heart rate

96
6. A nurse is caring for a client in labor who is receiving Pitocin by IV infusion to stimulate
uterine contractions. Which assessment finding would indicate to the nurse that the infusion
needs to be discontinued?

A. Three contractions occurring within a 10-minute period


B. A fetal heart rate of 90 beats per minute
C. Adequate resting tone of the uterus palpated between contractions
D. Increased urinary output
7. A nurse is beginning to care for a client in labor. The physician has prescribed an IV
infusion of Pitocin. The nurse ensures that which of the following is implemented before
initiating the infusion?

A. Placing the client on complete bed rest


B. Continuous electronic fetal monitoring
C. An IV infusion of antibiotics
D. Placing a code cart at the client’s bedside
8. A nurse is monitoring a client in active labor and notes that the client is having
contractions every 3 minutes that last 45 seconds. The nurse notes that the fetal heart rate
between contractions is 100 BPM. Which of the following nursing actions is most
appropriate?

A. Encourage the client’s coach to continue to encourage breathing exercises


B. Encourage the client to continue pushing with each contraction
C. Continue monitoring the fetal heart rate
D. Notify the physician or nurse midwife
9. A nurse is caring for a client in labor and is monitoring the fetal heart rate patterns. The
nurse notes the presence of episodic accelerations on the electronic fetal monitor tracing.
Which of the following actions is most appropriate?

A. Document the findings and tell the mother that the monitor indicates fetal well-being
B. Take the mother’s vital signs and tell the mother that bed rest is required to conserve
oxygen.
C. Notify the physician or nurse midwife of the findings.
D. Reposition the mother and check the monitor for changes in the fetal tracing
10. A nurse is admitting a pregnant client to the labor room and attaches an external
electronic fetal monitor to the client’s abdomen. After attachment of the monitor, the initial
nursing assessment is which of the following?

A. Identifying the types of accelerations


B. Assessing the baseline fetal heart rate
C. Determining the frequency of the contractions
D. Determining the intensity of the contractions
11. A nurse is reviewing the record of a client in the labor room and notes that the nurse
midwife has documented that the fetus is at (-1) station. The nurse determines that the fetal
presenting part is:

A. 1 cm above the ischial spine


B. 1 fingerbreadth below the symphysis pubis
C. 1 inch below the coccyx
97
D. 1 inch below the iliac crest
12. A pregnant client is admitted to the labor room. An assessment is performed, and the
nurse notes that the client’s hemoglobin and hematocrit levels are low, indicating anemia.
The nurse determines that the client is at risk for which of the following?

A. A loud mouth
B. Low self-esteem
C. Hemorrhage
D. Postpartum infections
13. A nurse assists in the vaginal delivery of a newborn infant. After the delivery, the nurse
observes the umbilical cord lengthen and a spurt of blood from the vagina. The nurse
documents these observations as signs of:

A. Hematoma
B. Placenta previa
C. Uterine atony
D. Placental separation
14. A client arrives at a birthing center in active labor. Her membranes are still intact, and
the nurse-midwife prepares to perform an amniotomy. A nurse who is assisting the nurse-
midwife explains to the client that after this procedure, she will most likely have:

A. Less pressure on her cervix


B. Increased efficiency of contractions
C. Decreased number of contractions
D. The need for increased maternal blood pressure monitoring
15. A nurse is monitoring a client in labor. The nurse suspects umbilical cord compression
if which of the following is noted on the external monitor tracing during a contraction?

A. Early decelerations
B. Variable decelerations
C. Late decelerations
D. Short-term variability
16. A nurse explains the purpose of effleurage to a client in early labor. The nurse tells the
client that effleurage is:

A. A form of biofeedback to enhance bearing down efforts during delivery


B. Light stroking of the abdomen to facilitate relaxation during labor and provide tactile
stimulation to the fetus
C. The application of pressure to the sacrum to relieve a backache
D. Performed to stimulate uterine activity by contracting a specific muscle group while other
parts of the body rest
17. A nurse is caring for a client in the second stage of labor. The client is experiencing
uterine contractions every 2 minutes and cries out in pain with each contraction. The nurse
recognizes this behavior as:

A. Exhaustion
B. Fear of losing control
C. Involuntary grunting
D. Valsalva’s maneuver

98
18. A nurse is monitoring a client in labor who is receiving Pitocin and notes that the client
is experiencing hypertonic uterine contractions. List in order of priority the actions that the
nurse takes.

A. Stop of Pitocin infusion


B. Perform a vaginal examination
C. Reposition the client
D. Check the client’s blood pressure and heart rate
E. Administer oxygen by face mask at 8 to 10 L/min
19. A nurse is assigned to care for a client with hypotonic uterine dysfunction and signs of a
slowing labor. The nurse is reviewing the physician’s orders and would expect to note
which of the following prescribed treatments for this condition?

A. Medication that will provide sedation


B. Increased hydration
C. Oxytocin (Pitocin) infusion
D. Administration of a tocolytic medication
20. A nurse in the labor room is preparing to care for a client with hypertonic uterine
dysfunction. The nurse is told that the client is experiencing uncoordinated contractions that
are erratic in their frequency, duration, and intensity. The priority nursing intervention would
be to:

A. Monitor the Pitocin infusion closely


B. Provide pain relief measures
C. Prepare the client for an amniotomy
D. Promote ambulation every 30 minutes
21. A nurse is developing a plan of care for a client experiencing dystocia and includes
several nursing interventions in the plan of care. The nurse prioritizes the plan of care and
selects which of the following nursing interventions as the highest priority?

A. Keeping the significant other informed of the progress of the labor


B. Providing comfort measures
C. Monitoring fetal heart rate
D. Changing the client’s position frequently
22. A maternity nurse is preparing to care for a pregnant client in labor who will be
delivering twins. The nurse monitors the fetal heart rates by placing the external fetal
monitor:

A. Over the fetus that is most anterior to the mother’s abdomen


B. Over the fetus that is most posterior to the mother’s abdomen
C. So that each fetal heart rate is monitored separately
D. So that one fetus is monitored for a 15-minute period followed by a 15 minute fetal
monitoring period for the second fetus
23. A nurse in the postpartum unit is caring for a client who has just delivered a newborn
infant following a pregnancy with placenta previa. The nurse reviews the plan of care and
prepares to monitor the client for which of the following risks associated with placenta
previa?

A. Disseminated intravascular coagulation


99
B. Chronic hypertension
C. Infection
D. Hemorrhage
24. A nurse in the delivery room is assisting with the delivery of a newborn infant. After the
delivery of the newborn, the nurse assists in delivering the placenta. Which observation
would indicate that the placenta has separated from the uterine wall and is ready for
delivery?

A. The umbilical cord shortens in length and changes in color


B. A soft and boggy uterus
C. Maternal complaints of severe uterine cramping
D. Changes in the shape of the uterus
25. A nurse in the labor room is performing a vaginal assessment on a pregnant client in
labor. The nurse notes the presence of the umbilical cord protruding from the vagina.
Which of the following would be the initial nursing action?

A. Place the client in Trendelenburg’s position


B. Call the delivery room to notify the staff that the client will be transported immediately
C. Gently push the cord into the vagina
D. Find the closest telephone and stat page the physician
26. A maternity nurse is caring for a client with abruptio placenta and is monitoring the
client for disseminated intravascular coagulopathy. Which assessment finding is least likely
to be associated with disseminated intravascular coagulation?

A. Swelling of the calf in one leg


B. Prolonged clotting times
C. Decreased platelet count
D. Petechiae, oozing from injection sites, and hematuria
27. A nurse is assessing a pregnant client in the 2nd trimester of pregnancy who was
admitted to the maternity unit with a suspected diagnosis of abruptio placentae. Which of
the following assessment findings would the nurse expect to note if this condition is
present?

A. Absence of abdominal pain


B. A soft abdomen
C. Uterine tenderness/pain
D. Painless, bright red vaginal bleeding
28. A maternity nurse is preparing for the admission of a client in the 3rd trimester of
pregnancy that is experiencing vaginal bleeding and has a suspected diagnosis of placenta
previa. The nurse reviews the physician’s orders and would question which order?

A. Prepare the client for an ultrasound


B. Obtain equipment for external electronic fetal heart monitoring
C. Obtain equipment for a manual pelvic examination
D. Prepare to draw a Hgb and Hct blood sample
29. An ultrasound is performed on a client at term gestation that is experiencing moderate
vaginal bleeding. The results of the ultrasound indicate that an abruptio placenta is present.
Based on these findings, the nurse would prepare the client for:

10
0
A. Complete bed rest for the remainder of the pregnancy
B. Delivery of the fetus
C. Strict monitoring of intake and output
D. The need for weekly monitoring of coagulation studies until the time of delivery
30. A nurse in a labor room is assisting with the vaginal delivery of a newborn infant. The
nurse would monitor the client closely for the risk of uterine rupture if which of the following
occurred?

A. Hypotonic contractions
B. Forceps delivery
C. Schultz delivery
D. Weak bearing down efforts
31. A client is admitted to the birthing suite in early active labor. The priority nursing
intervention on admission of this client would be:

A. Auscultating the fetal heart


B. Taking an obstetric history
C. Asking the client when she last ate
D. Ascertaining whether the membranes were ruptured
32. A client who is gravida 1, para 0 is admitted in labor. Her cervix is 100% effaced, and
she is dilated to 3 cm. Her fetus is at +1 station. The nurse is aware that the fetus’ head is:

A. Not yet engaged


B. Entering the pelvic inlet
C. Below the ischial spines
D. Visible at the vaginal opening
33. After doing Leopold’s maneuvers, the nurse determines that the fetus is in the ROP
position. To best auscultate the fetal heart tones, the Doppler is placed:

A. Above the umbilicus at the midline


B. Above the umbilicus on the left side
C. Below the umbilicus on the right side
D. Below the umbilicus near the left groin
34. The physician asks the nurse the frequency of a laboring client’s contractions. The
nurse assesses the client’s contractions by timing from the beginning of one contraction:

A. Until the time it is completely over


B. To the end of a second contraction
C. To the beginning of the next contraction
D. Until the time that the uterus becomes very firm
35. The nurse observes the client’s amniotic fluid and decides that it appears normal,
because it is:

A. Clear and dark amber in color


B. Milky, greenish yellow, containing shreds of mucus
C. Clear, almost colorless, and containing little white specks
D. Cloudy, greenish-yellow, and containing little white specks
36. At 38 weeks gestation, a client is having late decelerations. The fetal pulse oximeter
shows 75% to 85%. The nurse should:

A. Discontinue the catheter, if the reading is not above 80%


10
1
B. Discontinue the catheter, if the reading does not go below 30%
C. Advance the catheter until the reading is above 90% and continue monitoring
D. Reposition the catheter, recheck the reading, and if it is 55%, keep monitoring
37. When examining the fetal monitor strip after rupture of the membranes in a laboring
client, the nurse notes variable decelerations in the fetal heart rate. The nurse should:

A. Stop the oxytocin infusion


B. Change the client’s position
C. Prepare for immediate delivery
D. Take the client’s blood pressure
38. When monitoring the fetal heart rate of a client in labor, the nurse identifies an elevation
of 15 beats above the baseline rate of 135 beats per minute lasting for 15 seconds. This
should be documented as:

A. An acceleration
B. An early elevation
C. A sonographic motion
D. A tachycardic heart rate
39. A laboring client complains of low back pain. The nurse replies that this pain occurs
most when the position of the fetus is:

A. Breech
B. Transverse
C. Occiput anterior
D. Occiput posterior
40. The breathing technique that the mother should be instructed to use as the fetus’ head
is crowning is:

A. Blowing
B. Slow chest
C. Shallow
D. Accelerated-decelerated
41. During the period of induction of labor, a client should be observed carefully for signs
of:

A. Severe pain
B. Uterine tetany
C. Hypoglycemia
D. Umbilical cord prolapse
42. A client arrives at the hospital in the second stage of labor. The fetus’ head is crowning,
the client is bearing down, and the birth appears imminent. The nurse should:

A. Transfer her immediately by stretcher to the birthing unit


B. Tell her to breathe through her mouth and not to bear down
C. Instruct the client to pant during contractions and to breathe through her mouth
D. Support the perineum with the hand to prevent tearing and tell the client to pant
43. A laboring client is to have a pudendal block. The nurse plans to tell the client that once
the block is working she:

A. Will not feel the episiotomy


B. May lose bladder sensation
10
2
C. May lose the ability to push
D. Will no longer feel contractions
44. Which of the following observations indicates fetal distress?

A. Fetal scalp pH of 7.14


B. Fetal heart rate of 144 beats/minute
C. Acceleration of fetal heart rate with contractions
D. Presence of long term variability
45. Which of the following fetal positions is most favorable for birth?

A. Vertex presentation
B. Transverse lie
C. Frank breech presentation
D. Posterior position of the fetal head
46.  A laboring client has external electronic fetal monitoring in place. Which of the
following assessment data can be determined by examining the fetal heart rate strip
produced by the external electronic fetal monitor?

A. Gender of the fetus


B. Fetal position
C. Labor progress
D. Oxygenation
47. A laboring client is in the first stage of labor and has progressed from 4 to 7 cm in
cervical dilation. In which of the following phases of the first stage does cervical dilation
occur most rapidly?

A. Preparatory phase
B. Latent phase
C. Active phase
D. Transition phase
48. A multiparous client who has been in labor for 2 hours states that she feels the urge to
move her bowels. How should the nurse respond?

A. Let the client get up to use the potty


B. Allow the client to use a bedpan
C. Perform a pelvic examination
D. Check the fetal heart rate
49. Labor is a series of events affected by the coordination of the five essential factors.
One of these is the passenger (fetus). Which are the other four factors?

A. Contractions, passageway, placental position and function, pattern of care


B. Contractions, maternal response, placental position, psychological response
C. Passageway, contractions, placental position and function, psychological response
D. Passageway, placental position and function, paternal response, psychological response
50. Fetal presentation refers to which of the following descriptions?

A. Fetal body part that enters the maternal pelvis first


B. Relationship of the presenting part to the maternal pelvis
C. Relationship of the long axis of the fetus to the long axis of the mother
D. A classification according to the fetal part

10
3
51. A client is admitted to the L & D suite at 36 weeks’ gestation. She has a history of C-
section and complains of severe abdominal pain that started less than 1 hour earlier. When
the nurse palpates tetanic contractions, the client again complains of severe pain. After the
client vomits, she states that the pain is better and then passes out. Which is the probable
cause of her signs and symptoms?

A. Hysteria compounded by the flu


B. Placental abruption
C. Uterine rupture
D. Dysfunctional labor
52. Upon completion of a vaginal examination on a laboring woman, the nurse records:
50%, 6 cm, -1. Which of the following is a correct interpretation of the data?

A. Fetal presenting part is 1 cm above the ischial spines


B. Effacement is 4 cm from completion
C. Dilation is 50% completed
D. Fetus has achieved passage through the ischial spines
53. Which of the following findings meets the criteria of a reassuring FHR pattern?

A. FHR does not change as a result of fetal activity


B. Average baseline rate ranges between 100 – 140 BPM
C. Mild late deceleration patterns occur with some contractions
D. Variability averages between 6 – 10 BPM
54. Late deceleration patterns are noted when assessing the monitor tracing of a woman
whose labor is being induced with an infusion of Pitocin.  The woman is in a side-lying
position, and her vital signs are stable and fall within a normal range.  Contractions are
intense, last 90 seconds, and occur every 1 1/2 to 2 minutes. The nurse’s immediate action
would be to:

A. Change the woman’s position


B. Stop the Pitocin
C. Elevate the woman’s legs
D. Administer oxygen via a tight mask at 8 to 10 liters/minute
55. The nurse should realize that the most common and potentially harmful maternal
complication of epidural anesthesia would be:

A. Severe postpartum headache


B. Limited perception of bladder fullness
C. Increase in respiratory rate
D. Hypotension
56. Perineal care is an important infection control measure.  When evaluating a postpartum
woman’s perineal care technique, the nurse would recognize the need for further
instruction if the woman:

A. Uses soap and warm water to wash the vulva and perineum
B. Washes from symphysis pubis back to episiotomy
C. Changes her perineal pad every 2 – 3 hours
D. Uses the peribottle to rinse upward into her vagina
57. Which measure would be least effective in preventing postpartum hemorrhage?
10
4
A. Administer Methergine 0.2 mg every 6 hours for 4 doses as ordered
B. Encourage the woman to void every 2 hours
C. Massage the fundus every hour for the first 24 hours following birth
D. Teach the woman the importance of rest and nutrition to enhance healing
58. When making a visit to the home of a postpartum woman one week after birth, the
nurse should recognize that the woman would characteristically:

A. Express a strong need to review events and her behavior during the process of labor and
birth
B. Exhibit a reduced attention span, limiting readiness to learn
C. Vacillate between the desire to have her own nurturing needs met and the need to take
charge of her own care and that of her newborn
D. Have reestablished her role as a spouse/partner
59. Four hours after a difficult labor and birth, a primiparous woman refuses to feed her
baby, stating that she is too tired and just wants to sleep.  The nurse should:

A. Tell the woman she can rest after she feeds her baby
B. Recognize this as a behavior of the taking-hold stage
C. Record the behavior as ineffective maternal-newborn attachment
D. Take the baby back to the nursery, reassuring the woman that her rest is a priority at this
time
60. Parents can facilitate the adjustment of their other children to a new baby by:

A. Having the children choose or make a gift to give to the new baby upon its arrival home
B. Emphasizing activities that keep the new baby and other children together
C. Having the mother carry the new baby into the home so she can show the other children the
new baby
D. Reducing stress on other children by limiting their involvement in the care of the new baby
Answers and Rationales
1. Answer: D. The cervix is dilated completely. The second stage of labor begins when the
cervix is dilated completely and ends with the birth of the neonate.
2. Answer: C. Administer oxygen via face mask. Late decelerations are due to uteroplacental
insufficiency as the result of decreased blood flow and oxygen to the fetus during the uterine
contractions. This causes hypoxemia; therefore oxygen is necessary. The supine position is avoided
because it decreases uterine blood flow to the fetus. The client should be turned to her side to
displace pressure of the gravid uterus on the inferior vena cava. An intravenous pitocin infusion is
discontinued when a late deceleration is noted.
3. Answer: A. Fetal heart rate of 180 beats per minute. A normal fetal heart rate is 120-160
beats per minute. A count of 180 beats per minute could indicate fetal distress and would warrant
physician notification. By full term, a normal maternal hemoglobin range is 11-13 g/dL as a result of
the hemodilution caused by an increase in plasma volume during pregnancy.
4. Answer: D. Supine position with a wedge under the right hip. Vena cava and descending
aorta compression by the pregnant uterus impedes blood return from the lower trunk and extremities.
This leads to decreasing cardiac return, cardiac output, and blood flow to the uterus and the fetus.
The best position to prevent this would be side-lying with the uterus displaced off of abdominal
vessels. Positioning for abdominal surgery necessitates a supine position; however, a wedge placed
under the right hip provides displacement of the uterus.
5. Answer: D. Palpating the maternal radial pulse while listening to the fetal heart
rate. The nurse simultaneously should palpate the maternal radial or carotid pulse and auscultate the
fetal heart rate to differentiate the two. If the fetal and maternal heart rates are similar, the nurse may
mistake the maternal heart rate for the fetal heart rate. Leopold’s maneuvers may help the examiner
locate the position of the fetus but will not ensure a distinction between the two rates.
6. Answer: B. A fetal heart rate of 90 beats per minute. A normal fetal heart rate is 120-160
BPM. Bradycardia or late or variable decelerations indicate fetal distress and the need to discontinue
10
5
to pitocin. The goal of labor augmentation is to achieve three good-quality contractions in a 10-
minute period.
7. Answer: B. Continuous electronic fetal monitoring. Continuous electronic fetal
monitoring should be implemented during an IV infusion of Pitocin.
8. Answer: D. Notify the physician or nurse midwife. A normal fetal heart rate is 120-160
beats per minute. Fetal bradycardia between contractions may indicate the need for immediate
medical management, and the physician or nurse midwife needs to be notified.
9. Answer: A. Document the findings and tell the mother that the monitor indicates fetal
well-being.  Accelerations are transient increases in the fetal heart rate that often accompany
contractions or are caused by fetal movement. Episodic accelerations are thought to be a sign of
fetal-well being and adequate oxygen reserve.
10. Answer: B. Assessing the baseline fetal heart rate. Assessing the baseline fetal heart rate
is important so that abnormal variations of the baseline rate will be identified if they occur.
Identifying the types of accelerations and determining the frequency of the contractions are
important to assess, but not as the first priority.
11. Answer: A. 1 cm above the ischial spine. Station is the relationship of the presenting part
to an imaginary line drawn between the ischial spines, is measured in centimeters, and is noted as a
negative number above the line and a positive number below the line. At -1 station, the fetal
presenting part is 1 cm above the ischial spines.
12. Answer: D. Postpartum infections. Anemic women have a greater likelihood of cardiac
decompensation during labor, postpartum infection, and poor wound healing. Anemia does not
specifically present a risk for hemorrhage.
13. Answer:  D. Placental separation. As the placenta separates, it settles downward into the
lower uterine segment. The umbilical cord lengthens, and a sudden trickle or spurt of blood appears.
14. Answer: B. Increased efficiency of contractions. Amniotomy can be used to induce labor
when the condition of the cervix is favorable (ripe) or to augment labor if the process begins to slow.
Rupturing of membranes allows the fetal head to contact the cervix more directly and may increase
the efficiency of contractions.
15. Answer:  B. Variable decelerations. Variable decelerations occur if the umbilical cord
becomes compressed, thus reducing blood flow between the placenta and the fetus. Early
decelerations result from pressure on the fetal head during a contraction. Late decelerations are an
ominous pattern in labor because it suggests uteroplacental insufficiency during a contraction. Short-
term variability refers to the beat-to-beat range in the fetal heart rate.
16. Answer:  B. Light stroking of the abdomen to facilitate relaxation during labor and
provide tactile stimulation to the fetus. Effleurage is a specific type of cutaneous stimulation
involving light stroking of the abdomen and is used before transition to promote relaxation and
relieve mild to moderate pain. Effleurage provides tactile stimulation to the fetus.
17. Answer: B. Fear of losing control. Pains, helplessness, panicking, and fear of losing
control are possible behaviors in the 2nd stage of labor.
18. Answer: A, D, B. E, C. If uterine hypertonicity occurs, the nurse immediately would
intervene to reduce uterine activity and increase fetal oxygenation. The nurse would stop the Pitocin
infusion and increase the rate of the nonadditive solution, check maternal BP for hyper or
hypotension, position the woman in a side-lying position, and administer oxygen by snug face mask
at 8-10 L/min. The nurse then would attempt to determine the cause of the uterine hypertonicity and
perform a vaginal exam to check for prolapsed cord.
19. Answer: C. Oxytocin (Pitocin) infusion. Therapeutic management for hypotonic uterine
dysfunction includes oxytocin augmentation and amniotomy to stimulate a labor that slows.
20. Answer: B. Provide pain relief measures. Management of hypertonic labor depends on the
cause. Relief of pain is the primary intervention to promote a normal labor pattern.
21. Answer:  C. Monitoring fetal heart rate. The priority is to monitor the fetal heart rate.
22. Answer: C. So that each fetal heart rate is monitored separately. In a client with a multi-
fetal pregnancy, each fetal heart rate is monitored separately.
23. Answer: D. Hemorrhage. Because the placenta is implanted in the lower uterine segment,
which does not contain the same intertwining musculature as the fundus of the uterus, this site is
more prone to bleeding.
24. Answer: D. Changes in the shape of the uterus. Signs of placental separation include
lengthening of the umbilical cord, a sudden gush of dark blood from the introitus (vagina), a firmly
10
6
contracted uterus, and the uterus changing from a discoid (like a disk) to a globular (like a globe)
shape. The client may experience vaginal fullness, but not severe uterine cramping.
25. Answer: A. Place the client in Trendelenburg’s position. When cord prolapse occurs,
prompt actions are taken to relieve cord compression and increase fetal oxygenation. The mother
should be positioned with the hips higher than the head to shift the fetal presenting part toward the
diaphragm. The nurse should push the call light to summon help, and other staff members should
call the physician and notify the delivery room. No attempt should be made to replace the cord. The
examiner, however, may place a gloved hand into the vagina and hold the presenting part off of the
umbilical cord. Oxygen at 8 to 10 L/min by face mask is delivered to the mother to increase fetal
oxygenation.
26. Answer: A. Swelling of the calf in one leg. DIC is a state of diffuse clotting in which
clotting factors are consumed, leading to widespread bleeding. Platelets are decreased because they
are consumed by the process; coagulation studies show no clot formation (and are thus normal to
prolonged); and fibrin plugs may clog the microvasculature diffusely, rather than in an isolated area.
The presence of petechiae, oozing from injection sites, and hematuria are signs associated with DIC.
Swelling and pain in the calf of one leg are more likely to be associated with thrombophlebitis.
27. Answer: C. Uterine tenderness/pain. In abruptio placentae, acute abdominal pain is
present. Uterine tenderness and pain accompanies placental abruption, especially with a central
abruption and trapped blood behind the placenta. The abdomen will feel hard and boardlike on
palpation as the blood penetrates the myometrium and causes uterine irritability. Observation of the
fetal monitoring often reveals increased uterine resting tone, caused by failure of the uterus to relax
in attempt to constrict blood vessels and control bleeding.
28. Answer: C. Obtain equipment for a manual pelvic examination. Manual pelvic
examinations are contraindicated when vaginal bleeding is apparent in the 3rd trimester until a
diagnosis is made and placental previa is ruled out. Digital examination of the cervix can lead to
maternal and fetal hemorrhage. A diagnosis of placenta previa is made by ultrasound. The H/H
levels are monitored, and external electronic fetal heart rate monitoring is initiated. External fetal
monitoring is crucial in evaluating the fetus that is at risk for severe hypoxia.
29. Answer: B. Delivery of the fetus. The goal of management in abruptio placentae is to
control the hemorrhage and deliver the fetus as soon as possible. Delivery is the treatment of choice
if the fetus is at term gestation or if the bleeding is moderate to severe and the mother or fetus is in
jeopardy.
30. Answer: B. Forceps delivery. Excessive fundal pressure, forceps delivery, violent bearing
down efforts, tumultuous labor, and shoulder dystocia can place a woman at risk for traumatic
uterine rupture. Hypotonic contractions and weak bearing down efforts do not alone add to the risk
of rupture because they do not add to the stress on the uterine wall.
31. Answer: A. Auscultating the fetal heart. Determining the fetal well-being supersedes all
other measures. If the FHR is absent or persistently decelerating, immediate intervention is required.
32. Answer: C. Below the ischial spines. A station of +1 indicates that the fetal head is 1 cm
below the ischial spines.
33. Answer: C. Below the umbilicus on the right side. Fetal heart tones are best auscultated
through the fetal back; because the position is ROP (right occiput presenting), the back would be
below the umbilicus and on the right side.
34. Answer: C. To the beginning of the next contraction.  This is the way to determine the
frequency of the contractions
35. Answer: C. Clear, almost colorless, and containing little white specks.  By 36 weeks’
gestation, normal amniotic fluid is colorless with small particles of vernix caseosa present.
36. Answer: D. Reposition the catheter, recheck the reading, and if it is 55%, keep
monitoring. Adjusting the catheter would be indicated. Normal fetal pulse oximetry should be
between 30% and 70%. 75% to 85% would indicate maternal readings.
37. Answer: B. Change the client’s position. Variable decelerations usually are seen as a
result of cord compression; a change of position will relieve pressure on the cord.
38. Answer: A. An acceleration. An acceleration is an abrupt elevation above the baseline of
15 beats per minute for 15 seconds; if the acceleration persists for more than 10 minutes it is
considered a change in baseline rate. A tachycardic FHR is above 160 beats per minute.

10
7
39. Answer: D. Occiput posterior. A persistent occiput-posterior position causes intense back
pain because of fetal compression of the sacral nerves. Occiput anterior is the most common fetal
position and does not cause back pain.
40. Answer: A. Blowing. Blowing forcefully through the mouth controls the strong urge to
push and allows for a more controlled birth of the head.
41. Answer: B. Uterine tetany. Uterine tetany could result from the use of oxytocin to induce
labor. Because oxytocin promotes powerful uterine contractions, uterine tetany may occur. The
oxytocin infusion must be stopped to prevent uterine rupture and fetal compromise.
42. Answer: D. Support the perineum with the hand to prevent tearing and tell the client
to pant. Gentle pressure is applied to the baby’s head as it emerges so it is not born too rapidly. The
head is never held back, and it should be supported as it emerges so there will be no vaginal
lacerations. It is impossible to push and pant at the same time.
43. Answer: A. Will not feel the episiotomy. A pudendal block provides anesthesia to the
perineum.
44. Answer: A. Fetal scalp pH of 7.14. A fetal scalp pH below 7.25 indicates acidosis and fetal
hypoxia.
45. Answer: A. Vertex presentation. Vertex presentation (flexion of the fetal head) is the
optimal presentation for passage through the birth canal. Transverse lie is an unacceptable fetal
position for vaginal birth and requires a C-section. Frank breech presentation, in which the buttocks
present first, can be a difficult vaginal delivery. Posterior positioning of the fetal head can make it
difficult for the fetal head to pass under the maternal symphysis pubis.
46. Answer: D. Oxygenation. Oxygenation of the fetus may be indirectly assessed through
fetal monitoring by closely examining the fetal heart rate strip. Accelerations in the fetal heart rate
strip indicate good oxygenation, while decelerations in the fetal heart rate sometimes indicate poor
fetal oxygenation.
47. Answer: C. Active phase. Cervical dilation occurs more rapidly during the active phase
than any of the previous phases. The active phase is characterized by cervical dilation that
progresses from 4 to 7 cm. The preparatory, or latent, phase begins with the onset of regular uterine
contractions and ends when rapid cervical dilation begins. Transition is defined as cervical dilation
beginning at 8 cm and lasting until 10 cm or complete dilation.
48. Answer: C. Perform a pelvic examination.  A complaint of rectal pressure usually
indicates a low presenting fetal part, signaling imminent delivery. The nurse should perform a pelvic
examination to assess the dilation of the cervix and station of the presenting fetal part.
49. Answer: C. Passageway, contractions, placental position and function, psychological
response. The five essential factors (5 P’s) are passenger (fetus), passageway (pelvis), powers
(contractions), placental position and function, and psyche (psychological response of the mother).
50. Answer: A. Fetal body part that enters the maternal pelvis first. Presentation is the fetal
body part that enters the pelvis first; it’s classified by the presenting part; the three main
presentations are cephalic/occipital, breech, and shoulder. The relationship of the presenting fetal
part to the maternal pelvis refers to fetal position. The relationship of the long axis to the fetus to the
long axis of the mother refers to fetal lie; the three possible lies are longitudinal, transverse, and
oblique.
51. Answer: C. Uterine rupture. Uterine rupture is a medical emergency that may occur
before or during labor. Signs and symptoms typically include abdominal pain that may ease after
uterine rupture, vomiting, vaginal bleeding, hypovolemic shock, and fetal distress. With placental
abruption, the client typically complains of vaginal bleeding and constant abdominal pain.
52. Answer: A. Fetal presenting part is 1 cm above the ischial spines. Station of – 1 indicates
that the fetal presenting part is above the ischial spines and has not yet passed through the pelvic
inlet.  A station of zero would indicate that the presenting part has passed through the inlet and is at
the level of the ischial spines or is engaged.  Passage through the ischial spines with internal rotation
would be indicated by a plus station, such as + 1.  Progress of effacement is referred to by
percentages with 100% indicating full effacement and dilation by centimeters (cm) with 10 cm
indicating full dilation.
53. Answer: D. Variability averages between 6 – 10 BPM. Variability indicates a well
oxygenated fetus with a functioning autonomic nervous system. FHR should accelerate with fetal
movement.  Baseline range for the FHR is 120 to 160 beats per minute.  Late deceleration patterns
are never reassuring, though early and mild variable decelerations are expected, reassuring findings.
10
8
54. Answer: B. Stop the Pitocin. Late deceleration patterns noted are most likely related to
alteration in uteroplacental perfusion associated with the strong contractions described. The
immediate action would be to stop the Pitocin infusion since Pitocin is an oxytocic which stimulates
the uterus to contract. The woman is already in an appropriate position for uteroplacental perfusion.
Elevation of her legs would be appropriate if hypotension were present. Oxygen is appropriate but
not the immediate action.
55. Answer: D. Hypotension. Epidural anesthesia can lead to vasodilation and a drop in blood
pressure that could interfere with adequate placental perfusion.  The woman must be well hydrated
before and during epidural anesthesia to prevent this problem and maintain an adequate blood
pressure.  Headache is not a side effect since the spinal fluid is not disturbed by this anesthetic as it
would be with a low spinal (saddle block) anesthesia; 2 is an effect of epidural anesthesia but is not
the most harmful.  Respiratory depression is a potentially serious complication.
56. Answer: D. Uses the peribottle to rinse upward into her vagina. Responses A, B, and
C are all appropriate measures. The peri bottle should be used in a backward direction over the
perineum. The flow should never be directed upward into the vagina since debris would be forced
upward into the uterus through the still-open cervix.
57. Answer: C. Massage the fundus every hour for the first 24 hours following birth. The
fundus should be massaged only when boggy or soft.  Massaging a firm fundus could cause it to
relax.  Responses A, B, and D are all effective measures to enhance and maintain contraction of the
uterus and to facilitate healing.
58. Answer: C. Vacillate between the desire to have her own nurturing needs met and the
need to take charge of her own care and that of her newborn. One week after birth the woman
should exhibit behaviors characteristic of the taking-hold stage as described in response C. This
stage lasts for as long as 4 to 5 weeks after birth. Responses A and B are characteristic of the taking-
in stage, which lasts for the first few days after birth. Response D reflects the letting-go stage, which
indicates that psychosocial recovery is complete.
59. Answer: D. Take the baby back to the nursery, reassuring the woman that her rest is a
priority at this time. Response A does not take into consideration the need for the new mother to be
nurtured and have her needs met during the taking-in stage.  The behavior described is typical of this
stage and not a reflection of ineffective attachment unless the behavior persists.  Mothers need to
reestablish their own well-being in order to effectively care for their baby.
60. Answer: A. Having the children choose or make a gift to give to the new baby upon its
arrival home. Special time should be set aside just for the other children without interruption from
the newborn.  Someone other than the mother should carry the baby into the home so she can give
full attention to greeting her other children.  Children should be actively involved in the care of the
baby according to their ability without overwhelming them.

1. A nursing instructor is conducting lecture and is reviewing the functions of the female
reproductive system. She asks Mark to describe the follicle-stimulating hormone (FSH) and
the luteinizing hormone (LH). Mark accurately responds by stating that:

A. FSH and LH are released from the anterior pituitary gland.


B. FSH and LH are secreted by the corpus luteum of the ovary
C. FSH and LH are secreted by the adrenal glands
D. FSH and LH stimulate the formation of milk during pregnancy.
2. A nurse is describing the process of fetal circulation to a client during a prenatal visit.
The nurse accurately tells the client that fetal circulation consists of:

A. Two umbilical veins and one umbilical artery


B. Two umbilical arteries and one umbilical vein
C. Arteries carrying oxygenated blood to the fetus
D. Veins carrying deoxygenated blood to the fetus

10
9
3. During a prenatal visit at 38 weeks, a nurse assesses the fetal heart rate. The nurse
determines that the fetal heart rate is normal if which of the following is noted?

A. 80 BPM
B. 100 BPM
C. 150 BPM
D. 180 BPM
4. A client arrives at a prenatal clinic for the first prenatal assessment. The client tells a
nurse that the first day of her last menstrual period was September 19th, 2013. Using
Naegele’s rule, the nurse determines the estimated date of confinement as:

A. July 26, 2013


B. June 12, 2014
C. June 26, 2014
D. July 12, 2014
5. A nurse is collecting data during an admission assessment of a client who is pregnant
with twins. The client has a healthy 5-year old child that was delivered at 37 weeks and
tells the nurse that she doesn’t have any history of abortion or fetal demise. The nurse
would document the GTPAL for this client as:

A. G = 3, T = 2, P = 0, A = 0, L =1
B. G = 2, T = 0, P = 1, A = 0, L =1
C. G = 1, T = 1. P = 1, A = 0, L = 1
D. G = 2, T = 0, P = 0, A = 0, L = 1
6. A nurse is performing an assessment of a primipara who is being evaluated in a clinic
during her second trimester of pregnancy. Which of the following indicates an abnormal
physical finding necessitating further testing?

A. Consistent increase in fundal height


B. Fetal heart rate of 180 BPM
C. Braxton hicks contractions
D. Quickening
7. A nurse is reviewing the record of a client who has just been told that a pregnancy test is
positive. The physician has documented the presence of a Goodell’s sign. The nurse
determines this sign indicates:

A. A softening of the cervix


B. A soft blowing sound that corresponds to the maternal pulse during auscultation of the
uterus.
C. The presence of hCG in the urine
D. The presence of fetal movement
8. A nursing instructor asks a nursing student who is preparing to assist with the
assessment of a pregnant client to describe the process of quickening. Which of the
following statements if made by the student indicates an understanding of this term?

A. “It is the irregular, painless contractions that occur throughout pregnancy.”


B. “It is the soft blowing sound that can be heard when the uterus is auscultated.”
C. “It is the fetal movement that is felt by the mother.”
D.  “It is the thinning of the lower uterine segment.”

11
0
9. A nurse midwife is performing an assessment of a pregnant client and is assessing the
client for the presence of ballottement. Which of the following would the nurse implement to
test for the presence of ballottement?

A. Auscultating for fetal heart sounds


B. Palpating the abdomen for fetal movement
C. Assessing the cervix for thinning
D. Initiating a gentle upward tap on the cervix
10. A nurse is assisting in performing an assessment on a client who suspects that she is
pregnant and is checking the client for probable signs of
pregnancy. Select all probable signs of pregnancy.

A. Uterine enlargement
B. Fetal heart rate detected by nonelectric device
C. Outline of the fetus via radiography or ultrasound
D. Chadwick’s sign
E. Braxton Hicks contractions
F. Ballottement
11. A pregnant client calls the clinic and tells a nurse that she is experiencing leg cramps
and is awakened by the cramps at night. To provide relief from the leg cramps, the nurse
tells the client to:

A. Dorsiflex the foot while extending the knee when the cramps occur
B. Dorsiflex the foot while flexing the knee when the cramps occur
C. Plantar flex the foot while flexing the knee when the cramps occur
D. Plantar flex the foot while extending the knee when the cramps occur.
12. A nurse is providing instructions to a client in the first trimester of pregnancy regarding
measures to assist in reducing breast tenderness. The nurse tells the client to:

A. Avoid wearing a bra


B. Wash the nipples and areola area daily with soap, and massage the breasts with lotion.
C. Wear tight-fitting blouses or dresses to provide support
D. Wash the breasts with warm water and keep them dry
13. A pregnant client in the last trimester has been admitted to the hospital with a diagnosis
of severe preeclampsia. A nurse monitors for complications associated with the diagnosis
and assesses the client for:

A. Any bleeding, such as in the gums, petechiae, and purpura.


B. Enlargement of the breasts
C. Periods of fetal movement followed by quiet periods
D. Complaints of feeling hot when the room is cool
14. A client in the first trimester of pregnancy arrives at a health care clinic and reports that
she has been experiencing vaginal bleeding. A threatened abortion is suspected, and the
nurse instructs the client regarding management of care. Which statement, if made by the
client, indicates a need for further education?

A. “I will maintain strict bedrest throughout the remainder of pregnancy.”


B. “I will avoid sexual intercourse until the bleeding has stopped, and for 2 weeks following
the last evidence of bleeding.”

11
1
C. “I will count the number of perineal pads used on a daily basis and note the amount and
color of blood on the pad.”
D. “I will watch for the evidence of the passage of tissue.”
15. A prenatal nurse is providing instructions to a group of pregnant client regarding
measures to prevent toxoplasmosis. Which statement if made by one of the clients
indicates a need for further instructions?

A. “I need to cook meat thoroughly.”


B. “I need to avoid touching mucous membranes of the mouth or eyes while handling raw
meat.”
C. “I need to drink unpasteurized milk only.”
D. “I need to avoid contact with materials that are possibly contaminated with cat feces.”
16. A homecare nurse visits a pregnant client who has a diagnosis of mild Preeclampsia
and who is being monitored for pregnancy induced hypertension (PIH). Which assessment
finding indicates a worsening of the Preeclampsia and the need to notify the physician?

A. Blood pressure reading is at the prenatal baseline


B. Urinary output has increased
C. The client complains of a headache and blurred vision
D. Dependent edema has resolved
17. A nurse implements a teaching plan for a pregnant client who is newly diagnosed with
gestational diabetes. Which statement if made by the client indicates a need for further
education?

A. “I need to stay on the diabetic diet.”


B. “I will perform glucose monitoring at home.”
C. “I need to avoid exercise because of the negative effects of insulin production.”
D. “I need to be aware of any infections and report signs of infection immediately to my health
care provider.”
18. A primigravida is receiving magnesium sulfate for the treatment of pregnancy induced
hypertension (PIH). The nurse who is caring for the client is performing assessments every
30 minutes. Which assessment finding would be of most concern to the nurse?

A. Urinary output of 20 ml since the previous assessment


B. Deep tendon reflexes of 2+
C. Respiratory rate of 10 BPM
D. Fetal heart rate of 120 BPM
19. A nurse is caring for a pregnant client with Preeclampsia. The nurse prepares a plan of
care for the client and documents in the plan that if the client progresses from
Preeclampsia to eclampsia, the nurse’s first action is to:

A. Administer magnesium sulfate intravenously


B. Assess the blood pressure and fetal heart rate
C. Clean and maintain an open airway
D. Administer oxygen by face mask
20. A nurse is monitoring a pregnant client with pregnancy induced hypertension who is at
risk for Preeclampsia. The nurse checks the client for which specific signs of Preeclampsia
(select all that apply)?

A. Elevated blood pressure


11
2
B. Negative urinary protein
C. Facial edema
D. Increased respirations
21. Rho (D) immune globulin (RhoGAM) is prescribed for a woman following delivery of a
newborn infant and the nurse provides information to the woman about the purpose of the
medication. The nurse determines that the woman understands the purpose of the
medication if the woman states that it will protect her next baby from which of the following?

A. Being affected by Rh incompatibility


B. Having Rh positive blood
C. Developing a rubella infection
D. Developing physiological jaundice
22. A pregnant client is receiving magnesium sulfate for the management of preeclampsia.
A nurse determines the client is experiencing toxicity from the medication if which of the
following is noted on assessment?

A. Presence of deep tendon reflexes


B. Serum magnesium level of 6 mEq/L
C. Proteinuria of +3
D. Respirations of 10 per minute
23. A woman with preeclampsia is receiving magnesium sulfate. The nurse assigned to
care for the client determines that the magnesium therapy is effective if:

A. Ankle clonus in noted


B. The blood pressure decreases
C. Seizures do not occur
D. Scotomas are present
24. A nurse is caring for a pregnant client with severe preeclampsia who is receiving IV
magnesium sulfate. Select all nursing interventions that apply in the care for the client.

A. Monitor maternal vital signs every 2 hours


B. Notify the physician if respirations are less than 18 per minute.
C. Monitor renal function and cardiac function closely
D. Keep calcium gluconate on hand in case of a magnesium sulfate overdose
E. Monitor deep tendon reflexes hourly
F. Monitor I and O’s hourly
G. Notify the physician if urinary output is less than 30 ml per hour.
25. In the 12th week of gestation, a client completely expels the products of conception.
Because the client is Rh negative, the nurse must:

A. Administer RhoGAM within 72 hours


B. Make certain she receives RhoGAM on her first clinic visit
C. Not give RhoGAM, since it is not used with the birth of a stillborn
D. Make certain the client does not receive RhoGAM, since the gestation only lasted 12 weeks.
26. In a lecture on sexual functioning, the nurse plans to include the fact that ovulation
occurs when the:

A. Oxytocin is too high


B. Blood level of LH is too high
C. Progesterone level is high
D. Endometrial wall is sloughed off.
11
3
27. The chief function of progesterone is the:

A. Development of the female reproductive system


B. Stimulation of the follicles for ovulation to occur
C. Preparation of the uterus to receive a fertilized egg
D. Establishment of secondary male sex characteristics
28. The developing cells are called a fetus from the:

A. Time the fetal heart is heard


B. Eighth week to the time of birth
C. Implantation of the fertilized ovum
D. End of the send week to the onset of labor
29. After the first four months of pregnancy, the chief source of estrogen and progesterone
is the:

A. Placenta
B. Adrenal cortex
C. Corpus luteum
D. Anterior hypophysis
30. The nurse recognizes that an expected change in the hematologic system that occurs
during the 2nd trimester of pregnancy is:

A. A decrease in WBC’s
B. In increase in hematocrit
C. An increase in blood volume
D. A decrease in sedimentation rate
31. The nurse is aware than an adaptation of pregnancy is an increased blood supply to
the pelvic region that results in a purplish discoloration of the vaginal mucosa, which is
known as:

A. Ladin’s sign
B. Hegar’s sign
C. Goodell’s sign
D. Chadwick’s sign
32. A pregnant client is making her first Antepartum visit. She has a two year old son born
at 40 weeks, a 5 year old daughter born at 38 weeks, and 7 year old twin daughters born at
35 weeks. She had a spontaneous abortion 3 years ago at 10 weeks. Using the GTPAL
format, the nurse should identify that the client is:

A. G4 T3 P2 A1 L4
B. G5 T2 P2 A1 L4
C. G5 T2 P1 A1 L4
D. G4 T3 P1 A1 L4
33. An expected cardiopulmonary adaptation experienced by most pregnant women is:

A. Tachycardia
B. Dyspnea at rest
C. Progression of dependent edema
D. Shortness of breath on exertion
34. Nutritional planning for a newly pregnant woman of average height and weighing 145
pounds should include:
11
4
A. A decrease of 200 calories a day
B. An increase of 300 calories a day
C. An increase of 500 calories a day
D. A maintenance of her present caloric intake per day
35. During a prenatal examination, the nurse draws blood from a young Rh negative client
and explain that an indirect Coombs test will be performed to predict whether the fetus is at
risk for:

A. Acute hemolytic disease


B. Respiratory distress syndrome
C. Protein metabolic deficiency
D. Physiologic hyperbilirubinemia
36. When involved in prenatal teaching, the nurse should advise the clients that an
increase in vaginal secretions during pregnancy is called leukorrhea and is caused by
increased:

A. Metabolic rates
B. Production of estrogen
C. Functioning of the Bartholin glands
D. Supply of sodium chloride to the cells of the vagina
37. A 26-year old multigravida is 14 weeks’ pregnant and is scheduled for an alpha-
fetoprotein test. She asks the nurse, “What does the alpha-fetoprotein test indicate?” The
nurse bases a response on the knowledge that this test can detect:

A. Kidney defects
B. Cardiac defects
C. Neural tube defects
D. Urinary tract defects
38. At a prenatal visit at 36 weeks’ gestation, a client complains of discomfort with
irregularly occurring contractions. The nurse instructs the client to:

A. Lie down until they stop


B. Walk around until they subside
C. Time contraction for 30 minutes
D. Take 10 grains of aspirin for the discomfort
39. The nurse teaches a pregnant woman to avoid lying on her back. The nurse has based
this statement on the knowledge that the supine position can:

A. Unduly prolong labor


B. Cause decreased placental perfusion
C. Lead to transient episodes of hypotension
D. Interfere with free movement of the coccyx
40. The pituitary hormone that stimulates the secretion of milk from the mammary glands
is:

A. Prolactin
B. Oxytocin
C. Estrogen
D. Progesterone
41. Which of the following symptoms occurs with a hydatidiform mole?

11
5
A. Heavy, bright red bleeding every 21 days
B. Fetal cardiac motion after 6 weeks gestation
C. Benign tumors found in the smooth muscle of the uterus
D. “Snowstorm” pattern on ultrasound with no fetus or gestational sac
42. Which of the following terms applies to the tiny, blanched, slightly raised end
arterioles found on the face, neck, arms, and chest during pregnancy?

A. Epulis
B. Linea nigra
C. Striae gravidarum
D. Telangiectasias
43. Which of the following conditions is common in pregnant women in the 2nd trimester of
pregnancy?

A. Mastitis
B. Metabolic alkalosis
C. Physiologic anemia
D. Respiratory acidosis
44. A 21-year old client, 6 weeks’ pregnant is diagnosed with hyperemesis gravidarum.
This excessive vomiting during pregnancy will often result in which of the following
conditions?

A. Bowel perforation
B. Electrolyte imbalance
C. Miscarriage
D. Pregnancy induced hypertension (PIH)
45. Clients with gestational diabetes are usually managed by which of the following
therapies?

A. Diet
B. NPH insulin (long-acting)
C. Oral hypoglycemic drugs
D. Oral hypoglycemic drugs and insulin
46. The antagonist for magnesium sulfate should be readily available to any client receiving
IV magnesium. Which of the following drugs is the antidote for magnesium toxicity?

A. Calcium gluconate
B. Hydralazine (Apresoline)
C. Narcan
D. RhoGAM
47. Which of the following answers best describes the stage of pregnancy in which
maternal and fetal blood are exchanged?

A. Conception
B. 9 weeks’ gestation, when the fetal heart is well developed
C. 32-34 weeks gestation
D. maternal and fetal blood are never exchanged
48. Gravida refers to which of the following descriptions?

A. A serious pregnancy
B. Number of times a female has been pregnant
11
6
C. Number of children a female has delivered
D. Number of term pregnancies a female has had.
49. A pregnant woman at 32 weeks’ gestation complains of feeling dizzy and lightheaded
while her fundal height is being measured.  Her skin is pale and moist.  The nurse’s initial
response would be to:

A. Assess the woman’s blood pressure and pulse


B. Have the woman breathe into a paper bag
C. Raise the woman’s legs
D. Turn the woman on her side.
50. A pregnant woman’s last menstrual period began on April 8, 2005, and ended on April
13.  Using Naegele’s rule her estimated date of birth would be:

A. January 15, 2006


B. January 20, 2006
C. July 1, 2006
D. November 5, 2005
Answers and Rationales
1. Answer: A. FSH and LH are released from the anterior pituitary gland. FSH and LH,
when stimulated by gonadotropin-releasing hormone from the hypothalamus, are released from the
anterior pituitary gland to stimulate follicular growth and development, growth of the graafian
follicle, and production of progesterone.
2. Answer: B. Two umbilical arteries and one umbilical vein. Blood pumped by the
embryo’s heart leaves the embryo through two umbilical arteries. Once oxygenated, the blood then
is returned by one umbilical vein. Arteries carry deoxygenated blood and waste products from the
fetus, and veins carry oxygenated blood and provide oxygen and nutrients to the fetus.
3. Answer: C. 150 BPM. The fetal heart rate depends in gestational age and ranges from 160-
170 BPM in the first trimester but slows with fetal growth to 120-160 BPM near or at term. At or
near term, if the fetal heart rate is less than 120 or more than 160 BPM with the uterus at rest, the
fetus may be in distress.
4. Answer: C. June 26, 2014. Accurate use of Naegele’s rule requires that the woman have a
regular 28-day menstrual cycle. Add 7 days to the first day of the last menstrual period, subtract
three months, and then add one year to that date.
5. Answer: B. G = 2, T = 0, P = 1, A = 0, L =1. Pregnancy outcomes can be described with
the acronym GTPAL.
 “G” is Gravidity, the number of pregnancies.
 “T” is term births, the number of born at term (38 to 41 weeks).
 “P” is preterm births, the number born before 38 weeks gestation.
 “A” is abortions or miscarriages, included in “G” if before 20 weeks gestation,
included in parity if past 20 weeks AOE.
 “L” is live births, the number of births of living children.
 Therefore, a woman who is pregnant with twins and has a child has a gravida of 2.
Because the child was delivered at 37 weeks, the number of preterm births is 1, and the number of
term births is 0. The number of abortions is 0, and the number of live births is 1.
6. Answer: B. Fetal heart rate of 180 BPM. The normal range of the fetal heart rate depends
on gestational age. The heart rate is usually 160-170 BPM in the first trimester and slows with fetal
growth, near and at term, the fetal heart rate ranges from 120-160 BPM. The other options are
expected.
7. Answer: A. A softening of the cervix. In the early weeks of pregnancy the cervix becomes
softer as a result of increased vascularity and hyperplasia, which causes the Goodell’s sign.
8. Answer: C. “It is the fetal movement that is felt by the mother.” Quickening is fetal
movement and may occur as early as the 16th and 18th week of gestation, and the mother first
notices subtle fetal movements that gradually increase in intensity. Braxton Hicks contractions are

11
7
irregular, painless contractions that may occur throughout the pregnancy. A thinning of the lower
uterine segment occurs about the 6th week of pregnancy and is called Hegar’s sign.
9. Answer: D. Initiating a gentle upward tap on the cervix. Ballottement is a technique of
palpating a floating structure by bouncing it gently and feeling it rebound. In the technique used to
palpate the fetus, the examiner places a finger in the vagina and taps gently upward, causing the
fetus to rise. The fetus then sinks, and the examiner feels a gentle tap on the finger.
10. Answers: A, D, E, and F. 
 The probable signs of pregnancy include:
 Uterine Enlargement
 Hegar’s sign or softening and thinning of the uterine segment that occurs at
week 6.
 Goodell’s sign or softening of the cervix that occurs at the beginning of the
2nd month
 Chadwick’s sign or bluish coloration of the mucous membranes of the
cervix, vagina and vulva. Occurs at week 6.
 Ballottement or rebounding of the fetus against the examiner’s fingers of
palpation
 Braxton-Hicks contractions
 Positive pregnancy test measuring for hCG.
 Positive signs of pregnancy include:
 Fetal Heart Rate detected by electronic device (doppler) at 10-12 weeks
 Fetal Heart rate detected by nonelectronic device (fetoscope) at 20 weeks
AOG
 Active fetal movement palpable by the examiners
 Outline of the fetus via radiography or ultrasound
11. Answer: A. Dorsiflex the foot while extending the knee when the cramps occur. Legs
cramps occur when the pregnant woman stretches the leg and plantar flexes the foot. Dorsiflexion of
the foot while extending the knee stretches the affected muscle, prevents the muscle from
contracting, and stops the cramping.
12. Answer: D. Wash the breasts with warm water and keep them dry. The pregnant
woman should be instructed to wash the breasts with warm water and keep them dry. The woman
should be instructed to avoid using soap on the nipples and areola area to prevent the drying of
tissues. Wearing a supportive bra with wide adjustable straps can decrease breast tenderness. Tight-
fitting blouses or dresses will cause discomfort.
13. Answer: A. Any bleeding, such as in the gums, petechiae, and purpura. Severe
Preeclampsia can trigger disseminated intravascular coagulation because of the widespread damage
to vascular integrity. Bleeding is an early sign of DIC and should be reported to the M.D.
14. Answer: A. “I will maintain strict bedrest throughout the remainder of
pregnancy.” Strict bed rest throughout the remainder of pregnancy is not required. The woman is
advised to curtail sexual activities until the bleeding has ceased, and for 2 weeks following the last
evidence of bleeding or as recommended by the physician. The woman is instructed to count the
number of perineal pads used daily and to note the quantity and color of blood on the pad. The
woman also should watch for the evidence of the passage of tissue.
15. Answer: C. “I need to drink unpasteurized milk only.” All pregnant women should be
advised to do the following to prevent the development of toxoplasmosis. Women should be
instructed to cook meats thoroughly, avoid touching mucous membranes and eyes while handling
raw meat; thoroughly wash all kitchen surfaces that come into contact with uncooked meat, wash the
hands thoroughly after handling raw meat; avoid uncooked eggs and unpasteurized milk; wash fruits
and vegetables before consumption, and avoid contact with materials that possibly are contaminated
with cat feces, such as cat litter boxes, sandboxes, and garden soil.
16. Answer: C. The client complains of a headache and blurred vision. If the client
complains of a headache and blurred vision, the physician should be notified because these are signs
of worsening Preeclampsia.
17. Answer: C. “I need to avoid exercise because of the negative effects of insulin
production.” Exercise is safe for the client with gestational diabetes and is helpful in lowering the
blood glucose level.

11
8
18. Answer: C. Respiratory rate of 10 BPM. Magnesium sulfate depresses the respiratory
rate. If the respiratory rate is less than 12 breaths per minute, the physician or other health care
provider needs to be notified, and continuation of the medication needs to be reassessed. A urinary
output of 20 ml in a 30 minute period is adequate; less than 30 ml in one hour needs to be reported.
Deep tendon reflexes of 2+ are normal. The fetal heart rate is WNL for a resting fetus.
19. Answer: C. Clean and maintain an open airway. The immediate care during a seizure
(eclampsia) is to ensure a patent airway. The other options are actions that follow or will be
implemented after the seizure has ceased.
20. Answers: A Elevated blood pressure and 3 Facial edema. The three classic signs of
preeclampsia are hypertension, generalized edema, and proteinuria. Increased respirations are not a
sign of preeclampsia.
21. Answer: A. Being affected by Rh incompatibility. Rh incompatibility can occur when an
Rh-negative mom becomes sensitized to the Rh antigen. Sensitization may develop when an Rh-
negative woman becomes pregnant with a fetus who is Rh positive. During pregnancy and at
delivery, some of the baby’s Rh positive blood can enter the maternal circulation, causing the
woman’s immune system to form antibodies against Rh positive blood. Administration of Rho(D)
immune globulin prevents the woman from developing antibodies against Rh positive blood by
providing passive antibody protection against the Rh antigen.
22. Answer: D. Respirations of 10 per minute. Magnesium toxicity can occur from
magnesium sulfate therapy. Signs of toxicity relate to the central nervous system depressant effects
of the medication and include respiratory depression, loss of deep tendon reflexes, and a sudden
drop in the fetal heart rate and maternal heart rate and blood pressure. Therapeutic levels of
magnesium are 4-7 mEq/L. Proteinuria of +3 would be noted in a client with preeclampsia.
23. Answer: C. Seizures do not occur. For a client with preeclampsia, the goal of care is
directed at preventing eclampsia (seizures). Magnesium sulfate is an anticonvulsant, not an
antihypertensive agent. Although a decrease in blood pressure may be noted initially, this effect is
usually transient. Ankle clonus indicated hyperreflexia and may precede the onset of eclampsia.
Scotomas are areas of complete or partial blindness. Visual disturbances, such as scotomas, often
precede an eclamptic seizure.
24. Answers: C, D, E, F, and G. When caring for a client receiving magnesium sulfate therapy,
the nurse would monitor maternal vital signs, especially respirations, every 30-60 minutes and notify
the physician if respirations are less than 12, because this would indicate respiratory depression.
Calcium gluconate is kept on hand in case of magnesium sulfate overdose, because calcium
gluconate is the antidote for magnesium sulfate toxicity. Deep tendon reflexes are assessed hourly.
Cardiac and renal function is monitored closely. The urine output should be maintained at 30 ml per
hour because the medication is eliminated through the kidneys.
25. Answer: A. Administer RhoGAM within 72 hours. RhoGAM is given within 72 hours
postpartum if the client has not been sensitized already.
26. Answer: B. Blood level of LH is too high.  It is the surge of LH secretion in mid cycle that
is responsible for ovulation.
27. Answer: C. Preparation of the uterus to receive a fertilized egg. Progesterone stimulates
differentiation of the endometrium into a secretory type of tissue.
28. Answer: B. Eighth week to the time of birth.  In the first 7-14 days the ovum is known as
a blastocyst; it is called an embryo until the eighth week; the developing cells are then called a fetus
until birth.
29. Answer: A. Placenta. When placental formation is complete, around the 16th week of
pregnancy; it produces estrogen and progesterone.
30. Answer: C. An increase in blood volume. The blood volume increases by approximately
40-50% during pregnancy. The peak blood volume occurs between 30 and 34 weeks of gestation.
The hematocrit decreases as a result of the increased blood volume.
31. Answer: D. Chadwick’s sign. A purplish color results from the increased vascularity and
blood vessel engorgement of the vagina.
32. Answer: C. G5 T2 P1 A1 L4. 5 pregnancies; 2 term births; twins count as 1; one abortion;
4 living children.
33. Answer: D. Shortness of breath on exertion. This is an expected cardiopulmonary
adaptation during pregnancy; it is caused by an increased ventricular rate and elevated diaphragm.

11
9
34. Answer: B. An increase of 300 calories a day. This is the recommended caloric increase
for adult women to meet the increased metabolic demands of pregnancy.
35. Answer: A. Acute hemolytic disease. When an Rh negative mother carries an Rh positive
fetus there is a risk for maternal antibodies against Rh positive blood; antibodies cross the placenta
and destroy the fetal RBC’s.
36. Answer: B. Production of estrogen. The increase of estrogen during pregnancy causes
hyperplasia of the vaginal mucosa, which leads to increased production of mucus by the
endocervical glands. The mucus contains exfoliated epithelial cells.
37. Answer: C. Neural tube defects. The alpha-fetoprotein test detects neural tube defects and
Down syndrome.
38. Answer: B. Walk around until they subside. Ambulation relieves Braxton Hicks.
39. Answer: B. Cause decreased placental perfusion. This is because impedance of venous
return by the gravid uterus, which causes hypotension and decreased systemic perfusion.
40. Answer: A. Prolactin. Prolactin is the hormone from the anterior pituitary gland that
stimulates mammary gland secretion. Oxytocin, a posterior pituitary hormone, stimulates the uterine
musculature to contract and causes the “let down” reflex.
41. Answer: D. “Snowstorm” pattern on ultrasound with no fetus or gestational sac. The
chorionic villi of a molar pregnancy resemble a snowstorm pattern on ultrasound. Bleeding with a
hydatidiform mole is often dark brown and may occur erratically for weeks or months.
42. Answer: D. Telangiectasias. The dilated arterioles that occur during pregnancy are due to
the elevated level of circulating estrogen. The linea nigra is a pigmented line extending from the
symphysis pubis to the top of the fundus during pregnancy.
43. Answer: C. Physiologic anemia. Hemoglobin and hematocrit levels decrease during
pregnancy as the increase in plasma volume exceeds the increase in red blood cell production.
44. Answer: B. Electrolyte imbalance. Excessive vomiting in clients with hyperemesis
gravidarum often causes weight loss and fluid, electrolyte, and acid-base imbalances.
45. Answer: A. Diet. Clients with gestational diabetes are usually managed by diet alone to
control their glucose intolerance. Oral hypoglycemic agents are contraindicated in pregnancy. NPH
isn’t usually needed for blood glucose control for GDM.
46. Answer: A. Calcium gluconate. Calcium gluconate is the antidote for magnesium toxicity.
Ten ml of 10% calcium gluconate is given IV push over 3-5 minutes. Hydralazine is given for
sustained elevated blood pressures in preeclamptic clients.
47. Answer: D. maternal and fetal blood are never exchanged. Only nutrients and waste
products are transferred across the placenta. Blood exchange only occurs in complications and some
medical procedures accidentally.
48. Answer: B. Number of times a female has been pregnant. Gravida refers to the number
of times a female has been pregnant, regardless of pregnancy outcome or the number of neonates
delivered.
49. Answer: D. Turn the woman on her side. During a fundal height measurement the woman
is placed in a supine position.  This woman is experiencing supine hypotension as a result of uterine
compression of the vena cava and abdominal aorta.  Turning her on her side will remove the
compression and restore cardiac output and blood pressure.  Then vital signs can be assessed. 
Raising her legs will not solve the problem since pressure will still remain on the major abdominal
blood vessels, thereby continuing to impede cardiac output.  Breathing into a paper bag is the
solution for dizziness related to respiratory alkalosis associated with hyperventilation.
50. Answer: A. January 15, 2006. Naegele’s rule requires subtracting 3 months and adding 7
days and 1 year if appropriate to the first day of a pregnant woman’s last menstrual period.  When
this rule, is used with April 8, 2005, the estimated date of birth is January 15, 2006.

12
0

You might also like